Você está na página 1de 296

APOSTILA

AVALIAÇÕES
PROFMAT
MA11
MA12
MA13
MA14

2011-2014
MA11
AVALIAÇÕES

MA11

2011-2013
MA11
-
2011
MA11 — Números, conjuntos e funções elementares – Prova 1 – 2011

Questão 1.
Um pequeno barco a vela, com 7 tripulantes, deve atravessar o oceano em 42 dias. Seu suprimento de água potável
permite a cada pessoa dispor de 3,5 litros de água por dia (e é o que os tripulantes fazem). Após 12 dias de viagem,
o barco encontra 3 náufragos numa jangada e os acolhe. Pergunta-se:

(1.0) (a) Quantos litros de água por dia caberão agora a cada pessoa se a viagem prosseguir como antes?

(1.0) (b) Se os 10 ocupantes de agora continuarem consumindo 3,5 litros de água cada um, em quantos dias, no máximo,
será necessário encontrar uma ilha onde haja água?

Questão 2.

(1.0) (a) Quais são os valores de y para os quais existe uma função quadrática f : R → R tal que f (1) = 3, f (2) = 5 e
f (3) = y?

(1.0) (b) Tome y = 9 e determine a função quadrática correspondente. Justifique seus argumentos.

Questão 3.

(1.0) (a) Seja f : A → B uma função. Dê as definições de f (X) e f −1 (Y ), para X ⊂ A e Y ⊂ B. Se f : R → R é dada
por f (x) = 2x2 + 3x + 4, determine os conjuntos f (R) e f −1 (3).

(1.0) (b) Seja f : A → B uma função. Prove que f (X ∪ Y ) = f (X) ∪ f (Y ), quaisquer que sejam X, Y ⊂ A. Dê um
exemplo em que f (X ∩ Y ) 6= f (X) ∩ f (Y ).

Questão 4.

(0.5) (a) Se r 6= 0 é um número racional, prove que r 2 é irracional.

(0.5) (b) Dado qualquer número real ǫ > 0, prove que existe um número irracional α tal que 0 < α < ǫ.

(1.0) (c) Mostre que todo intervalo [a, b], com a < b, contém algum número irracional.

Questão 5.
Sejam m e n números naturais primos entre si.

(1.0) (a) Mostre que m


n é equivalente a uma fração decimal (isto é, com denominador potência de 10) se, e somente se,
n não tem fatores primos diferentes de 2 ou 5.

(1.0) (b) Mostre que se n tem outros fatores primos além de 2 ou 5 então a expansão decimal é infinita e, a partir de
um certo ponto, periódica.
PROFMAT – P1 – MA 11 – 2011

Questão 1.
Um pequeno barco a vela, com 7 tripulantes, deve atravessar o oceano em 42 dias. Seu suprimento de água potável
permite a cada pessoa dispor de 3,5 litros de água por dia (e é o que os tripulantes fazem). Após 12 dias de viagem,
o barco encontra 3 náufragos numa jangada e os acolhe. Pergunta-se:

(1.0) (a) Quantos litros de água por dia caberão agora a cada pessoa se a viagem prosseguir como antes?

(1.0) (b) Se os 10 ocupantes de agora continuarem consumindo 3,5 litros de água cada um, em quantos dias, no máximo,
será necessário encontrar uma ilha onde haja água?

UMA RESPOSTA

Uma solução concisa é a seguinte:

10 7
(a) O número de pessoas aumentou em 7 . Portanto a água disponı́vel para cada um deve ser 10 do que era antes
49
(3,5 litros), isto é, 20 = 2, 45 litros.

10
(b) As 7 pessoas teriam água pelos 30 dias restantes, mas agora há 7 vezes o número anterior de pessoas. Isso reduz
7
os dias a 10 · 30 = 21.

Outra forma de pensar é a seguinte. Primeiro calcula-se a quantidade Q de água que resta após 12 dias. Como
restam 30 dias de viagem, com 7 pessoas consumindo 3,5 litros por dia, são Q = 30×7×3, 5 litros (como a quantidade
de água é justa para os 42 dias e os primeiros 12 dias transcorreram como previsto, conclui-se que o que resta para
os outros 30 dias também é justo).

(a) Esse total deve ser consumido nos mesmos 30 dias, mas agora por 10 pessoas. Então o consumo diário de cada
7
um é Q dividido por 30 × 10, que dá 10 × 3, 5 = 2, 45 litros.

(b) Se todos consumirem 3,5 litros por dia, a cada dia transcorrido após o décimo segundo dia serão consumidos 35
litros. Portanto, após n dias restarão Q − 35n litros. Queremos saber o maior n tal que Q − 35n ≥ 0, isto é, o maior
Q Q 7
n que seja menor ou igual a 35 . Mas 35 = 30 × 10 = 21, então em 21 dias (exatamente) se esgotará o reservatório
de água.

1
PROFMAT – P1 – MA 11 – 2011

Questão 2.

(1.0) (a) Quais são os valores de y para os quais existe uma função quadrática f : R → R tal que f (1) = 3, f (2) = 5 e
f (3) = y?

(1.0) (b) Tome y = 9 e determine a função quadrática correspondente. Justifique seus argumentos.

UMA RESPOSTA

(a) Para que exista uma função quadrática f : R → R tal que f (1) = 3, f (2) = 5 e f (3) = y é necessário e suficiente
5−3 y−5
que os pontos (1, 3), (2, 5) e (3, y) não sejam colineares, isto é, que 2−1 6= 3−2 , ou seja, que y − 5 6= 2, ou ainda,
y 6= 7.

(b) Para obter os coeficientes a, b, c da função f (x) = ax2 + bx + c, deve-se resolver o sistema (nas incógnitas a, b, c)



 a+b+c = 3
4a + 2b + c = 5

 9a + 3b + c

= 9

Isto é feito de modo simples: basta subtrair a primeira equação das duas seguintes. Tem-se
(
3a + b = 2
8a + 2b = 6

Por subtração (segunda menos duas vezes a primeira), ficamos com 2a = 2, de onde sai imediatamente a = 1.
Substituindo esse valor em 3a + b = 2, obtemos b = −1, e voltando à equação a + b + c = 3 obtemos c = 3. Portanto
x2 − x + 3 é a função quadrática procurada.

Comentário: Há diversas outras formas de se resolver o problema. Por exemplo: tome primeiro a função g(x) = 1+2x,
que é a função afim tal que g(1) = 3 e g(2) = 5. Observe que f (x) = g(x) + a(x − 1)(x − 2) é uma função quadrática
que assume os mesmos valores que g nos pontos x = 1 e x = 2. Então basta achar a que faça f (3) = y. Ora,

f (3) = 1 + 2 · 3 + a(3 − 1)(3 − 2) = 7 + 2a .

y−7
Então 7 + 2a = y e, portanto, a = 2 . Por conseguinte,

y−7
f (x) = 1 + 2x + (x − 1)(x − 2)
2
responde o problema para qualquer y. Em particular, para y = 9,

f (x) = 1 + 2x + (x − 1)(x − 2) = x2 − x + 3 .

2
PROFMAT – P1 – MA 11 – 2011

Questão 3.

(1.0) (a) Seja f : A → B uma função. Dê as definições de f (X) e f −1 (Y ), para X ⊂ A e Y ⊂ B. Se f : R → R é dada
por f (x) = 2x2 + 3x + 4, determine os conjuntos f (R) e f −1 (3).

(1.0) (b) Seja f : A → B uma função. Prove que f (X ∪ Y ) = f (X) ∪ f (Y ), quaisquer que sejam X, Y ⊂ A. Dê um
exemplo em que f (X ∩ Y ) 6= f (X) ∩ f (Y ).

UMA RESPOSTA

(a) Definição da imagem de um subconjunto X de A:

f (X) = {y ∈ B; f (x) = y para algum x ∈ X} .

Definição da pré-imagem de um subconjunto Y de B:

f −1 (Y ) = {x ∈ A; f (x) ∈ Y } .

Agora consideremos a função f : R → R tal que f (x) = 2x2 + 3x + 4. Como o coeficiente de x2 é positivo, a função
b
quadrática assume seu valor mı́nimo f (− 43 ) = 23 3 23
8 para x = − 2a = − 4 . Assim, f (x) ≥ 8 para todo x ∈ R, ou seja,
f (R) ⊂ [ 23
8 , +∞). Além disso, para todo y ≥ 23 2
8 , a equação f (x) = y, ou seja, 2x + 3x + 4 = y, que equivale a
2
2x + 3x + 4 − y = 0, tem discriminante ∆ = 9 − 32 + 8y ≥ −23 + 23 = 0, logo existe(m) valor(es) de x com f (x) = y.
Assim f (R) = [ 23
8 , +∞).

f −1 (3) é o conjunto dos pontos x tais que f (x) = 3, isto é, tais que 2x2 + 3x + 4 = 3. Então é o conjunto das soluções
de 2x2 + 3x + 1 = 0, que é igual a {−1, − 12 }.

Comentário: f −1 (3) é um abuso de linguagem amplamente aceito para designar f −1 ({3}).

(b) z ∈ f (X ∪ Y ) se, e somente se, existe w ∈ X ∪ Y tal que f (w) = z, que por sua vez ocorre se, e somente se, existe
x ∈ X tal que f (x) = z ou existe y ∈ Y tal que f (y) = z, que ocorre se, e somente se, z ∈ f (X) ou z ∈ f (Y ), que
ocorre se, e somente se, z ∈ f (X) ∪ f (Y ).

Tome f : R → R com f (x) = x2 , X = [−1, 0] e Y = [0, 1]. Neste caso, X ∩ Y = {0} e f (X) = f (Y ) = [0, 1]. Logo
f (X ∩ Y ) = {f (0)} = {0} e f (X) ∩ f (Y ) = [0, 1].

3
PROFMAT – P1 – MA 11 – 2011

Questão 4.

(0.5) (a) Se r 6= 0 é um número racional, prove que r 2 é irracional.

(0.5) (b) Dado qualquer número real ǫ > 0, prove que existe um número irracional α tal que 0 < α < ǫ.

(1.0) (c) Mostre que todo intervalo [a, b], com a < b, contém algum número irracional.

UMA RESPOSTA
√ √ √
(a) Se r 2 é racional, então r 2 = pq , para p, q ∈ Z, q 6= 0. Como r 6= 0, podemos dividir por r para obter 2 = p
rq ,

de que resulta 2 racional, contradição.

√ √
2 2
(b) Escolha n um número natural maior do que ǫ . Então α = n é positivo, irracional (pelo item (a)) e
√ √
2 2
α= <√ = ǫ.
n 2/ǫ

(c) Se a ou b for irracional, não há o que provar. Se a for racional, subtraindo a de todos os números do intervalo
[a, b], ficamos com o intervalo [0, b − a]. Tomando ǫ igual a b − a no item (b), obtemos o irracional α menor do que
b − a e maior do que zero. Então a + α é irracional (se não fosse, então α seria a soma de dois racionais e, portanto,
um racional, contradizendo (b)) e pertence ao intervalo [a, b].

4
PROFMAT – P1 – MA 11 – 2011

Questão 5.
Sejam m e n números naturais primos entre si.
m
(1.0) (a) Mostre que n é equivalente a uma fração decimal (isto é, com denominador potência de 10) se, e somente se,
n não tem fatores primos diferentes de 2 ou 5.

(1.0) (b) Mostre que se n tem outros fatores primos além de 2 ou 5 então a expansão decimal é infinita e, a partir de
um certo ponto, periódica.

UMA RESPOSTA

m mp
(a) Sendo m e n primos entre si, uma fração equivalente a n deve ter a forma np (obtida multiplicando-se m e n
pelo mesmo número natural p).
mp
Os fatores primos de uma potência de 10 são 2 e 5. Se np é fração decimal para algum p então np = 10r . Logo,
np só admite fatores primos iguais a 2 ou 5, e, portanto, n também.
Reciprocamente, se n possui apenas fatores primos iguais a 2 ou 5, então podemos multiplicar n por p de forma
mp
que o resultado seja uma potência de 10 (p pode ser ou uma potência de 2 ou uma potência de 5). Com esse p, np
é uma fração decimal.

m
(b) Usando o processo tradicional da divisão continuada para transformar n em fração decimal, como há fatores
de n diferentes de 2 ou 5, em nenhuma etapa o resto da divisão é zero, logo a expansão nunca termina, ou seja, é
infinita. Além disso, os diferentes restos (diferentes de zero) que ocorrem são todos menores do que n, portanto o
número deles é no máximo n − 1. Assim, algum resto deve repetir-se e, a partir daı́, o processo se repete: os restos
se sucedem na mesma ordem anterior e, portanto, os quocientes também, o que fornece a periodicidade (observe que
o perı́odo tem, no máximo, n − 1 números).

5
MA11 — Números, conjuntos e funções elementares – Prova 2 – 2011

Questão 1.
Calcule as seguintes expressões:
 qp 

(1,0) (a) logn logn n n n n

(1,0) (b) xlog a/ log x , onde a > 0, x > 0 e a base dos logaritmos é fixada arbitrariamente.

Questão 2.
(Como caracterizar a função exponencial a partir da função logaritmo.) Seja f : R → R uma função crescente, tal
que f (x + y) = f (x) · f (y) para quaisquer x, y ∈ R. Prove as seguintes afirmações:

(1,0) (a) f (x) > 0 para todo x ∈ R e f (1) > 1.

(1,0) (b) Pondo a = f (1), a função g : R → R definida por g(x) = loga f (x) é linear. (Use o Teorema Fundamental da
Proporcionalidade.)

(0,5) (c) Para todo x ∈ R, g(x) = x, onde g é a função definida no item (b).

(0,5) (d) f (x) = ax para todo x ∈ R.

Questão 3.

(1,0) (a) 24h após sua administração, a quantidade de uma droga no sangue reduz-se a 10% da inicial. Que percentagem
resta 12h após a administração? Justifique sua resposta, admitindo que o decaimento da quantidade de droga
no sangue é exponencial.

(1,0) (b) Em quanto tempo a quantidade de droga no organismo se reduz a 50% da dose inicial?

(0,5) (c) Se a mesma droga for administrada em duas doses de 10 mg com um intervalo de 12h, qual é a quantidade
presente no organismo após 24h da primeira dose?

(Questão 4 na próxima página)


MA11 — Números, conjuntos e funções elementares – Prova 2 – 2011

Questão 4.

(1,0) (a) Usando as fórmulas para cos(x + y) e sen(x + y), prove que

tg(x) − tg(y)
tg(x − y) =
1 + tg(x) · tg(y)

(desde que tg(x − y), tg(x) e tg(y) estejam definidas).

(1,5) (b) Levando em conta que um ângulo é máximo num certo intervalo quando sua tangente é máxima, use a fórmula
acima para resolver o seguinte problema:
Dentro de um campo de futebol, um jogador corre para a linha de fundo do time adversário ao longo de uma
reta paralela à lateral do campo que cruza a linha de fundo fora do gol (ver figura). Os postes da meta distam
a e b (com a < b) da reta percorrida por ele. Mostre que o jogador vê a meta sob ângulo máximo quando sua

distância x ao fundo do campo é igual a ab.

b
a

x
PROFMAT – AV2 – MA 11 – 2011

Questão 1.
Calcule as seguintes expressões:
 
n n √
qp
(1,0) (a) logn logn n
n

(1,0) (b) xlog a/ log x , onde a > 0, x > 0 e a base dos logaritmos é fixada arbitrariamente.

UMA SOLUÇÃO


qp
3
(a) Como n n n
n = n1/n , temos
rq
n n √ 1
logn n
n= = n−3 ,
n3
logo o valor da expressão dada é −3.

(b) Tomando logaritmo na base b que foi fixada, temos

  log a
log xlog a/ log x = · log x = log a .
log x
Como a função log é injetiva, segue-se que
xlog a/ log x = a .

1
PROFMAT – AV2 – MA 11 – 2011

Questão 2.
(Como caracterizar a função exponencial a partir da função logaritmo.) Seja f : R → R uma função crescente, tal
que f (x + y) = f (x) · f (y) para quaisquer x, y ∈ R. Prove as seguintes afirmações:

(1,0) (a) f (x) > 0 para todo x ∈ R e f (1) > 1.

(1,0) (b) Pondo a = f (1) a função g : R → R definida por g(x) = loga f (x) é linear. (Use o Teorema Fundamental da
Proporcionalidade.)

(0,5) (c) Para todo x ∈ R, g(x) = x, onde g é a função definida no item (b).

(0,5) (d) f (x) = ax para todo x ∈ R.

UMA SOLUÇÃO

O objetivo desta questão é mostrar que é possı́vel caracterizar a função exponencial a partir da função logaritmo,
sem usar argumentos geométricos, como está no livro no caso de logaritmos naturais.
(a) Sendo crescente, f não é identicamente nula. Daı́ resulta que f (x) 6= 0 para todo x ∈ R, pois se existisse x0 ∈ R
com f (x0 ) = 0 terı́amos, para qualquer x ∈ R,

f (x) = f (x0 + (x − x0 )) = f (x0 ) · f (x − x0 ) = 0

e f seria identicamente nula.


Em seguida, notamos que
x x x x x
f (x) = f ( + ) = f ( ) · f ( ) = [f ( )]2 > 0
2 2 2 2 2
para todo x ∈ R.
Vamos mostrar que f (0) = 1. Como f (0) = f (0 + 0) = f (0) · f (0), então f (0) é solução positiva da equação x = x2 .
Como essa equação só tem 1 como solução positiva, a igualdade está demonstrada.
Finalmente, como f é crescente, f (1) > f (0) = 1.

(b) O Teorema Fundamental da Proporcionalidade diz que se g : R → R é crescente e satisfaz g(x + y) = g(x) + g(y)
para quaisquer x, y ∈ R, então g é linear, isto é, g(x) = cx, com c > 0. No nosso caso, temos

g(x + y) = loga f (x + y) = loga [f (x) · f (y)] = loga f (x) + loga f (y) = g(x) + g(y) ,

para quaisquer x, y ∈ R.

(c) Temos g(1) = loga f (1) = loga a = 1, portanto g(x) = x para todo x ∈ R.

(d) Como acabamos de ver, loga f (x) = x, para todo x ∈ R. Como loga ax = x e a função loga é injetiva, segue-se
que f (x) = ax .

2
PROFMAT – AV2 – MA 11 – 2011

Questão 3.

(1,0) (a) Usando as fórmulas para cos(x + y) e sen(x + y), prove que

tg(x) − tg(y)
tg(x − y) =
1 + tg(x) · tg(y)

(desde que tg(x − y), tg(x) e tg(y) estejam definidas).

(1,5) (b) Levando em conta que um ângulo é máximo num certo intervalo quando sua tangente é máxima, use a fórmula
acima para resolver o seguinte problema:
Dentro de um campo de futebol, um jogador corre para a linha de fundo do time adversário ao longo de uma
reta paralela à lateral do campo que cruza a linha de fundo fora do gol (ver figura). Os postes da meta distam
a e b (com a < b) da reta percorrida por ele. Mostre que o jogador vê a meta sob ângulo máximo quando sua

distância x ao fundo do campo é igual a ab.

b
a

UMA SOLUÇÃO

(a) A manipulação é direta:

sen(x − y) sen(x) · cos(y) − sen(y) · cos(x)


tg(x − y) = = .
cos(x − y) cos(x) · cos(y) + sen(x) · sen(y)

Dividindo o numerador e o denominador por cos(x) · cos(y) (se tg(x) e tg(y) estão definidas, cos(x) e cos(y) são não
nulos), vem
sen(x) sen(y)
cos(x) − cos(y) tg(x) − tg(y)
tg(x − y) = sen(x) sen(y) = 1 + tg(x) · tg(y) .
1 + cos(x) · cos(y)
(b) Em cada instante, o jogador vê a meta sob o ângulo α = α2 − α1 , onde α1 e α2 são os ângulos entre sua trajetória
e as retas que o ligam aos postes da meta. Temos

tg(α2 ) − tg(α1 )
tg(α) = .
1 + tg(α1) · tg(α2 )

3
a
Se x é a distância do jogador ao fundo do campo, temos tg(α1 ) = x e tg(α2 ) = xb , logo
b a
x − x b−a
tg(α) = ab
= .
1+ x2 x + ab
x

Como o numerador b − a é constante, tg(α) é máxima quando o denominador for mı́nimo. Ou seja, é preciso achar
ab
x que minimiza a expressão x + x . q √
Como a média aritmética é sempre maior do que ou igual à média geométrica, então 12 (x + ab
x )≥ x · ab
x = ab,

ou seja, o denominador é sempre maior do que ou igual a a 2 ab. A igualdade vale se e somente se os dois termos
√ √
da média são iguais, isto é, quando x = ab. Portanto x + ab
x atinge seu menor valor quando x = ab.

Obs. É possı́vel resolver a questão (b) com outros argumentos. Sejam A e B os extremos da meta, que distam a
e b da linha do jogador, respectivamente (veja figura abaixo, à esquerda). Para cada posição P do jogador, existe
um único cı́rculo que passa por A, B e P . O centro desse cı́rculo, O, está na mediatriz dos pontos A e B (pois
b+a
AOB é triângulo isósceles), estando, portanto, a 2 de distância da linha do jogador. Os segmentos OA e OB têm
comprimento igual ao raio do cı́rculo, digamos r, cujo valor depende de P .
Pelo Teorema do Ângulo Inscrito, AÔB = 2AP̂ B. Assim, AP̂ B é máximo quando AÔB é máximo. E AÔB é
máximo quando a distância OA = OB = r é mı́nima. Mas o menor r possı́vel é aquele tal que o cı́rculo de centro
sobre a mediatriz de A e B e raio r tangencia a linha do jogador. Nessa situação, OP é perpendicular à linha do
b+a
jogador e r = 2 (ver figura abaixo, à direita).
O valor de x, neste caso, é a altura do triângulo AOB com relação à base AB (ou seja, o comprimento da apótema
da corda AB). Esse valor sai do Teorema de Pitágoras aplicado ao triângulo AOQ, em que Q é o ponto médio de
AB. Ou seja,
 2  2
2 b−a 2 a+b
x + =r = .
2 2

Dessa equação resulta a solução x = ab.

B B
O 2α O 2α
A A

α α

P x P x

4
PROFMAT – AV2 – MA 11 – 2011

Questão 4.

(1,0) (a) 24h após sua administração, a quantidade de uma droga no sangue reduz-se a 10% da inicial. Que percentagem
resta 12h após a administração? Justifique sua resposta, admitindo que o decaimento da quantidade de droga
no sangue é exponencial.

(1,0) (b) Em quanto tempo a quantidade de droga no organismo se reduz a 50% da dose inicial?

(0,5) (c) Se a mesma droga for administrada em duas doses de 10 mg com um intervalo de 12h, qual é a quantidade
presente no organismo após 24h da primeira dose?

UMA SOLUÇÃO

(a) Sendo exponencial, a quantidade de droga no organismo obedece à lei c0 at , onde a é um número entre 0 e 1, c0 é
a dose inicial (obtida da expressão para t = 0) e t é medido, por exemplo, em horas. Após 24h a quantidade se reduz
1
a 10 da inicial, isto é,
c0
c0 a24 = .
10
1 √1 ,
Portanto a24 = 10 . Daı́ segue que a12 = 10
e que

c0
c0 a12 = √ .
10

Então a quantidade de droga após 12h é a quantidade inicial dividida por 10.

(b) Para saber o tempo necessário para a redução da quantidade de droga à metade (isto é, a meia-vida da droga no
organismo), basta achar t que cumpra at = 21 . Como a24 = 1
10 implica
 s
24s 1
a =
10

a resposta é t = 24s, onde s é tal que 10−s = 2−1 . Daı́ segue que s = log10 2 e que t = 24 log10 2.

(c) A quantidade logo após a primeira dose é c0 . Após 12h ela decai para √c10
0
. Uma nova administração a eleva para
c0 1

c0 + 10 = c0 (1 + √10 ). Após mais 12h essa quantidade é dividida por 10, passando a ser
 
1 1
c0 √ + ,
10 10
logo, com c0 = 10 mg, restarão, após 24h da primeira dose,

(1 + 10) mg.

5
MA11 — Números, conjuntos e funções elementares – Prova 3 – 2011

Questão 1.

(1,0) (a) Prove isto: Se um número natural não é o quadrado de um outro número natural, sua raiz quadrada é irracional.
√ √
(1,0) (b) Mostre que 2 + 5 é irracional.

Questão 2.

(2,0) No instante em que uma pedra caiu (sem sofrer impulso inicial) ao momento em que se ouviu o som de seu
choque com a água no fundo do poço decorreram S segundos. Calcular a profundidade do poço. Dar a resposta
em função da aceleração g da gravidade e da velocidade v do som. Usar a fórmula s = g2 t2 do espaço percorrido
no tempo t por um corpo em queda livre que partiu do repouso.

Questão 3.

(2,0) Percorrendo, ao longo de uma reta horizontal, a distância d = AB em direção à base inacessı́vel de um poste
CD, nota-se (com o auxı́lio de um teodolito) que os ângulos C ÂD e C B̂D medem, respectivamente, α e β
radianos. Qual é a altura do poste CD?

α d β
A B C
MA11 — Números, conjuntos e funções elementares – Prova 3 – 2011

Questão 4.

(2,0) Um reservatório contém uma mistura de água com sal (uma salmoura), que se mantém homogênea graças a
um misturador. Num certo momento, são abertas duas torneiras, com igual capacidade. Uma despeja água no
reservatório e a outra escoa. Após 8 horas de funcionamento, verifica-se que a quantidade de sal na salmoura
reduziu-se a 80% do que era antes que as torneiras fossem abertas. Que percentagem do sal inicial permanecerá
na salmoura após 24h de abertura das torneiras?

Questão 5.
Considere a função f : [1, +∞) → R, definida por f (x) = x3 − x2 .

(1,0) (a) Defina função crescente e prove que f é crescente.

(1,0) (b) Defina função ilimitada e prove que f é ilimitada.


PROFMAT – AV3 – MA 11 – 2011

Questão 1.

(1,0) (a) Prove isto: Se um número natural não é o quadrado de um outro número natural, sua raiz quadrada é irracional.
√ √
(1,0) (b) Mostre que 2+ 5 é irracional.

UMA SOLUÇÃO

 2
p p
(a) Seja n ∈ N. Se q ∈ Q é tal que q = n, então p2 = nq 2 . Como os fatores primos de p2 e q 2 aparecem todos com
expoente par, o mesmo deve ocorrer com os fatores primos de n. Então n é o quadrado de algum número natural.

√ √
(b) Se 2+ 5 fosse racional então seu quadrado
√ √ √ √
q = ( 2 + 5)2 = 2 + 2 10 + 5 = 7 + 2 10

q−7

também seria. Mas aı́ 2 = 10 também seria racional, o que não é possı́vel, pois 10 não é o quadrado de um
número natural.

1
PROFMAT – AV3 – MA 11 – 2011

Questão 2.

(2,0) No instante em que uma pedra caiu (sem sofrer impulso inicial) ao momento em que se ouviu o som de seu
choque com a água no fundo do poço decorreram S segundos. Calcular a profundidade do poço. Dar a resposta
em função da aceleração g da gravidade e da velocidade v do som. Usar a fórmula s = g2 t2 do espaço percorrido
no tempo t por um corpo em queda livre que partiu do repouso.

DUAS SOLUÇÕES

Uma solução. O tempo S = t1 + t2 é a soma do tempo t1 que a pedra levou para chegar ao fundo mais o tempo t2
que o som levou para vir até o nı́vel da borda. Chamando de x a profundidade do poço, temos x = g2 t21 e, por outro
lado, x = vt2 = v(S − t1 ). Logo
g 2
t = v(S − t1 )
2 1
ou
gt21 + 2vt1 − 2vS = 0 ,
que é uma equação quadrática na incógnita t1 . As soluções desta equação são
p p
−2v + 4v 2 + 8gvS −2v − 4v 2 + 8gvS
, .
2g 2g
p √
A segunda é negativa e neste problema não faz sentido. A primeira é positiva, porque 4v 2 + 8gvS > 4v 2 = 2v.
Então, dividindo por 2 o numerador e o denominador da fração,
p
−v + v 2 + 2gvS
t1 = ,
g
logo
v2 vp 2
x = vt2 = v(S − t1 ) = Sv + − v + 2gvS .
g g

Outra solução. A solução é essencialmente determinada por aquilo que escolhemos como
q incógnita (t1 , t2 ou x).
2x x
Se equacionarmos diretamente em x iremos pelo seguinte caminho. Observe que t1 = g e t2 = v . Então, de
t1 + t2 = S resulta uma equação em x:
x p −1 √
+ 2g x − S = 0.
√ v
Definamos y = x. Então precisamos achar soluções positivas de
p
v −1 y 2 + 2g −1 y − S = 0 .

A única solução positiva dessa equação quadrática é


p p
− 2g −1 + 2g −1 + 4Sv −1
y= .
2v −1
Então " s #
v2 2
 
2 2 4S 4 8S
x=y = + + −2 + ,
4 g g v g2 vg
que equivale à expressão obtida na primeira solução.

2
PROFMAT – AV3 – MA 11 – 2011

Questão 3.

(2,0) Percorrendo, ao longo de uma reta horizontal, a distância d = AB em direção à base inacessı́vel de um poste
CD, nota-se (com o auxı́lio de um teodolito) que os ângulos C ÂD e C B̂D medem, respectivamente, α e β
radianos. Qual é a altura do poste CD?

α d β
A B C

UMA SOLUÇÃO

Temos CD = AC tg α = BC tg β. Como AC = BC + d, vem (BC + d)tg α = BC tg β, e daı́


tg α
BC = d ·
tg α − tg β
e
tg α tg β
CD = BC tg β = d · ,
tg α − tg β
que é a resposta para a pergunta.

3
PROFMAT – AV3 – MA 11 – 2011

Questão 4.

(2,0) Um reservatório contém uma mistura de água com sal (uma salmoura), que se mantém homogênea graças a
um misturador. Num certo momento, são abertas duas torneiras, com igual capacidade. Uma despeja água no
reservatório e a outra escoa. Após 8 horas de funcionamento, verifica-se que a quantidade de sal na salmoura
reduziu-se a 80% do que era antes que as torneiras fossem abertas. Que percentagem do sal inicial permanecerá
na salmoura após 24h de abertura das torneiras?

UMA SOLUÇÃO

Seja M0 a massa de sal existente no inı́cio da operação. Decorrido o tempo t, essa massa será M (t) = M0 at , onde
a é uma constante (0 < a < 1). Isto se justifica porque, sendo a salmoura da torneira de saı́da uma amostra da
salmoura do tanque, supostamente homogênea, a quantidade de sal que sai por unidade de tempo é proporcional à
quantidade de sal no tanque, e isto é o princı́pio que rege o decaimento exponencial.
No entanto, a constante a não precisa ser calculada para se resolver o problema. O enunciado nos diz (supondo o
tempo t medido em horas) que M (8) = M0 a8 = 0, 8M0 , logo a8 = 0, 8. Após 24 horas, a quantidade de sal é M0 a24 .
Ora, a24 = (a8 )3 = 0, 83 = 0, 512. Portanto a resposta é 51, 2%, isto é, pouco mais que a metade.

4
PROFMAT – AV3 – MA 11 – 2011

Questão 5.
Considere a função f : [1, +∞) → R, definida por f (x) = x3 − x2 .
(1,0) (a) Defina função crescente e prove que f é crescente.

(1,0) (b) Defina função ilimitada e prove que f é ilimitada.

UMA SOLUÇÃO

(a) Uma função f : X → R, definida no conjunto X ⊂ R, chama-se crescente quando, para x, y ∈ X, x < y implica
f (x) < f (y).
Em nosso caso, sejam x, y ∈ [1, +∞), com x < y. Vamos mostrar que f (y) − f (x) > 0. Temos
f (y) − f (x) = (y 3 − y 2 ) − (x3 − x2 )
= (y 3 − x3 ) − (y 2 − x2 )
= (y − x)(y 2 + xy + x2 ) − (y − x)(y + x)
> (y − x)(y 2 + x2 ) − (y − x)(y + x)
= (y − x)(y 2 − y + x2 − x)
= (y − x)(y(y − 1) + x(x − 1)) .
Como x ≥ 1, então x(x − 1) ≥ 0; e como y > x ≥ 1, então y(y − 1) > 0 e y − x > 0. Portanto f (y) − f (x) > 0.

Outra solução. Podemos definir o número positivo h = y − x, ou seja, escrever y como x + h, e provar que
f (x + h) − f (x) > 0. Temos
f (x + h) − f (x) = (x + h)3 − (x + h)2 − x3 − x2
   

= (x3 + 3x2 h + 3xh2 + h3 ) − (x2 + 2hx + h2 ) − x3 + x2


= 3x2 h + 3xh2 + h3 − 2hx − h2 .
Para mostrar que essa expressão é positiva, precisamos achar termos positivos que, somados aos negativos, resultem
em um número positivo. Então a reescrevemos:
f (x + h) − f (x) = 3x2 h + 3xh2 + h3 − 2hx − h2
= x2 h + 2xh2 + h3 + (2x2 h − 2hx) + (xh2 − h2 )
= x2 h + 2xh2 + h3 + 2hx(x − 1) + h2 (x − 1) .
Como x ≥ 1 então os dois últimos termos são maiores do que ou iguais a zero. Acrescido do fato que os três primeiros
são positivos, tem-se que f (x + h) − f (x) > 0, para qualquer x ≥ 1 e h > 0.

(b) Uma função f : X → R, definida no conjunto X ⊂ R, chama-se ilimitada quando, dado qualquer A > 0, pode-se
achar x ∈ X tal que f (x) > A. No nosso caso, f (x) > A significa x3 − x2 > A, ou seja, x3 (1 − x1 ) > A. Ora, quando
1
x > 2 já se tem 1 − > 21 . Então, para se ter x3 (1 − x1 ) > A, basta tomar um x ∈ [1, +∞) que seja maior do que
x

2 e tal que x3 · 21> A, isto é, x3 > 2A, o que se obtém simplesmente tomando x > 3 2A. Portanto, basta tomar

x > max{2, 3 2A}.

5
MA11
-
2012
MA11 – Números, conjuntos e funções elementares – AV1 – 2012

√ √ √ √
Questão 1. (2,0) Prove que se a, b, c e d são números racionais tais que a 2 + b 3 = c 2 + d 3 então a = c e
b = d.

Questão 2. (2,0) Seja f : R → R uma função crescente tal que, para todo x racional, vale f (x) = ax + b (com
a, b ∈ R constantes). Prove que se tem f (x) = ax + b também se x for irracional.

Questão 3.

(a) (1,0) Determine uma função afim f : R → R tal que g : R → R, definida por g(x) = | |f (x)| − 1|, tenha o gráfico
abaixo.

(b) (1,0) Expresse g na forma g(x) = A + α1 |x − a1 | + α2 |x − a2 | + . . . + αn |x − an |, para algum n, explicitando os valores


de A, α1 , . . . , αn .

-3 -2 -1 0
X

Questão 4. (2,0) Ache uma fração ordinária igual ao número real α = 3, 757575 . . .

Questão 5. Considere as seguintes possibilidades a respeito das funções afins f, g : R → R, em que f (x) = ax + b e
g(x) = cx + d.

A) f (x) = g(x) para todo x ∈ R.

B) f (x) 6= g(x) seja qual for x ∈ R.

C) Existe um único x ∈ R tal que f (x) = g(x).

Com essas informações,

(i) (1,0) Exprima cada uma das possibilidades acima por meio de relações entre os coeficientes a, b, c e d.

(ii) (1,0) Interprete geometricamente cada uma dessas 3 possibilidades usando os gráficos de f e g.
AV1 - MA 11 - 2012

Questão 1.
√ √ √ √
Prove que se a, b, c e d são números racionais tais que a 2 + b 3 = c 2 + d 3 então a = c e b = d.

UMA SOLUÇÃO

√ √ √ √ √ √
A igualdade a 2 + b 3 = c 2 + d 3 implica que ( a − c) 2 = (d − b) 3. Suponha que tenhamos ( a, b) 6= (c, d).
Então teremos a 6= c ou b 6= d. Digamos que b 6= d (o caso a 6= c é análogo). Neste caso podemos dividir ambos os
lados por d − b, e teremos √
a−c 3
= √ .
d−b 2
p
Como a, b, c, d são todos racionais, o lado esquerdo é racional e igual a alguma fração irredutível q . Mas aí teríamos

3q2 = 2p2 ,

o que é impossível, pois o lado esquerdo tem um número par de fatores 2 e o lado direito tem um número ímpar
(ou: o lado esquerdo tem um número ímpar de fatores 3 e o lado direito tem um número par).

1
AV1 - MA 11 - 2012

Questão 2.

Seja f : R → R uma função crescente tal que, para todo x racional, vale f ( x ) = ax + b (com a, b ∈ R
constantes). Prove que se tem f ( x ) = ax + b também se x for irracional.

UMA SOLUÇÃO

Dado x irracional, podemos achar r e s racionais com r < x < s, sendo s − r tão pequeno quanto desejemos.
Como f é crescente, daí vem f (r ) < f ( x ) < f (s), ou seja, ar + b < f ( x ) < as + b. Como f é crescente, então a > 0,
logo podemos subtrair b de cada termo e dividir por a, sem alterar a direção das desigualdades:

f (x) − b
r< < s.
a
f ( x )−b
Como r e s podem ser escolhidos tão próximos de x quanto desejemos, isto nos obriga a ter a = x e, portanto,
f ( x ) = ax + b.

2
AV1 - MA 11 - 2012

Questão 3.

(a) Determine uma função afim f : R → R tal que g : R → R, definida por g( x ) = | | f ( x )| − 1|, tenha o gráfico
abaixo.

(b) Expresse g na forma g( x ) = A + α1 | x − a1 | + α2 | x − a2 | + . . . + αn | x − an |, para algum n, explicitando os


valores de A, α1 , . . . , αn .
Y

-3 -2 -1 0
X

UMA SOLUÇÃO

(a)
Observação: Em princípio não é necessário “deduzir” quem é f , basta apresentar uma função candidata e verificar.
No entanto, dois argumentos para obtê-la seguem abaixo.

Primeiro argumento: No trecho afim mais à direita, vale g( x ) = 2x + 2. Portanto para x ≥ −1, vale, || f ( x )| − 1| =
2x + 2. Então, no intervalo (−1, ∞), a expressão | f ( x )| − 1 não se anula, logo ou é sempre negativa, e neste caso
ter-se-á || f ( x )| − 1| = −| f ( x )| + 1, ou é sempre positiva, e neste caso ter-se-á || f ( x )| − 1| = | f ( x )| − 1. No primeiro
caso, teríamos −| f ( x )| + 1 = 2x + 2, ou | f ( x )| = −1 − 2x, em particular | f (0)| = −1, o que é impossível. Então só
resta segunda opção, e | f ( x )| − 1 = 2x + 2, de onde | f ( x )| = 2x + 3, para x ≥ −1. Concluímos que f ( x ) = 2x + 3
ou f ( x ) = −2x − 3. Ambas as possibilidades são válidas, e escolhemos a primeira f ( x ) = 2x + 3. Aí observamos
que essa escolha de f ( x ) também funciona nos demais trechos afins.

Segundo argumento: Suponha que a taxa de variação de f seja positiva. Então, para x suficientemente afastado para
a direita da raiz de f , f é positiva e maior do que 1, de modo que || f ( x )| − 1| = f ( x ) − 1. No trecho mais à direita,
isso dá 2x + 2, e daí se conclui que f ( x ) = 2x + 3. Nos outros intervalos, basta verificar.

Verificação: Para verificar que g( x ) = || f ( x )| − 1| olha-se a coincidência das funções em cada trecho afim. Os dois
lados são afins nos mesmos intervalos: (−∞, −2], [−2, − 23 ], [− 23 , −1] e [1, ∞). Logo basta verificar a coincidência
entre as funções em dois pontos de cada intervalo. Basta, portanto, verificar que coincidem em −3, −2, − 32 , −1, 0, o
que pode ser feito facilmente.

3
(b) É natural tomar a1 = −2, a2 = − 32 e a3 = −1. Então buscamos escrever

3
g ( x ) = A + α | x + 2| + β | x + | + γ | x + 1| .
2

Impondo g(0) = 2, g(−1) = 0, g(− 23 ) = 1 e g(−2) = 0, obtemos quatro equações lineares nas incógnitas A, α, β e
γ. Resolvendo o sistema, chegamos em A = −1, α = γ = 2 e β = −2, logo na função dada por

3
x 7 → −1 + 2| x + 2| − 2| x + | + 2| x + 1| .
2
Resta ver que essa função é realmente a função g. Essa verificação é feita da mesma maneira que na questão (a).

4
AV1 - MA 11 - 2012

Questão 4.

Ache uma fração ordinária igual ao número real α = 3, 757575 . . .

UMA SOLUÇÃO

Se α é o número acima então 100α = 375, 757575 . . .. Subtraindo as duas igualdades, vem 99α = 372, 0000 . . ..
372
Logo α = 99 .

5
AV1 - MA 11 - 2012

Questão 5.

Considere as seguintes possibilidades a respeito das funções afins f , g : R → R, em que f ( x ) = ax + b e


g( x ) = cx + d.
A) f ( x ) = g( x ) para todo x ∈ R.
B) f ( x ) 6= g( x ) seja qual for x ∈ R.
C) Existe um único x ∈ R tal que f ( x ) = g( x ).
Com essas informações,

i) Exprima cada uma das possibilidades acima por meio de relações entre os coeficientes a, b, c e d.

ii) Interprete geometricamente cada uma dessas 3 possibilidades usando os gráficos de f e g.

UMA SOLUÇÃO

(i) A possibilidade A) ocorre se, e somente se, a = c e b = d. Prova: Se a = c e b = d então, para qualquer x ∈ R,
tem-se f ( x ) = ax + b = cx + d = g( x ). Por outro lado, se f ( x ) = g( x ) para qualquer x ∈ R, então, em particular,
f (0) = g(0), ou seja, a · 0 + b = c · 0 + d, isto é, b = d; além disso, f (1) = g(1), implicando a · 1 + b = c · 1 + d, ou
seja, a = c (usando que b = d).
A possibilidade B) ocorre se, e somente se, a = c e b 6= d. Prova: Se a = c e b 6= d, então f ( x ) − g( x ) =
( a − c) x + (b − d) = b − d 6= 0, para qualquer x ∈ R. Por outro lado, se f ( x ) 6= g( x ) para qualquer x ∈ R então
f ( x ) − g( x ) = ( a − c) x + (b − d) 6= 0 para qualquer x ∈ R, ou seja, ( a − c) x + (b − d) não tem raiz. Mas isto só
ocorre se a = c e b 6= d.
A possibilidade C) ocorre se, e somente se, a 6= c. Prova: Se a 6= c então f ( x ) − g( x ) = ( a − c) x + (b − d) tem
única raiz igual a da− b
−c , logo este é o único ponto x tal que f ( x ) = g ( x ). Por outro lado, se existe um único ponto x
tal que f ( x ) = g( x ) é porque a diferença f ( x ) − g( x ) = ( a − c) x + (b − d) tem uma única raiz, ou seja, a − c 6= 0.

(ii) No caso A), os gráficos de f e g são retas coincidentes. No caso B), os gráficos de f e g são retas paralelas. No
caso C), os gráficos de f e g são retas concorrentes.

6
MA11 – Números, conjuntos e funções elementares – AV2 – 2012

Questão 1. (2,0) Seja f : R → R uma função tal que f (0) = 0 e |f (x) − f (y)| = |x − y| para quaisquer x, y ∈ R.
Prove que ou f (x) = x para todo x ou então f (x) = −x seja qual for x.

Questão 2. (2,0) Dada a função quadrática f (x) = ax2 + bx + c, consideremos as funções afins g(x) = mx + t,
onde m é fixo e t será escolhido convenientemente. Prove que existe uma (única) escolha de t para a qual a equação
f (x) = g(x) tem uma, e somente uma, raiz x. Interprete este fato geometricamente em termos dos gráficos de f e g.

Questão 3. (2,0) Dados os pontos A = (3, 7), B = (4, 5), C = (5, 5) e D = (5, 3) em R2 , determine a função afim
f (x) = ax + b cujo gráfico contém três desses pontos.

Questão 4. (2,0) A população de uma cultura de bactérias, num ambiente estável e controlado, é estimada pela
área que ocupa sobre uma superfı́cie plana. Se, decorridos 20 dias, a população duplicou, então ela ficou 50% maior

(a) antes de 10 dias.

(b) ao completar 10 dias.

(c) após 10 dias.

Escolha a resposta certa e justifique sua opção.

1 1
Questão 5. (2,0) Dados números reais positivos x e y, ache α e β tais que cos x · cos y = 2 cos α + 2 cos β. Em
seguida mostre como (mediante o uso de uma tabela de funções trigonométricas) esta igualdade pode ser empregada
para reduzir o produto de dois números reais positivos quaisquer às operações de soma e divisão por 2.
AV2 - MA 11 - 2012

Questão 1. Seja f : R → R uma função tal que f (0) = 0 e | f ( x ) − f (y)| = | x − y| para quaisquer x, y ∈ R. Prove
que ou f ( x ) = x para todo x ou então f ( x ) = − x seja qual for x.

UMA SOLUÇÃO

Tomando y = 0, vemos que | f ( x )| = | x |, logo f ( x ) = ± x para todo x. Resta mostrar que não se pode ter
f ( x1 ) = x1 e f ( x2 ) = − x2 com x1 e x2 não nulos. De fato, se isto ocorresse, então

| x1 + x2 | = | x1 − (− x2 )| = | f ( x1 ) − f ( x2 )| = | x1 − x2 | .

Elevando ao quadrado ambos os lados da igualdade | x1 + x2 | = | x1 − x2 | concluiríamos que x1 x2 = − x1 x2 , isto é,


que x1 x2 = 0, o que é uma contradição com o fato de x1 e x2 serem ambos nulos.

1
AV2 - MA 11 - 2012

Questão 2. Dada a função quadrática f ( x ) = ax2 + bx + c, consideremos as funções afins g( x ) = mx + t, onde


m é fixo e t será escolhido convenientemente. Prove que existe uma (única) escolha de t para a qual a equação
f ( x ) = g( x ) tem uma, e somente uma, raiz x. Interprete este fato geometricamente em termos dos gráficos de f e g.

UMA SOLUÇÃO

A equação f ( x ) = g( x ) significa ax2 + (b − m) x + c − t = 0. Esta equação do segundo grau tem uma raiz única
( b − m )2
se, e somente se, seu discriminante (b − m)2 − 4a(c − t) é igual a zero, ou seja, se t = c − 4a (observando que
a 6= 0, já que f é quadrática).
Ao variar t, a reta gráfico de g se desloca paralelamente a si mesma e toca a parábola gráfico de f num só ponto
quando é sua tangente. Este é o valor de t que foi calculado.

2
AV2 - MA 11 - 2012

Questão 3. Dados os pontos A = (3, 7), B = (4, 5), C = (5, 5) e D = (5, 3) em R2 , determine a função afim
f ( x ) = ax + b cujo gráfico contém três desses pontos.

UMA SOLUÇÃO

As inclinações dos segmentos AB, AC e AD são, respectivamente, −2, −1 e −2. Portanto A, B e D são colineares.
O segmento CD é vertical, logo C e D não podem pertencer ao gráfico de uma função afim. Logo, além de A, B,
D só resta a possibilidade de que A, B e C sejam colineares. No entanto, AB tem inclinação −2 e BC tem inclinação
0, então A, B e C não podem ser colineares.
Assim, A, B e D são os únicos três pontos colineares dentre os quatro pontos dados. A função afim cujo gráfico
os contém é f ( x ) = ax + b tal que f (3) = 7 e f (4) = 5. Portanto 3a + b = 7 e 4a + b = 5. Daí resulta que a = −2 e
b = 13. A função procurada é f ( x ) = −2x + 13.

3
AV2 - MA 11 - 2012

Questão 4. A população de uma cultura de bactérias, num ambiente controlado, é estimada pela área que ocupa
sobre uma superfície plana e tem taxa de crescimento diária proporcional a seu tamanho. Se, decorridos 20 dias, a
população duplicou, então ela ficou 50% maior

(a) antes de 10 dias.

(b) ao completar 10 dias.

(c) após 10 dias.

Escolha a resposta certa e justifique sua opção.

UMA SOLUÇÃO

Se p0 é a população original, após decorridos t dias a população p = p(t) será dada por p = p0 at , onde a é uma
√ √
constante maior do que 1. Temos p0 a20 = 2p0 , logo a20 = 2. Então p(10) = p0 a10 = p0 a20 = p0 2 ≃ 1, 414p0 .
Então p(10) < 1, 5p0 , o que nos faz concluir que o crescimento de 50% será atingido após os primeiros 10 dias. A
opção correta é (c).

4
AV2 - MA 11 - 2012

1 1
Questão 5. Dados números reais positivos x e y, ache α e β tais que cos x · cos y = 2 cos α + 2 cos β. Em seguida
mostre como (mediante o uso de uma tabela de funções trigonométricas) esta igualdade pode ser empregada para
reduzir o produto de dois números reais positivos quaisquer às operações de soma e divisão por 2.

UMA SOLUÇÃO

A fórmula do cosseno de uma soma, junto com a observação de que sen(−y) = −sen y, nos dá

cos( x + y) = cos x · cos y − sen x · sen y

e
cos( x − y) = cos x · cos y + sen x · sen y ,

logo cos( x + y) + cos( x − y) = 2 cos x · cos y. Daí resulta a igualdade proposta, com α = x + y e β = x − y.
Em seguida, se a e b são números reais positivos quaisquer, dados por suas expressões decimais, deslocando as
vírgulas que separam suas partes inteiras (alteração que pode facilmente ser refeita no final), podemos supor que
esses números são ambos compreendidos entre 0 e 1. A tabela nos dá x e y tais que cos x = a e cos y = b. E a
igualdade inicial fornece ab = cos x · cos y = 21 (cos( x + y) + cos( x − y)). Na prática, é preciso (i) tomar x e y pela
tabela; (ii) calcular x + y e x − y; (iii) obter seus cossenos, também pela tabela; (iv) somar os cossenos; e (v) dividir
por 2.
Este artifício era usado pelos astrônomos antes da descoberta dos logaritmos.

5
MA11 – Números e funções reais – AV3 – 2012

√ √
Questão 1. (2,0) Sejam a, x números reais positivos, com a < x. Pondo y = 12 (x + xa ), prove que a < y < x.

Questão 2. (2,0) A imagem (ou conjunto de valores) de uma função f : R → R é o conjunto f (R) cujos elementos
são os números f (x), onde x é qualquer número real.
Determine as imagens da função afim f : R → R, f (x) = rx + s, e da função quadrática g : R → R, g(x) =
ax2 + bx + c. Discuta as possibilidades e justifique suas afirmações.

Questão 3. (2,0) Uma torneira leva x horas para encher um tanque, outra leva y horas e uma terceira enche esse
mesmo tanque em z horas. Em quanto tempo as três juntas encherão o tanque?

Questão 4. (2,0) Uma cultura de bactérias, cuja população é medida pela área que ocupa sobre uma superfı́cie
plana, ficou 64 vezes maior após 1 ano. Quantas vezes maior ela estava após 1 trimestre?

Questão 5. (2,0) Seja r o raio da circunferência sobre a qual estão os vértices do triângulo ABC. Se a é a medida
b prove que sen A
do lado oposto ao ângulo A, b = a (dica: baixe, do centro da circunferência, a perpendicular a BC).
2r
Conclua daı́ a Leis dos Senos.
AV3 - MA 11 - 2012

√ √
Questão 1. Sejam a, x números reais positivos, com a < x. Pondo y = 12 ( x + xa ), prove que a < y < x.

UMA SOLUÇÃO


Primeiro notamos que x é maior do que xa : a < x significa a < x2 , logo a
x < x, usando que x é positivo. Como y
a
é a média aritmética dos números x e x, dos quais x é o maior, então y < x.
Outra maneira:
1 a 1 x2
y= (x + ) < (x + ) = x ,
2 x 2 x
usando a < x2 .
Além disso, como a média aritmética de dois números diferentes é maior do que a média geométrica e como a
√ √
média geométrica de x e xa é igual a a, resulta que y > a.
Essa última desigualdade também poderia ser feita diretamente:

1 2 a2 1 a2 1 a
y2 = ( x + 2a + 2 ) = ( x2 − 2a + 2 + 4a) = a + ( x − )2 > a .
4 x 4 x 4 x

1
AV3 - MA 11 - 2012

Questão 2. A imagem (ou conjunto de valores) de uma função f : R → R é o conjunto f (R ) cujos elementos são os
números f ( x ), onde x é qualquer número real.
Determine as imagens da função afim f : R → R, f ( x ) = rx + s, e da função quadrática g : R → R, g( x ) =
ax2 + bx + c. Discuta as possibilidades e justifique suas afirmações.

UMA SOLUÇÃO

Para a função afim f , há duas possibilidades: se r = 0 então f é constante e sua imagem é o conjunto {s}, com um
só elemento. A segunda possibilidade ocorre se r 6= 0. Então f (R ) = R pois, dado qualquer y ∈ R, existe x ∈ R tal
y−s
que f ( x ) = y, ou seja, rx + s = y. Basta tomar x = r .
No caso da função quadrática g( x ) = ax2 + bx + c, há duas possibilidades para a imagem g(R ). Se a > 0 então
a imagem é a semirreta (intervalo infinito) [k, +∞) e se a < 0 então f (R ) = (−∞, k ], onde k (em ambos os casos) é
igual a
b 4ac − b2
g(− ) = .
a 4a
Justificando: se a > 0 então, tomando qualquer y ∈ [k, +∞), ou seja, y ≥ k, para achar x ∈ R tal que f ( x ) = y,
devemos mostrar que a equação ax2 + bx + c = y, isto é, ax2 + bx + c − y = 0, possui raízes reais. Isto ocorre se, e
4ac−b2
somente se, seu discriminante b2 − 4a(c − y) é maior do que ou igual a 0. Como y ≥ 4a , isto sempre ocorre.
O que acabamos de mostrar foi que [k, +∞) ⊂ g(R ). Para ver que g(R ) ⊂ [k, +∞), basta observar que, em
virtude da forma canônica g( x ) = a( x − m)2 + k, quando a > 0 todos os valores g( x ) são maiores do que ou iguais
a k.
A discussão do caso a < 0 é inteiramente análoga.

2
AV3 - MA 11 - 2012

Questão 3. Uma torneira leva x horas para encher um tanque, outra leva y horas e uma terceira enche esse mesmo
tanque em z horas. Em quanto tempo as três juntas encherão o tanque?

UMA SOLUÇÃO

1 1 1
As três torneiras separadamente, abertas durante uma hora, encherão respectivamente as frações x, y e z do
1 1 1
tanque e, abertas juntas, encherão x + y + z do tanque. Logo, juntas, encherão o tanque em

1 xyz
1 1 1
=
x + y + z
yz + xz + xy

horas.

3
AV3 - MA 11 - 2012

Questão 4. Uma cultura de bactérias, cuja população é medida pela área que ocupa sobre uma superfície plana,
ficou 64 vezes maior após 1 ano. Quantas vezes maior ela estava após 1 trimestre?

UMA SOLUÇÃO

Seja p0 a população inicial. Após decorrido o tempo t, a população será p0 at = p, onde a é uma constante maior
do que 1, determinada experimentalmente. Medindo o tempo em meses, temos p = p0 a12 = 64p0 , após 1 ano.
√ √
Quer-se saber o valor de p = p0 a3 . De a12 p0 = 64p0 temos a12 = 64 e daí a3 = 4 64 = 2 2 ≃ 2, 83. Portanto, após
um trimestre, a população de bactérias estava 2, 83 vezes maior do que a população original.

4
AV3 - MA 11 - 2012

Questão 5. Seja r o raio da circunferência sobre a qual estão os vértices do triângulo ABC. Se a é a medida do lado
oposto ao ângulo A,b prove que sen Ab = a (dica: baixe, do centro da circunferência, a perpendicular a BC). Conclua
2r
daí a Leis dos Senos.

UMA SOLUÇÃO

Seja O o centro da circunferência e seja P no segmento BC tal que OP é perpendicular a BC. É sabido que o
ângulo BOC (ângulo central da corda BC) é o dobro do ângulo inscrito A b (fato conhecido como o Teorema do
Ângulo Inscrito). Como o triângulo BOC é isósceles, então OP é bissetriz e, portanto, o ângulo POC é exatamente
b Também pelo fato de BOC ser isósceles, OP é mediatriz, de forma que PC = a . Sendo OC = r e OPC
igual a A. 2
b) =
triângulo-retângulo, segue que sen ( A = a/2
PC a
OC r = 2r .
O que acabamos de provar é que o seno do ângulo em A dividido pelo comprimento do lado oposto ao vértice A
1
é igual a 2r . O mesmo argumento se aplica a B ou a C, de modo que essa razão (seno de um ângulo dividido pelo
lado oposto) é constante. Essa é a Lei dos Senos.

5
MA11
-
2013
Sociedade Brasileira de Matemática
Mestrado Profissional em Matemática em Rede Nacional

MA11 – Números e Funções Reais


Avaliação 1
13 de abril de 2013

1. Determine se as afirmações a seguir são verdadeiras ou falsas, justificando adequadamente


e em detalhes as suas respostas.

(a) A soma de dois números irracionais é um número irracional. (pontuação 1,0)


(b) O produto de dois números reais com representação decimal infinita e periódica é um
número real que não possui representação decimal finita. (pontuação 1,0)

2. Da mesma forma que se expressa um número real no sistema de numeração decimal, é


possível expressá-lo em um sistema de numeração posicional qualquer, de base β ∈ N,
β > 2. Dizemos que um número a ∈ R está expresso no sistema de base β se ele é escrito
na forma:
+∞
X
a = a0 + an β −n
n=1

em que a0 ∈ Z e os an são dígitos entre 0 e β − 1.

(a) Sejam x e y os números reais cujas representações no sistema de numeração de base


4 são dadas por 0, 321 e 0, 111 . . ., respectivamente. Determine as representações de
x e de y no sistema decimal. (pontuação 1,0)
(b) Mostre que um número racional a = m n
∈ R, com m, n ∈ Z, n 6= 0 e mdc(m, n) = 1,
possui representação finita no sistema de numeração posicional de base β se, e somente
se, o denominador n não possui fatores primos que não sejam fatores de β. (pontuação
1,0)
√ √
3. (a) Considere a função h : [0, +∞[ → R definida por h(x) = x + 2x . Usando o fato

de que a função g : [0, +∞[ → R, definida por g(x) = x é monótona crescente,
mostre que h é monótona crescente. (pontuação 0,5)
√ √
(b) Conclua, com base no item anterior, que, ∀ a ∈ R, a > 0 a equação x = a − 2x
admite uma única solução real. (pontuação 0,5)
√ √
(c) Considere a seguinte resolução para a equação x = 1 − 2x :
√ √ √ √
x = 1 − 2x ⇒ x = 1 − 2 2x + 2x ⇒ 1 + x = 2 2x ⇒

1 + 2x + x2 = 8x ⇒ x2 − 6x + 1 = 0 ⇒ x = 3 ± 2 2

Este método de resolução está correto? Justifique sua resposta. (pontuação 1,0)

4. Considere a função p : [−1, 5] → R definida por:


(
3 x − x2 se −1 6 x < 1
||x − 2| − 1| se 1 6 x 6 5

(a) Faça um esboço do gráfico de p. (pontuação 0,5)


(b) Determine todas as soluções reais da equação p(x) = 2. (pontuação 0,5)
(c) Determine todos os pontos de máximo e de mínimo locais e absolutos de p. (pontu-
ação 0,5)
(d) Faça um esboço do gráfico da função q : [−1, 2]] → R definida por:

q(x) = p(2x + 1) − 2 .

(pontuação 0,5)

5. Considere a função quadrática f : R → R, f (x) = a x2 + b x + c, com a > 0. Use a forma


canônica do trinômio de segundo grau

y = a (x − x0 )2 + y0

para mostrar que:

(a) (x0 , y0 ) é um ponto de mínimo absoluto de f ; (pontuação 1,0)


(b) a reta x = x0 é um eixo de simetria vertical do gráfico de f . (pontuação 1,0)
Sociedade Brasileira de Matemática
Mestrado Profissional em Matemática em Rede Nacional

MA11 – Números e Funções Reais


Avaliação 1 - GABARITO
13 de abril de 2013

1. Determine se as afirmações a seguir são verdadeiras ou falsas, justificando adequadamente


e em detalhes as suas respostas.

(a) A soma de dois números irracionais é um número irracional. (pontuação 1,0)


(b) O produto de dois números reais com representação decimal infinita e periódica é um
número real que não possui representação decimal finita. (pontuação 1,0)

Uma solução:
a) Falso.
Contra-exemplo: x = π e y = 1 − π são irracionais, mas x + y = 1 não é irracional.
b)Falso.
7 6 1
Contra-exemplo: x = 12 ey= 7
têm representação decimal infinita, mas x.y = 2
possui
representação decimal finita.

2. Da mesma forma que se expressa um número real no sistema de numeração decimal, é


possível expressá-lo em um sistema de numeração posicional qualquer, de base β ∈ N,
β > 2. Dizemos que um número a ∈ R está expresso no sistema de base β se ele é escrito
na forma:
+∞
X
a = a0 + an β −n
n=1

em que a0 ∈ Z e os an são dígitos entre 0 e β − 1.

(a) Sejam x e y os números reais cujas representações no sistema de numeração de base


4 são dadas por 0, 321 e 0, 111 . . ., respectivamente. Determine as representações de
x e de y no sistema decimal. (pontuação 1,0)
(b) Mostre que um número racional a = m n
∈ R, com m, n ∈ Z, n 6= 0 e mdc(m, n) = 1,
possui representação finita no sistema de numeração posicional de base β se, e somente
se, o denominador n não possui fatores primos que não sejam fatores de β. (pontuação
1,0)
Uma solução:
a) Pela definição da expressão de um número real no sistema de numeração posicional de
base β, temos que:

1 1 1 3 1 1 57
x = (0, 321)β = 3 × +2× 2 +1× 3 = + + = = 0, 890625
4 4 4 4 8 64 64

+∞
X 1
y = (0, 111 . . .)β =
k=1
4k

Portanto, a expressão acima é a soma da progressão geométrica infinita cujo termo inicial
é 14 e a razão é 14 . Essa soma converge para:

1
4 1
1 = = 0, 333 . . .
1− 4
3

b) Observamos que um número a possui representação finita no sistema de numeração


posicional de base β se, e somente se, existe um expoente k ∈ N tal que β k a ∈ N.
m
Assim, se possui representação finita no sistema de numeração posicional de base β,
n
βkm
então ∈ N para algum k ∈ N. Logo, n | β k m. Como mdc(m, n) = 1, então n | β k .
n
Portanto, n não possui fatores primos que não sejam fatores de β k . Mas estes são os
mesmos fatores primos de β.
Reciprocamente, se n não possui fatores primos que não sejam fatores de β, então n | β k ,
βkm
para um expoente k suficientemente grande. Logo, n | β k m, portanto ∈ N. Então,
n
m
possui representação finita no sistema de numeração posicional de base β.
n
√ √
3. (a) Considere a função h : [0, +∞[ → R definida por h(x) = x + 2x . Usando o fato

de que a função g : [0, +∞[ → R, definida por g(x) = x é monótona crescente,
mostre que h é monótona crescente. (pontuação 0,5)
√ √
(b) Conclua, com base no item anterior, que, ∀ a ∈ R, a > 0 a equação x = a − 2x
admite uma única solução real. (pontuação 0,5)
√ √
(c) Considere a seguinte resolução para a equação x = 1 − 2x :

√ √ √ √
x = 1 − 2x ⇒ x = 1 − 2 2x + 2x ⇒ 1 + x = 2 2x ⇒

1 + 2x + x2 = 8x ⇒ x2 − 6x + 1 = 0 ⇒ x = 3 ± 2 2

Este método de resolução está correto? Justifique sua resposta. (pontuação 1,0)
Uma solução:
√ √ √ √ √
a) Temos que h(x) = x + 2 x = (1 + 2) x = (1 + 2) g(x). Como g é crescente,
então, x1 , x2 ∈ [0, +∞[ , x1 < x2 ⇒ g(x1 ) < g(x2 ) ⇒ h(x1 ) < h(x2 ). Portanto, h é
crescente.
√ √ 2
b) A existência da solução da equação x = a− 2x é explícita: dado a ≥ 0, x = (1+a√2)2
é uma solução. Mesmo que não conseguíssemos uma solução explícita, a garantia teórica
da existência de uma solução desta equação é uma consequência da continuidade de h
e de que limx→+∞ h(x) = +∞. Assim, para todo a ∈ R, a ≥ 0, existe pelo menos
um x ∈ [0, +∞[ tal que h(x) = a. Vejamos a unicidade: suponhamos que existam
x1 x2 ∈ [0, +∞[ , x1 6= x2 tais que h(x1 ) = h(x2 ) = a. Digamos x1 < x2 . Como h
é crescente, deveríamos ter h(x1 ) < h(x2 ). Logo, existe um único x ∈ [0, +∞[ tal que
√ √
h(x) = a, isto é, x = a − 2x .
√ √
c) Pelo item anterior, a equação x = 1 − 2x admite uma única solução. Portanto, a
resolução não está correta.
√ √ √
Na primeira passagem da resolução, é verdade que x = 1 − 2x ⇒ x = 1 − 2 2x + 2x.
√ √ √
Entretanto, x = 1 − 2 2x + 2x 6⇒ x = 1 − 2x. De fato, nesta implicação, estamos
implicitamente fazendo:
√ √ 2 √
q √ √
x = 1 − 2 2x + 2x = (1 − 2x) ⇒ x = (1 − 2x)2 = 1 − 2x .

Em primeiro lugar, para que a implicação acima seja verdadeira, devemos supor que x > 0,
o que já é uma hipótese inicial para a resolução q da equação. Além disso, temos que
√ √ 2 √ √
1 − 2 2x + 2x = (1 − 2x) , mas a igualdade (1 − 2x)2 = 1 − 2x só vale se

1 − 2x > 0. Logo, a implicação acima só é verdadeira se 0 6 x 6 12 .
Portanto, nessa passagem ocorre uma inclusão de raízes estranhas à equação.

4. Considere a função p : [−1, 5] → R definida por:


(
3 x − x2 se −1 6 x < 1
||x − 2| − 1| se 1 6 x 6 5

(a) Faça um esboço do gráfico de p. (pontuação 0,5)


(b) Determine todas as soluções reais da equação p(x) = 2. (pontuação 0,5)
(c) Determine todos os pontos de máximo e de mínimo locais e absolutos de p. (pontu-
ação 0,5)
(d) Faça um esboço do gráfico da função q : [−1, 2]] → R definida por:

q(x) = p(2x + 1) − 2 .
(pontuação 0,5)

Uma solução:
a) O gráfico da função p é dado por:

-1 1 2 3 4 5 6

-1

-2

-3

-4

b) Resolvendo 3 x − x2 = 2, obtemos x = 1 ou x = 2, mas estes valores estão fora do


intervalo em que p é definida pela expressão y = 3 x − x2 . Resolvendo ||x − 2| − 1| = 2,
obtemos |x − 2| = 1 ± 2. Como não há valores de x tais que |x − 2| = −1, resta apenas
a alternativa |x − 2| = 3. Esta implica em x = 2 ± 3, portanto x = −1 ou x = 5, mas
x = −1 está fora do intervalo em que p é definida pela expressão y = ||x − 2| − 1| = 2,
portanto, a única solução da equação p(x) = 2 é x = 5. De fato, percebemos pelo gráfico
esboçado no item anterior que a reta y = 2 intercepta o gráfico de p apenas quando x = 5.

c) Analisando o gráfico, concluímos que a função p possui:


• mínimo absoluto em x = −1;
• mínimo local em x = 1;
• máximo local em x = 2;
• mínimo local em x = 3;
• máximo absoluto em x = 5.

d) Na definição da função q, a variável de p é multiplicada por 2 e somada a 1 e, em


seguida, a função p é somada à constante −2. Estas operações podem ser descritas
geometricamente por meio das seguintes funções:
• p(x), cujo gráfico foi obtido em a),
• p1 (x) = p(2x), cujo gráfico é obtido do de p(x) por uma contração horizontal de
razão 12 ,
• p2 (x) = p1 (x + 12 ) = p(2(x + 12 )) = p(2x + 1), cujo gráfico é obtido do de p1 (x)
por uma translação horizontal de 12 unidade no sentido negativo do eixo (isto é, para a
esquerda),
e, finalmente,
• q(x) = p2 (x) − 2, cujo gráfico é obtido do de p2 (x) por meio de uma translação
vertical de 2 unidades no sentido negativo do eixo (isto é, para baixo).
Portanto, o gráfico de q tem o seguinte aspecto:

-1 1 2 3 4 5 6

-1

-2

-3

-4

-5

-6
5. Considere a função quadrática f : R → R, f (x) = a x2 + b x + c, com a > 0. Use a forma
canônica do trinômio de segundo grau

y = a (x − x0 )2 + y0

para mostrar que:


a) (x0 , y0 ) é um ponto de mínimo absoluto de f ; (pontuação 1,0)
b) a reta x = x0 é um eixo de simetria vertical do gráfico de f . (pontuação 1,0)

Uma solução:
a) Temos que f (x0 ) = y0 . Além disso, para qualquer x ∈ R, x 6= x0 , temos a (x − x0 )2 > 0,
portanto:
f (x) = a (x − x0 )2 + y0 > y0 = f (x0 )
Segue que (x0 , y0 ) é ponto de mínimo absoluto de f .
b) Dado r > 0 qualquer temos:

f (x0 − r) = a r2 + y0
f (x0 + r) = a r2 + y0
Portanto, f (x0 − r) = f (x0 + r), ∀r > 0. Logo, a reta x = x0 é um eixo de simetria vertical
do gráfico de f .
Sociedade Brasileira de Matemática
Mestrado Profissional em Matemática em Rede Nacional

MA11 – Números e Funções Reais


Avaliação 2
22 de junho de 2013

1. [ 2 pontos ] Em cada um dos itens abaixo, dê, se possível, um exemplo de um polinômio


p(x) satisfazendo todas as condições dadas.
Caso o exemplo não seja possível, justifique a sua resposta.
Lembre-se que se p(x) = a0 + a1 x + a2 x2 + · · · + an−1 xn−1 + an xn então

p′ (x) = a1 + 2a2 x + · · · + (n − 1)an−1 xn−2 + nan xn−1

(a) p(1) = p(−1) = 0, p′ (0) = 1 e p(x) é de grau 2.


(b) p(1) = p(−1) = 0 e p′ (0) = 1.
(c) p(1) = p(−1) = 0, p′ (0) = 0 e p(x) é de grau 2.
(d) p(1) = p(−1) = 0, p(0) = p(2) = 1 e p(x) é de grau 2.
(e) p(1) = p(−1) = 0, p(0) = p(2) = 1 e p(x) é de grau 3.

[pontuação de cada ítem: 0,4 ponto]

2. [ 2 pontos ] Um número real x0 é raiz de multiplicidade k do polinômio p(x) se p(x) =


(x − x0 )k q(x), para algum polinômio q(x), com q(x0 ) 6= 0.
Sugestão para resolver os ítens abaixo: Use o fato de que toda função polinomial é uma
função contínua e que "Se f é uma função real contínua e f (x0 ) 6= 0, então existe uma
vizinhança de x0 em que f não se anula".

(a) Mostre que x0 é raiz de multiplicidade par de p(x) se, e somente se, existe r > 0 tal
que p(x) não muda de sinal para x pertencente ao conjunto ]x0 − r, x0 + r[ \{x0 } =
{x ∈ R : x0 − r < x < x0 + r, x 6= x0 }.[ 0,8 ponto ]
(b) Mostre que x0 é raiz de multiplicidade ímpar de p(x) se, e somente se, existe r > 0 tal
que o sinal de p(x) para x ∈ ]x0 −r, x0 [ é oposto ao sinal de p(x) para x ∈ ]x0 , x0 +r[ .
[ 0,8 ponto ]
(c) Interprete geometricamente os resultados dos dois ítens anteriores.[ 0,4 ponto ]
3. [ 2 pontos ] A massa de certas substâncias radioativas decresce a uma taxa proporcional
à própria massa. A meia-vida T de uma substância como essa é definida como o tempo
transcorrido para que sua massa se reduza à metade da inicial. Considere uma substância
radioativa S que cuja meia-vida é de 5.000 anos.

(a) Considere uma massa de m0 = 7 g da substância S. Qual é o tempo transcorrido


para a massa se reduza a 18 da inicial? Este tempo depende da massa inicial m0 ?
Justifique sua resposta. [ 0,2 ponto ]
(b) Determine a função m : [ 0, +∞[ → R que dá a massa da substância S, com massa
inicial m0 , em função do tempo medido em anos. [ 0,8 ponto ]
(c) Use as aproximações log10 (2) ∼
= 0, 3 e log10 (5) ∼
= 0, 7 para determinar uma aproxi-
mação para o tempo gasto para a massa da substância S se reduzir a 10% da inicial.
[ 1 ponto ]

4. [ 2 pontos ] Considere uma reta r, um ponto A 6∈ r e três pontos B, C, D ∈ r, tais que C


está entre B e D. Em cada um dos itens a seguir, decida se os dados são suficientes para
determinar com certeza as medidas de:
(i) cada um dos lados do triângulo ABC;
(ii) cada um dos ângulos do triângulo ABC.
Justifique rigorosamente as suas respostas.

[ = 60◦ ;[ 0,5 ponto ]


(a) BC = 1 e BAC
\ = 135◦ ;[ 0,5 ponto ]
(b) BC = 1 e ACD
[ = 60◦ e ACD
(c) BC = 1, BAC \ = 135◦ ;[ 0,5 ponto ]
[ = 60◦ e ACD
(d) BAC \ = 135◦ .[ 0,5 ponto ]

5. [ 2 pontos ] A figura a seguir representa um esboço do gráfico da função g : ]0, +∞[ → R


definida por g(x) = sen (log10 (x)), feito por um aplicativo computacional. Observe que o
aplicativo não conseguiu desenhar em detalhes o que ocorre perto da origem do sistema de
coordenadas.

2
(a) Determine todas as raízes da equação g(x) = 0. É possível determinar a menor raiz?
[ 0,6 ponto ]
(b) Faça um esboço do gráfico de g na janela gráfica 0 < x < 0, 1 e −2 < y < 2.
[ 0,7 ponto ]
(c) Considere um sistema de coordenadas x′ y em que o eixo horizontal x′ está em escala
logarítmica decimal (isto é, se x representa um eixo em coordenadas cartesianas
convencionais, então x′ = log10 x) e o eixo vertical y está em coordenadas cartesianas
convencionais. Faça um esboço do gráfico de g neste sistema, para 10−4 < x < 104
e −2 < y < 2.[ 0,7 ponto ]
Sociedade Brasileira de Matemática
Mestrado Profissional em Matemática em Rede Nacional

MA11 – Números e Funções Reais


Avaliação 2 GABARITO
22 de junho de 2013

1. Em cada um dos itens abaixo, dê, se possível, um exemplo de um polinômio p(x) satisfa-
zendo todas as condições dadas.
Caso o exemplo não seja possível, justifique a sua resposta.
Lembre-se que se p(x) = a0 + a1 x + a2 x2 + · · · + an−1 xn−1 + an xn então

p′ (x) = a1 + 2a2 x + · · · + (n − 1)an−1 xn−2 + nan xn−1

(a) p(1) = p(−1) = 0, p′ (0) = 1 e p(x) é de grau 2.


(b) p(1) = p(−1) = 0 e p′ (0) = 1.
(c) p(1) = p(−1) = 0, p′ (0) = 0 e p(x) é de grau 2.
(d) p(1) = p(−1) = 0, p(0) = p(2) = 1 e p(x) é de grau 2.
(e) p(1) = p(−1) = 0, p(0) = p(2) = 1 e p(x) é de grau 3.

Uma solução:

[pontuação total: 2,0]

(a) (0,4 ponto) Como x1 = 1 e x2 = −1 são raízes de p e p é um polinômio de grau 2,


então, pela simetria do gráfico das funções quadráticas, em seu ponto médio, x0 = 0,
deve ocorre um extremo de p. Logo, deve-se ter p′ (0) = 0.
Uma outra solução é a seguinte: como p(x) = a(x − 1)(x + 1) = ax2 − a, para algum
a 6= 0, já que x1 = 1 e x2 = −1 são raízes, então p′ (x) = 2ax e p′ (0) = 0.
Portanto, o exemplo não é possível.
(b) (0,4 ponto) Consideremos, por exemplo, o polinômio de grau 3, p(x) = a x3 + b x2 +
c x + d. Então, p′ (x) = 3 a x2 + 2 b x + c. Logo, para satisfazer as condições dadas,
deve-se ter:

 a+b+c+d=0

−a + b − c + d = 0

c=1

Isto é: a = −1, b = −d e c = 1. Portanto, pode-se tomar como exemplo: p(x) =
−x3 + x.
(c) (0,4 ponto) Pelo mesmo argumento do item (a), a condição p′ (0) = 0 é corolário da
condição p(1) = p(−1) = 0. Logo, basta garantir que p(1) = p(−1) = 0.
Portanto, pode-se tomar como exemplo: p(x) = x2 − 1.
(d) (0,4 ponto) Consideremos o polinômio de grau 2, p(x) = a x2 +b x+c. Para satisfazer
as condições dadas, deve-se ter:


 a+b+c=0

 a−b+c=0


 c=1
4a + 2b + c = 1

Entretanto, substituindo c = 1 nas duas primeiras equações conclui-se que a = −1 e


b = 0, o que contradiz a última equação. Logo, o sistema não tem soluções.
Portanto, o exemplo não é possível.
(e) (0,4 ponto) Consideremos, por exemplo, o polinômio de grau 3, p(x) = a x3 + b x2 +
c x + d. Para satisfazer as condições dadas, deve-se ter:


 a+b+c+d=0

 −a + b − c + d = 0


 d=1
8a + 4b + 2c + d = 1

Substituindo d = 1 nas duas primeiras equações conclui-se que b = −1 e a + c = 0.


Da última equação, segue que a = 32 e c = − 23 .
Portanto, o (único) exemplo possível é: p(x) = 23 x3 − x2 − 23 x + 1.

2. Um número real x0 é raiz de multiplicidade k do polinômio p(x) se p(x) = (x − x0 )k q(x),


para algum polinômio q(x), com q(x0 ) 6= 0.
Sugestão para resolver os ítens abaixo: Use o fato de que toda função polinomial é uma
função contínua e que "Se f é uma função real contínua e f (x0 ) 6= 0, então existe uma
vizinhança de x0 em que f não se anula".

(a) Mostre que x0 é raiz de multiplicidade par de p(x) se, e somente se, existe r > 0 tal
que p(x) não muda de sinal para x pertencente ao conjunto ]x0 − r, x0 + r[ \{x0 } =
{x ∈ R : x0 − r < x < x0 + r, x 6= x0 }.

2
(b) Mostre que x0 é raiz de multiplicidade ímpar de p(x) se, e somente se, existe r > 0 tal
que o sinal de p(x) para x ∈ ]x0 −r, x0 [ é oposto ao sinal de p(x) para x ∈ ]x0 , x0 +r[ .
(c) Interprete geometricamente os resultados dos dois ítens anteriores.

Uma solução:

[pontuação total: 2,0]


Nesta questão, embora não explicitamente afirmado, deve-se supor, como uma hipótese
global, que x0 é uma raiz de p(x), embora a parte b) possa ser provada sem que isto seja
assumido.
Tem-se que q(x0 ) 6= 0, digamos q(x0 ) > 0. Então, existe r > 0 tal que q(x) > 0 ∀ x ∈ I,
em que I é o intervalo ]x0 −r, x0 +r[ . Como p(x) = (x−x0 )k q(x), então p(x) e (x−x0 )k
possuem o mesmo sinal em I \ {x0 } (se q(x0 ) < 0, p(x) e (x − x0 )k terão sinais opostos
em I \ {x0 }, e as conclusões dos itens a seguir valerão com os sinais contrários).

(a) (0,8 ponto) Tem-se que k é par se, e somente se, (x − x0 )k > 0, ∀ x ∈ I \ {x0 }.
Logo, k é par se, e somente se, p(x) > 0, ∀ x ∈ I \ {x0 }.
(b) (0,8 ponto) Tem-se que k é ímpar se, e somente se, (x − x0 )k < 0, ∀ x ∈ ]x0 − r, x0 [
e (x − x0 )k > 0, ∀ x ∈ ]x0 , x0 + r[ . Logo, k é ímpar se, e somente se, p(x) < 0,
∀ x ∈ ]x0 − r, x0 [ e p(x) > 0, ∀ x ∈ ]x0 , x0 + r[ .
(c) (0,4 ponto) Geometricamente, tem-se que:
i. se a multiplicidade de x0 é par, então o gráfico de p “toca” o eixo em x0 , no sentido
em que, próximo ao ponto (x0 , 0), fica no mesmo semi-plano determinado pelo
eixo horizontal;
ii. se a multiplicidade de x0 é ímpar, então o gráfico de p “cruza” o eixo em x0 , no
sentido em que, próximo ao ponto (x0 , 0), fica em semi-planos opostos determi-
nados pelo eixo horizontal.

3. A massa de certas substâncias radioativas decresce a uma taxa proporcional à própria


massa. A meia-vida T de uma substância como essa é definida como o tempo transcorrido
para que sua massa se reduza à metade da inicial. Considere uma substância radioativa S
que cuja meia-vida é de 5.000 anos.

(a) Considere uma massa de m0 = 7 g da substância S. Qual é o tempo transcorrido


para a massa se reduza a 18 da inicial? Este tempo depende da massa inicial m0 ?
Justifique sua resposta.

3
(b) Determine a função m : [ 0, +∞[ → R que dá a massa da substância S, com massa
inicial m0 , em função do tempo medido em anos.
(c) Use as aproximações log (2) ∼ = 0, 3 e log (5) ∼
10 = 0, 7 para determinar uma apro-
10
ximação para o tempo gasto para a massa da substância S se reduzir a 10% da
inicial.

Uma solução:

[pontuação total: 2,0]

(a) (0,2 ponto) Após os primeiros 5.000 anos, a massa será igual m1 = 12 m0 . Após mais
5.000 a massa será m2 = 12 m1 = 14 m0 ; e após outros 5.000, esta será m3 = 12 m2 =
1
8
m0 . Portanto, são decorridos 15.000 para que a massa se reduza a 18 da inicial.
Independentemente de m0 = 7g ou outro valor, o tempo para que a massa seja
reduzida a 18 da inicial será sempre de 15.000 anos.
Como este argumento mostra, devido à definição de meia-vida, a resposta não depende
do valor da massa inicial.
(b) (0,8 ponto) Consideremos uma variável s representando o tempo medido em unidades
de 5.000 anos. Como a cada 5.000 anos, a massa cai a metade, então a massa em
função de s é dada por:
1
m(s) = s m0 .
2
Se t é uma variável representando o tempo medido em anos, então t = 5.000 s.
Portanto:
1
m(t) = t/5.000 m0 .
2
(c) (1,0 ponto) Devemos resolver a equação m(t) = 1
10
m0 , que corresponde a 2t/5.000 =
10. Portanto, temos:

t log10 5
= log2 10 = 1 + log2 5 = 1 +
5.000 log10 2
Logo:
   
log10 5 7 50.000
t = 5.000 1 + ≃ 5.000 1 + = ≃ 16.667 anos.
log10 2 3 3

4
4. Considere uma reta r, um ponto A 6∈ r e três pontos B, C, D ∈ r, tais que C está entre
B e D. Em cada um dos itens a seguir, decida se os dados são suficientes para determinar
com certeza as medidas de:
(i) cada um dos lados do triângulo ABC;
(ii) cada um dos ângulos do triângulo ABC.
Justifique rigorosamente as suas respostas.

[ = 60◦ ;
(a) BC = 1 e BAC
\ = 135◦ ;
(b) BC = 1 e ACD
[ = 60◦ e ACD
(c) BC = 1, BAC \ = 135◦ ;
[ = 60◦ e ACD
(d) BAC \ = 135◦ .

Uma solução:

[pontuação total: 2,0]


Sejam a = BC, b = AC e c = AB.

[ Isto é:
(a) (0,5 ponto) Pela Lei dos Cossenos, temos: a2 = b2 + c2 − 2 b c cos(BAC).

b2 + c 2 − b c = 1 .

Quaisquer soluções da equação acima satisfazem às condições dadas. Podemos tomar,


por exemplo, os triângulos T1 e T2 , como lados dados respectivamente por:
√ √
3 2 3
a1 = b 1 = c 1 = 1 a2 = 1, b2 = , c2 =
3 3
Além de terem lados diferentes, estes triângulos possuem, claramente, ângulos dife-
rentes (isto é, não são semelhantes), uma vez que o primeiro é equilátero e o segundo
não.
Portanto, não é possível determinar com certeza as medidas dos lados ou dos ângulos
do triângulo. De fato, o lugar geométrico dos pontos A que satisfazem as condições
dadas formam um arco de círculo (chamado arco capaz). Portanto, há infinitos
triângulos satisfazendo essas condições.
[ Isto é:
(b) (0,5 ponto) Pela Lei dos Cossenos, temos: c2 = a2 + b2 − 2 a b cos(BCA).

c 2 = 1 + b2 − 2b.

5
Como no item anterior, quaisquer soluções da equação acima satisfazem às condi-
ções dadas. Podemos tomar, por exemplo, os triângulos T1 e T2 , como lados dados
respectivamente por:

√ √ √
a1 = c1 = 1 , b1 = 2 a2 = 1, b2 = 2 2, c2 = 5

De forma análoga ao item anterior, estes triângulos possuem, claramente, ângulos


diferentes (isto é, não são semelhantes), uma vez que o primeiro é isósceles e o
segundo não.
Portanto, não é possível determinar com certeza as medidas dos lados ou dos ângulos
do triângulo. De fato, o lugar geométrico dos pontos A que satisfazem as condições
~ Portanto, há infinitos triângulos satisfazendo essas condições.
dadas é a semi-reta CA.
(c) (0,5 ponto) Neste caso, os ângulos do triângulo estão determinados, pois temos:

[ = 60◦
BAC [ = 45◦
BCA [ = 75◦
ABC
[
sen (BAC) [
sen (ABC) [
sen (BCA)
Pela Lei dos Senos, temos: = = .
a b c
Temos
√ √ que √ sen (75◦ )√= sen (45◦ + 30◦ ) = sen (45◦ ) cos(30◦ ) + cos(45◦ ) sen (30◦ ) =
2 3 21 2 √
+ = ( 3 + 1).
2 2 2 2 4
Isto permite determinar também as medidas dos lados dos triângulo:
√ √ √ √
3 2( 3 + 1) 2
= =
2 4b 2c
Portanto:
√ √ √ √
2( 3 + 1) 2( 3 + 3)
b= √ =
√ 2 √ 3 6
2 6
c= √ =
3 3
Como as condições dadas nesse item são a união das condições dos itens (a) e (b),
o ponto A que satisfaz essas condições está na interseção dos lugares geométricos
mencionados nos itens (a) e (b) (arco de círculo e semi-reta). Portanto, há um único
triângulo satisfazendo às condições dadas.
(d) (0,5 ponto) Neste caso, os ângulos do triângulo estão determinados, pois temos:

[ = 60◦
BAC [ = 45◦
BCA [ = 75◦
ABC

6
Porém como não há nenhuma informação sobre as medidas dos lados, não é possível
determiná-las. De fato, há uma família de triângulos semelhantes que satisfazem as
condições dadas.

5. A figura a seguir representa um esboço do gráfico da função g : ]0, +∞[ → R definida por
g(x) = sen (log10 (x)), feito por um aplicativo computacional. Observe que o aplicativo não
conseguiu desenhar em detalhes o que ocorre perto da origem do sistema de coordenadas.

(a) Determine todas as raízes da equação g(x) = 0. É possível determinar a menor raiz?
(b) Faça um esboço do gráfico de g na janela gráfica 0 < x < 0, 1 e −2 < y < 2.
(c) Considere um sistema de coordenadas x′ y em que o eixo horizontal x′ está em escala
logarítmica decimal (isto é, se x representa um eixo em coordenadas cartesianas
convencionais, então x′ = log10 x) e o eixo vertical y está em coordenadas cartesianas
convencionais. Faça um esboço do gráfico de g neste sistema, para 10−4 < x < 104
e −2 < y < 2.

Uma solução:

[pontuação total: 2,0]

(a) (0,6 ponto) Temos que:

sen (log10 x) = 0 ⇔ log10 x = k π , k ∈ Z ⇔ x = 10k π , k ∈ Z

7
Então, tomando valores negativos de k, obtemos raízes tão próximas de 0 quanto se
queira. Portanto, não existe uma raiz mínima para a equação.
(b) (0,7 ponto) Nesta janela, o gráfico terá o seguinte aspecto:

(c) (0,7 ponto) No sistema de variáveis x′ y, a equação de g adquire a forma g(x′ ) =


sen (x). Portanto, neste sistema de eixos o gráfico terá o aspecto de uma curva
senóide. Neste intervalo da variável x, encontram-se as raízes: x1 = 10−π ≃ 0, 001,
x2 = 1 e x3 = 10π ≃ 1000.
Sociedade Brasileira de Matemática
Mestrado Profissional em Matemática em Rede Nacional

MA11 – Números e Funções Reais


Avaliação 3
06 de julho de 2013

1. (1,5 pontos) Determine se as afirmações a seguir são verdadeiras ou falsas, justificando


adequadamente e em detalhes as suas respostas.

(a) (0,5 ponto) Se f : R → R e g : R → R são funções monótonas crescentes, então


a função soma f + g : R → R, definida por (f + g)(x) = f (x) + g(x) é monótona
crescente.
(b) (0,5 ponto) Se f : R → R é uma função limitada superiormente, então f admite um
ponto de máximo absoluto.
(c) (0,5 ponto) Se f : R → R admite um ponto de máximo local, então f admite um
ponto de máximo absoluto.

2. (2,0 pontos) Da mesma forma que se expressa um número real no sistema de numeração
decimal, é possível expressá-lo em um sistema de numeração posicional qualquer, de base
β ∈ N, β > 2. Dizemos que um número a ∈ R está expresso no sistema de base β se ele
é escrito na forma:
X
+∞
a = a0 + ak β −k
k=1

em que a0 ∈ Z e os ak são dígitos entre 0 e β − 1 (incluindo-os).

(a) (1,0 ponto) Mostre que, se um número x ∈ R é irracional, então x possui represen-
tação infinita em toda base β.
(b) (1,0 ponto) Reciprocamente, mostre que, se um número x ∈ R possui representação
infinita em toda base β, então x é irracional.

3. (2,0 pontos) Considere a função p1 : R → R, p1 (x) = (x2 − 1)2 . A figura abaixo mostra
o gráfico de uma função p2 : R → R na forma p2 (x) = c p1 (a x − b) + d, sendo a, b, c e d
constantes reais. Determine a, b, c e d. Justifique sua resposta.
4. (2,0 pontos) Considere as funções u, v : R → R, definidas por u(x) = 2 sen (x) e v(x) =
sen (2x ).

(a) (1,0 ponto) Determine o maior e menor valores atingidos por u e v.


(b) (1,0 ponto) Esboce os gráficos de u e de v.

1
5. (2,5 pontos) Considere a função g : R∗ → R, g(x) = 21− x .

(a) (1,0 ponto) Faça um esboço o gráfico de g.


(b) (0,75 ponto) Determine todas as soluções reais das equações g(x) = 2 e g(x) = 4.
(c) (0,75 ponto) Resolva a inequação g(x) < 4, para x ∈ R.

2
Sociedade Brasileira de Matemática
Mestrado Profissional em Matemática em Rede Nacional

MA11 – Números e Funções Reais


Avaliação 3 - GABARITO
06 de julho de 2013

1. (1,5 pontos) Determine se as afirmações a seguir são verdadeiras ou falsas, justificando


adequadamente e em detalhes as suas respostas.

(a) (0,5 ponto) Se f : R → R e g : R → R são funções monótonas crescentes, então


a função soma f + g : R → R, definida por (f + g)(x) = f (x) + g(x) é monótona
crescente.
(b) (0,5 ponto) Se f : R → R é uma função limitada superiormente, então f admite um
ponto de máximo absoluto.
(c) (0,5 ponto) Se f : R → R admite um ponto de máximo local, então f admite um
ponto de máximo absoluto.

2. (2,0 pontos) Da mesma forma que se expressa um número real no sistema de numeração
decimal, é possível expressá-lo em um sistema de numeração posicional qualquer, de base
β ∈ N, β > 2. Dizemos que um número a ∈ R está expresso no sistema de base β se ele
é escrito na forma:
+∞
X
a = a0 + ak β −k
k=1

em que a0 ∈ Z e os ak são dígitos entre 0 e β − 1 (incluindo-os).

(a) (1,0 ponto) Mostre que, se um número x ∈ R é irracional, então x possui represen-
tação infinita em toda base β.
(b) (1,0 ponto) Reciprocamente, mostre que, se um número x ∈ R possui representação
infinita em toda base β, então x é irracional.

3. (2,0 pontos) Considere a função p1 : R → R, p1 (x) = (x2 − 1)2 . A figura abaixo mostra
o gráfico de uma função p2 : R → R na forma p2 (x) = c p1 (a x − b) + d, sendo a, b, c e d
constantes reais. Determine a, b, c e d. Justifique sua resposta.
4. (2,0 pontos) Considere as funções u, v : R → R, definidas por u(x) = 2 sen (x) e v(x) =
sen (2x ).

(a) (1,0 ponto) Determine o maior e menor valores atingidos por u e v.


(b) (1,0 ponto) Esboce os gráficos de u e de v.

1
5. (2,5 pontos) Considere a função g : R∗ → R, g(x) = 21− x .

(a) (1,0 ponto) Faça um esboço o gráfico de g.


(b) (0,75 ponto) Determine todas as soluções reais das equações g(x) = 2 e g(x) = 4.
(c) (0,75 ponto) Resolva a inequação g(x) < 4, para x ∈ R.

2
Sociedade Brasileira de Matemática
Mestrado Profissional em Matemática em Rede Nacional

MA11 – Números e Funções Reais


Avaliação 1 – Gabarito

1. (a) Verdadeiro.
Sejam x1 , x2 ∈ R, com x1 < x2 . Como f e g são monótonas crescentes, então
f (x1 ) < f (x2 ) e g(x1 ) < g(x2 ). Logo, f (x1 ) + g(x1 ) < f (x2 ) + g(x2 ). Portanto,
f + g é monótona crescente.
(b) Falso.
Contra-exemplo: A função f : R → R, f (x) = −2x é limitada superiormente, mas
não admite um ponto de máximo absoluto.
(c) Falso.
Contra-exemplo: A função f : R → R, f (x) = (x2 −1)2 admite um ponto de máximo
local em (0, 1), mas não um ponto de máximo absoluto.

2. (a) Suponhamos que x tenha representação finita em alguma base β. Então, pela defini-
ção (dada no enunciado da questão), x é soma finita de números racionais, portanto
é racional.
(b) Suponhamos que x seja racional; pelo algoritmo da divisão, podemos supor, sem
a
perda de generalidade, que 0 ≤ x < 1. Então x se escreve na forma x = , com
b
a, b ∈ N, 0 ≤ a < b. Consideremos o sistema de numeração posicional β = b.
Como x = a b−1 , então, por definição, esta é a representação de x na base b (isto é,
xb = 0, a). Assim, existe uma base em que x possui representação finita.

3. O gráfico da função p1 tem o aspecto mostrado na figura abaixo. O gráfico de p2 é dado


pela aplicação de translações e de dilatações no gráfico de p1 .

1
Observemos primeiro os efeitos da translação e da dilatação horizontais, determinadas pelas
constantes a e b. Os pontos (−1, 0) e (1, 0) são transformados nos pontos − 12 , 0 e 12 , 0 ,
 

respectivamente. Assim, a distância entre as abscissas desses pontos é multiplicada pelo


fator 12 . Podemos concluir que a = 2. Como o eixo de simetria vertical não se altera, não
há deslocamento horizontal, isto é, b = 0.
Passemos agora a analisar a translação e a dilatação verticais, determinadas pelas constan-
tes c e d. Observamos que não há dilatação vertical do gráfico, pois as distâncias entre as
ordenadas de pontos do gráfico de p1 e as distâncias entre as ordenadas dos correspondentes
pontos do gráfico de p2 permanecem as mesmas. Isto pode ser facilmente visto olhando-se
os pontos de máximo e de mínimo locais das funções. Segue que c = 1. Finalmente, como
a ordenada de (0, 1) é subtraída de duas unidades, concluímos que d = −2 (translação
vertical).
Desta forma, temos que:

a=2 b=0 c=1 d = −2

Portanto: p2 (x) = p1 (2 x) − 2 = ((2 x)2 − 1)2 − 2 = (4 x2 − 1)2 − 2.

4. (a) Como u é dada por uma função exponencial x 7→ 2x aplicada sobre a função seno e
esta função exponencial é estritamente crescente, segue que o valor de u será máximo
quando o valor de sen x for máximo e será mínimo quando o valor de sen x for
mínimo.

π
x= + 2 k π, k ∈ Z ⇔ sen (x) = 1 ⇔ u(x) = 2
2
3π 1
x= + 2 k π, k ∈ Z ⇔ sen (x) = −1 ⇔ u(x) =
2 2
1
Portanto, o maior e o menor valores atingidos por u são 2 e 2
.

2
Como v é dada pela função seno aplicada sobre outra função real, temos necessaria-
mente que −1 6 v(x) 6 1 ∀ x ∈ R. Mais precisamente, temos que:

xπ π 
v(x) = 1 ⇔ 2 = + 2 k π, k ∈ N ⇔ x = log2 + 2kπ , k ∈ N
2 2 
x 3π 3π
v(x) = −1 ⇔ 2 = + 2 k π, k ∈ N ⇔ x = log2 + 2kπ , k ∈ N
2 2

Portanto, o maior e o menor valores atingidos por v são 1 e −1.


(b) Com base no item anterior, concluímos que os gráficos de u e de v têm os seguintes
aspectos:

3
5. (a) Quando x cresce muito em valor absoluto (isto é, quando x tende a ±∞), o expoente
1 − x1 se aproxima de 1, portanto g(x) se aproxima de 2. Quando x se aproxima de 0
com valores positivos, o expoente 1 − x1 tende de −∞, logo g(x) se aproxima de 0.
Quando x se aproxima de 0 com valores negativos, o expoente 1 − x1 tende de +∞,
logo g(x) também tende de +∞. Logo, o gráfico de g tem o seguinte aspecto:

1
(b) Para que tivéssemos g(x) = 2, deveríamos ter 1 − x
= 1. Portanto, a equação
g(x) = 2 não possui soluções reais.
Resolvendo a equação g(x) = 4, temos

1 1
g(x) = 4 ⇔ 1 − = 2 ⇔ = −1 ⇔ x = −1
x x
Portanto, a única solução real da equação g(x) = 2 é x = −1.
(c) Em primeiro lugar, observamos que, como a função exponencial é estritamente cres-
cente, então:

1
g(x) < 4 ⇔ 1 − <2
x
Então:

1
g(x) < 4 ⇔ > −1
x
Para continuar a resolução da inequação, devemos considerar separadamente os casos
em x > 0 e x < 0:
1
• Se x > 0, então x
> −1 ⇔ x > −1.

4
1
• Se x < 0, então x
> −1 ⇔ x < −1.
Portanto a solução da inequação é dada pelo conjunto ] − ∞, −1[ ∪ ] 0, +∞[ .
Observe que esta solução pode ser visualizada no gráfico de g, pelos pontos do domínio
cujas imagens ficam abaixo da reta y = 4.

5
MA12
AVALIAÇÕES

MA12

2011-2013
MA12
-
2011
MA12 – Matemática Discreta – Prova 1 – 2011

Questão 1.
Considere a sequência (an )n≥1 definida como indicado abaixo:

a1 = 1
a2 = 2+3
a3 = 4+5+6
a4 = 7 + 8 + 9 + 10
...

(0.5) (a) O termo a10 é a soma de 10 inteiros consecutivos. Qual é o menor e o qual é o maior desses inteiros?

(0.5) (b) Calcule a10 .

(1.0) (c) Forneça uma expressão geral para o termo an .

Questão 2.
Um comerciante, para quem o dinheiro vale 5% ao mês, oferece determinado produto por 3 prestações mensais
iguais a R$ 100,00, a primeira paga no ato da compra.

(1.0) (a) Que valor o comerciante deve cobrar por esse produto, no caso de pagamento à vista?

(1.0) (b) Se um consumidor desejar pagar o produto em três prestações mensais iguais, mas sendo a primeira paga um
mês após a compra, qual deve ser o valor das parcelas?

Utilize, se desejar, os seguintes valores para as potências de 1, 05: 1, 052 = 1, 1025; 1, 05−1 = 0, 9524; 1, 05−2 = 0, 9070.

Questão 3.
Considere o conjunto dos números escritos apenas com os algarismos 1, 2 e 3, em que o algarismo 1 aparece uma
quantidade par de vezes (por exemplo, 2322 e 12123). Seja an a quantidade desses números contendo exatamente n
algarismos.

(0.4) (a) Liste todos esses números para n = 1 e n = 2, indicando os valores de a1 e a2 .

(0.8) (b) Explique por que an satisfaz a equação de recorrência an+1 = (3n − an ) + 2an , para n ≥ 1 (note que 3n é o
número total de números com n algarismos iguais a 1, 2 ou 3).

(0.8) (c) Resolva a equação de recorrência em (b).


MA12 – Matemática Discreta – Prova 1 – 2011

Questão 4.

(1.0) (a) Mostre, por indução finita, que

(2n − 1)3n + 1
1 · 30 + 2 · 31 + 3 · 32 + . . . + n · 3n−1 = .
4

(1.0) (b) Seja (an )n≥1 progressão geométrica com termo inicial a1 positivo e razão r > 1, e Sn a soma dos n primeiros
termos da progressão. Prove, por indução finita, que Sn ≤ r−1 an ,
r
para qualquer n ≥ 1.

Questão 5.
Seja (xn )n≥0 sequência definida pela relação de recorrência xn+1 = 2xn + 1, com termo inicial x0 ∈ R.

(0.5) (a) Encontre x0 tal que a sequência seja constante e igual a um número real a.

(1.0) (b) Resolva a recorrência com a substituição xn = yn + a, em que a é valor encontrado em (a).

(0.5) (c) Para que valores de x0 a sequência é crescente? Justifique.


P1 - MA 12 - 2011

Questão 1.
Considere a sequência ( an )n≥1 definida como indicado abaixo:

a1 = 1
a2 = 2+3
a3 = 4+5+6
a4 = 7 + 8 + 9 + 10
...

(0.5) (a) O termo a10 é a soma de 10 inteiros consecutivos. Qual é o menor e o qual é o maior desses inteiros?

(0.5) (b) Calcule a10 .

(1.0) (c) Forneça uma expressão geral para o termo an .

UMA RESPOSTA

(a) O primeiro inteiro da soma que define an é igual ao número de inteiros utilizados nos termos a1 , . . . , an−1 , isto
é, 1 + 2 + . . . + n − 1 mais um, isto é, é igual a 21 (n − 1)n + 1. O último inteiro é esse número mais n − 1. Portanto,
para n = 10, o primeiro inteiro é 46 e o último é 55.

(b) a10 é a soma de uma progressão aritmética de 10 termos, sendo o primeiro igual a 46 e o último igual a 55. Então

(46 + 55) · 10
a10 = = 101 · 5 = 505 .
2

(c) No caso de an , trata-se da soma de uma progressão aritmética de n termos, sendo o primeiro igual a 21 n(n − 1) + 1
e o último igual a 21 n(n − 1) + 1 + (n − 1), ou seja, 21 n(n − 1) + n, como visto em (a). Então
h i h i
1 1
2 n ( n − 1 ) + 1 + 2 n ( n − 1 ) + n ( n − 1) n2 + ( n + 1) n n3 + n
an = ·n = = .
2 2 2

1
P1 - MA 12 - 2011

Questão 2.
Um comerciante, para quem o dinheiro vale 5% ao mês, oferece determinado produto por 3 prestações mensais
iguais a R$ 100,00, a primeira paga no ato da compra.

(1.0) (a) Que valor o comerciante deve cobrar por esse produto, no caso de pagamento à vista?

(1.0) (b) Se um consumidor desejar pagar o produto em três prestações mensais iguais, mas sendo a primeira paga um
mês após a compra, qual deve ser o valor das parcelas?

Utilize, se desejar, os seguintes valores para as potências de 1, 05: 1, 052 = 1, 1025; 1, 05−1 = 0, 9524; 1, 05−2 =
0, 9070.

UMA RESPOSTA

(a) Trazendo os valores da segunda e da terceira prestações para o ato da compra, e somando, obtém-se

100 100
100 + + = 100 + 95, 24 + 90, 70 = 285, 94 .
1, 05 1, 052
Então o comerciante poderá cobrar 285,94 reais, de forma que, se deixar seu dinheiro valorizar 5% ao mês, poderá
dispor de 100 reais no ato da compra (tirando 100 reais dos 285,94), 100 reais ao final do primeiro mês (deixando
95,24 reais valorizarem 5% durante um mês) e 100 reais ao final do segundo mês (deixando 90,70 reais valorizarem
5% ao mês durante dois meses).

(b) Para o parcelamento desejado pelo consumidor, as parcelas se deslocam um mês adiante. Então em cada uma
das três parcelas de 100 reais devem incidir juros de 5%. Portanto, são 3 parcelas de 105 reais.

2
P1 - MA 12 - 2011

Questão 3.
Considere o conjunto dos números escritos apenas com os algarismos 1, 2 e 3, em que o algarismo 1 aparece uma
quantidade par de vezes (por exemplo, 2322 e 12123). Seja an a quantidade desses números contendo exatamente n
algarismos.

(0.4) (a) Liste todos esses números para n = 1 e n = 2, indicando os valores de a1 e a2 .

(0.8) (b) Explique por que an satisfaz a equação de recorrência an+1 = (3n − an ) + 2an , para n ≥ 1 (note que 3n é o
número total de números com n algarismos iguais a 1, 2 ou 3).

(0.8) (c) Resolva a equação de recorrência em (b).

UMA RESPOSTA

(a) Para n = 1 só há três números possíveis: 1, 2 e 3. Somente os dois últimos têm um número par de algarismos
iguais a 1 (neste caso, nenhum algarismo igual a 1). Então a1 = 2. Os números de 2 algarismos são: 11, 12, 13, 21,
22, 23, 31, 32, 33, num total de 9 = 32 . Cinco deles têm uma quantidade par de algarismos iguais a 1, então a2 = 5.

(b) (Antes de fazer o exercício, pode-se verificar se a fórmula está correta para n = 1: 5 = a2 = (31 − a1 ) + 2a1 =
3 + a1 = 3 + 2 = 5.) Observa-se primeiro que a quantidade de números com n algarismos tendo uma quantidade
ímpar de algarismos iguais a 1 é 3n − an , pois o número total de sequências é 3n .
Para obter a relação de recorrência, observe que todo número de n + 1 algarismos é uma concatenação de um
número de n algarismos com um número de 1 algarismo. Para que a quantidade de algarismos iguais a 1 do
número de n + 1 algarismos seja par é preciso que: ou o número de algarismos iguais a 1 de cada um dos números
concatenados seja ímpar ou o número de algarismos iguais a 1 de cada um dos números concatenados seja par.
Então, para calcular an+1 , soma-se o número de concatenações do primeiro caso (ímpar-ímpar) com o número de
concatenações do segundo caso (par-par). Isto dá

an+1 = (3n − an ) · (31 − a1 ) + an · a1 ,

isto é, a fórmula do enunciado, já que a1 = 2.

(c) Observa-se que an+1 = an + 3n , apenas simplificando-se a expressão. Isto implica

an = a1 + 31 + 32 + . . . + 3n−1 = 1 + (1 + 3 + 32 + . . . + 3n−1 ) ,

em que a expressão entre parênteses é a soma dos n primeiros termos da progressão geométrica de termo inicial 1 e
razão 3, que vale
3n − 1
.
3−1
Portanto
3n + 1
an = .
2

3
P1 - MA 12 - 2011

Questão 4.

(1.0) (a) Mostre, por indução finita, que

(2n − 1)3n + 1
1 · 30 + 2 · 31 + 3 · 32 + . . . + n · 3n−1 = .
4

(1.0) (b) Seja ( an )n≥1 progressão geométrica com termo inicial a1 positivo e razão r > 1, e Sn a soma dos n primeiros
r
termos da progressão. Prove, por indução finita, que Sn ≤ r −1 a n , para qualquer n ≥ 1.

UMA RESPOSTA

(a) A equação é verdadeira para n = 1, pois 1 · 30 = 1 e

(2 · 1 − 1)31 + 1
= 1.
4
Supondo válida para n, vamos mostrar que vale para n + 1, isto é, vamos mostrar que, acrescentando o termo
(n + 1) · 3n , a soma resultará em
(2 ( n + 1 ) − 1 )3n +1 + 1
.
4
Usando a hipótese de indução,

(2n − 1)3n + 1
1 · 30 + 2 · 31 + 3 · 32 + . . . + n · 3n−1 + (n + 1)3n = + ( n + 1 )3n .
4
Manipulando a expressão à direita,

(2n − 1)3n + 1 [2n − 1 + 4(n + 1)]3n + 1 (2n + 1)3n+1 + 1 (2 ( n + 1 ) − 1 )3n +1 + 1


+ ( n + 1 )3n = = = ,
4 4 4 4
como queríamos demonstrar.

r
(b) Para n = 1 a desigualdade é verdadeira: como r > 1, então > 1; e como S1 = a1 > 0, então S1 = a1 < r−r 1 a1 .
r −1
Suponha agora que a desigualdade vale para n, isto é, suponha que Sn ≤ r−r 1 an é verdadeira. Vamos provar que
ela vale para n + 1, isto é, vamos provar que Sn+1 ≤ r−r 1 an+1 . Primeiro, escrevemos Sn+1 = Sn + an+1 , pois Sn+1
é a soma dos primeiros n termos adicionada do termo n + 1. Usando a hipótese de indução, Sn+1 ≤ r−r 1 an + an+1 .
a
Como se trata de uma progressão geométrica an+1 = ran , ou seja, podemos trocar an por nr+1 . Então Sn+1 ≤
r a n +1 1 r
r −1 · r + an+1 , isto é, Sn+1 ≤ ( r −1 + 1) an+1 = r −1 an+1 , que é o que queríamos demonstrar.

4
P1 - MA 12 - 2011

Questão 5.
Seja ( xn )n≥0 sequência definida pela relação de recorrência xn+1 = 2xn + 1, com termo inicial x0 ∈ R.

(0.5) (a) Encontre x0 tal que a sequência seja constante e igual a um número real a.

(1.0) (b) Resolva a recorrência com a substituição xn = yn + a, em que a é valor encontrado em (a).

(0.5) (c) Para que valores de x0 a sequência é crescente? Justifique.

UMA RESPOSTA

(a) Basta achar a tal que 2a + 1 = a. Isto dá a = −1. Se x0 = a então x1 = 2x0 + 1 = 2a + 1 = a = x0 , e, da mesma
forma, x2 = x1 , x3 = x2 , . . ., xn+1 = xn para qualquer n ≥ 0, ou seja, a sequência é constante.

(b) Com a substituição sugerida, xn = yn − 1. Então yn+1 − 1 = 2(yn − 1) + 1, isto é, yn+1 = 2yn , com y0 = x0 + 1.
Então yn = 2n y0 = 2n ( x0 + 1) e xn = yn − 1 = −1 + 2n ( x0 + 1).

(c) Se x0 + 1 > 0, isto é, x0 > −1, então 2n ( x0 + 1) é crescente e xn = −1 + 2n ( x0 + 1) é crescente. Se x0 + 1 < 0, isto
é x0 < −1, então xn = −1 + 2n ( x0 + 1) = −1 − 2n | x0 + 1| é descrescente. E se x0 = −1 então xn é constante. De
onde se conclui que xn é crescente se, e somente se, x0 ∈ (−1, +∞).

5
MA12 – Matemática Discreta – Prova 2 – 2011

Questão 1.
Considere os caminhos no plano iniciados no ponto (0, 0) com deslocamentos paralelos aos eixos coordenados,
sempre de uma unidade e no sentido positivo dos eixos x e y (não se descarta a possibilidade de dois movimentos
unitários seguidos na mesma direção, ver ilustração mostrando um caminho que termina em (5, 4)).
y
4

x
5

(1,0) (a) Explique por que o número de caminhos que terminam no ponto (m, n) é Cm+n
m
.

(1,0) (b) Quantos são os caminhos que terminam no ponto (8, 7), passam por (2, 3) mas não passam por (5, 4)?

Questão 2.
Os professores de seis disciplinas (entre as quais Português e Matemática) devem escolher um dia, de segunda a
sexta, de uma única semana para a realização da prova de sua disciplina. Suponha que cada professor escolha o seu
dia de prova ao acaso, sem combinar com os demais professores.

(1,0) (a) Qual é a probabilidade de que as provas de Português e Matemática sejam realizada no mesmo dia?

(1,0) (b) Qual é a probabilidade de que os alunos façam provas em todos os dias da semana?

Questão 3.
Em um jogo, uma moeda honesta é jogada seguidamente. Cada vez que sai cara, o jogador ganha 1 real; cada vez
que sai coroa, o jogador ganha 2 reais. O jogo termina quando o jogador tiver acumulado 4 ou mais reais.

(0,5) (a) Qual é a probabilidade de que o jogador ganhe exatamente 4 reais?

(0,5) (b) Qual é a probabilidade de que no último lançamento saia cara?

(1,0) (c) Dado que o jogador ganhou exatamente 4 reais, qual é a probabilidade de que tenha saı́do cara no último
lançamento?
MA12 – Matemática Discreta – Prova 2 – 2011

Questão 4.
Uma prova de concurso é formada por questões de múltipla escolha, com 4 alternativas por questão. Admita que
nenhum candidato deixe questões sem responder.

(1,0) (a) Qual é o número mı́nimo de candidatos para que seja possı́vel garantir que pelo menos 3 deles darão exatamente
as mesmas respostas nas 5 primeiras questões?

(1,0) (b) Qual é o valor máximo de n para o qual é possı́vel garantir que, em um concurso com 1000 candidatos, pelo
menos 2 darão as mesmas respostas nas primeiras n questões?

Questão 5.
Uma caixa retangular sem tampa tem arestas medindo x, y e z (veja figura, onde as linhas tracejadas indicam
segmentos de arestas obstruı́dos por alguma face).

(0,5) (a) Exprima a área e o volume da caixa em função de x, y e z.

(1,0) (b) Use a desigualdade das médias para mostrar que, se o volume da caixa é igual a 32, então sua área é maior ou
igual a 48.

(0,5) (c) Determine as medidas das arestas da caixa de área mı́nima com volume igual a 32.

z
y
x
AV2 - MA 12 - 2011

Questão 1.
Considere os caminhos no plano iniciados no ponto (0, 0) com deslocamentos paralelos aos eixos coordenados,
sempre de uma unidade e no sentido positivo dos eixos x e y (não se descarta a possibilidade de dois movimentos
unitários seguidos na mesma direção, ver ilustração mostrando um caminho que termina em (5, 4)).
y
4

x
5

(1,0) (a) Explique por que o número de caminhos que terminam no ponto (m, n) é Cm
m .
+n

(1,0) (b) Quantos são os caminhos que terminam no ponto (8, 7), passam por (2, 3) mas não passam por (5, 4)?

UMA SOLUÇÃO

(a) Chamaremos de horizontais os movimentos paralelos ao eixo x e de verticais os paralelos ao eixo y. Como todos
os movimentos são positivos e unitários, são necessários m movimentos horizontais e n movimentos verticais para
se chegar em (m, n), totalizando m + n movimentos. Um caminho fica totalmente determinado se dissermos quais
desses m + n movimentos são, digamos, movimentos horizontais. Portanto, precisamos saber de quantas maneiras
podemos escolher m movimentos horizontais entre os m + n movimentos do caminho. Isso dá Cm
m .
+n
Evidentemente poderíamos ter determinado os caminhos dizendo quais são os n movimentos horizontais dentre
os m + n movimentos. Esse raciocínio nos levaria a Cm
n
+n . Mas Cm+n = Cm+n .
n m

(b) Se um caminho até (8, 7) é obrigado a passar por (2, 3) então ele é a junção de um caminho que vai de (0, 0) a
(2, 3) com um caminho que vai de (2, 3) a (8, 7). No entanto, queremos que o caminho que vai de (2, 3) a (8, 7) não
passe por (5, 4), ou seja, queremos que ele vá de (2, 3) a (8, 7) sem ser a junção de um caminho de (2, 3) a (5, 4) com
um caminho de (5, 4) a (8, 7). Isso nos indica que precisamos calcular quantos caminhos temos de (0, 0) a (2, 3),
quantos de (2, 3) a (5, 4) e quantos de (5, 4) a (8, 7).
Segundo o item anterior, há C22+3 = C52 maneiras de ir de (0, 0) a (2, 3). Há C33+1 = C43 maneiras de se ir de (2, 3) a
(5, 4), pois são necessários 3 movimentos horizontais e 1 vertical. Há C33+3 = C63 maneiras de se ir de (5, 4) a (8, 7),
pois são necessários 3 movimentos horizontais e 3 verticais. E há C10 6 maneiras de se ir de (2, 3) a (8, 7), pois são

necessários 6 movimentos horizontais e 4 verticais.


Há, portanto, C43 · C63 maneiras de se ir de (2, 3) a (8, 7) passando por (5, 4). Então há C10
6 − C3 · C3 maneiras de se
4 6
ir de (2, 3) a (8, 7) sem passar por (5, 4). E, por conseguinte, há

N = C52 · (C10
6
− C43 · C63 )

1
maneiras de se ir de (0, 0) a (8, 7) passando por (2, 3) mas não passando por (5, 4).
5! 10! 10·9·8·7
Para termos um número, calculamos essas combinações: C52 = 3! 2!
6 =
= 10, C10 4!6! = 4·3·2 = 210, C43 = 4 e
6! 6·5·4
C63 = 3!3! = 3·2 = 20. Então
N = 10 · (210 − 4 · 20) = 1300 .

2
AV2 - MA 12 - 2011

Questão 2.
Os professores de seis disciplinas (entre as quais Português e Matemática) devem escolher um dia, de segunda
a sexta, de uma única semana para a realização da prova de sua disciplina. Suponha que cada professor escolha o
seu dia de prova ao acaso, sem combinar com os demais professores.

(1,0) (a) Qual é a probabilidade de que as provas de Português e Matemática sejam realizadas no mesmo dia?

(1,0) (b) Qual é a probabilidade de que os alunos façam provas em todos os dias da semana?

UMA SOLUÇÃO

(a) Nesta questão, não é preciso olhar para as outras disciplinas. Há 5 possibilidades para o dia de prova de Por-
tuguês e 5 possibilidades para o dia de prova de Matemática. Portanto, há 25 possibilidades para o par de provas
Português e Matemática. Dessas 25, apenas 5 são ocorrências de Português e Matemática no mesmo dia (uma
ocorrência para cada dia da semana). Então a probabilidade de que essas duas provas ocorram no mesmo dia é
5/25 = 0, 2 (ou 20%).
Outra maneira de pensar: fixado o dia da prova de Matemática, há 5 possibilidades para o dia de Português, e
apenas uma delas é no mesmo dia que Matemática. Isso dá os mesmos 20% de chances.

(b) Vamos contar de quantas maneiras se distribuem 6 provas nos 5 dias da semana sem deixar um dia livre. Com
essa imposição, certamente um dia terá duas provas e os demais dias terão apenas uma. Então começamos esco-
lhendo entre as 5 possibilidades para o dia da semana que terá duas provas. Escolhido esse dia, temos que escolher
duas das seis disciplinas para preenchê-lo. Temos C62 escolhas. Escolhidas essas duas disciplinas, ainda restam 4
para distribuir nos 4 dias: são 4! escolhas. Portanto há 5 · C62 · 4! maneiras de se distribuir 6 provas em 5 dias sem
deixar um dia livre.
Agora precisamos do total de maneiras de se distribuir as 6 provas durante a semana. Cada disciplina tem 5
escolhas, então são 56 possibilidades.
Então a probabilidade de não ficar um dia livre é o quociente

5 · C62 · 4! 5! · 15 72 · 16 1152
6
= 6
= 4! · 354 = = = 0, 1152 ,
5 5 10000 10000
ou 11,52%.

3
AV2 - MA 12 - 2011

Questão 3.
Em um jogo, uma moeda honesta é jogada seguidamente. Cada vez que sai cara, o jogador ganha 1 real; cada vez
que sai coroa, o jogador ganha 2 reais. O jogo termina quando o jogador tiver acumulado 4 ou mais reais.

(0,5) (a) Qual é a probabilidade de que o jogador ganhe exatamente 4 reais?

(0,5) (b) Qual é a probabilidade de que no último lançamento saia cara?

(1,0) (c) Dado que o jogador ganhou exatamente 4 reais, qual é a probabilidade de que tenha saído cara no último
lançamento?

UMA SOLUÇÃO

Nesta questão, convém fazer primeiro a árvore das possibilidades. Indicaremos “cara” por A e “coroa” por B. Em
cada nó da árvore, indicamos a sequência obtida (linha superior), o valor acumulado pelo jogador (linha do meio)
e a probabilidade daquela sequência (linha inferior). Os nós em cinza são aqueles em que o jogo termina. Veja que
a soma das probabilidades em cada nó pintado em cinza é igual a 1, e que a probabilidade indicada em cada um é
a probabilidade de o jogo terminar com aquela sequência.
A B
1 2
1/2 1/2
AA AB BA BB
2 3 3 4
1/4 1/4 1/4 1/4
AAA AAB ABA ABB BAA BAB
3 4 4 5 4 5
1/8 1/8 1/8 1/8 1/8 1/8
AAAA AAAB
4 5
1/16 1/16
(a) Para ver a probabilidade de que o jogador termine com exatamente 4 reais, basta somar as probabilidades dos
nós em cinza que têm ganho de 4 reais. São eles: AAAA (1/16), AAB (1/8), ABA (1/8), BAA (1/8) e BB (1/4). A
11
soma é 16 .

(b) O jogo termina com cara em todos os nós em cinza que terminam com a letra A. Então basta somar as probabili-
5
dades de cada caso. São eles AAAA (1/16), ABA (1/8) e BAA (1/8), o que dá 16 .

(c) Das situações em que o jogador terminou com 4 reais, listadas em (a), que têm probabilidade de 11/16 de
ocorrer, apenas AAAA, ABA e BAA terminam com A (cara), com probabilidade de 5/16. Então a probabilidade de
5/16 5
se terminar com cara dado que o jogador terminou com 4 reais é 11/16 = 11 .

4
AV2 - MA 12 - 2011

Questão 4.
Uma prova de concurso é formada por questões de múltipla escolha, com 4 alternativas por questão. Admita que
nenhum candidato deixe questões sem responder.

(1,0) (a) Qual é o número mínimo de candidatos para que seja possível garantir que pelo menos 3 deles darão exata-
mente as mesmas respostas nas 5 primeiras questões?

(1,0) (b) Qual é o valor máximo de n para o qual é possível garantir que, em um concurso com 1000 candidatos, pelo
menos 2 darão as mesmas respostas nas primeiras n questões?

UMA SOLUÇÃO

(a) O conjunto de possibilidades de respostas para as 5 primeiras questões, cada uma com 4 alternativas, é 45 .
É possível distribuir as respostas de 2 · 45 = 2048 candidatos de forma que cada conjunto de respostas se repita
exatamente duas vezes, mas se houver 2 · 45 + 1 = 2049 candidatos isso não é mais possível, sempre haverá ao
menos 3 provas iguais nas cinco primeiras questões.

(b) Considerando agora as n primeiras questões, há 4n possibilidades de resposta. Para garantir que em 1000 can-
didatos pelo menos 2 respondam de forma igual a essas primeiras n questões, é necessário que 1000 ≥ 4n + 1, isto
é, 4n ≤ 999. O valor máximo de n tal que 4n ≤ 999 é 4 (pois 44 = 28 = 256 e 45 = 210 = 1024). Resposta: n = 4.

5
AV2 - MA 12 - 2011

Questão 5.
Uma caixa retangular sem tampa tem arestas medindo x, y e z (veja figura, onde as linhas tracejadas indicam
segmentos de arestas obstruídos por alguma face).

(0,5) (a) Exprima a área e o volume da caixa em função de x, y e z.

(1,0) (b) Use a desigualdade das médias para mostrar que, se o volume da caixa é igual a 32, então sua área é maior ou
igual a 48.

(0,5) (c) Determine as medidas das arestas da caixa de área mínima com volume igual a 32.

z
y
x

UMA SOLUÇÃO

(a) A área da caixa é igual a xy + 2xz + 2yz e seu volume é igual a xyz.
(b) A soma xy + 2xz + 2yz é igual a 3 vezes a média aritmética simples de seus termos. Essa média é sempre maior
do que ou igual à média geométrica dos mesmos termos, isto é

1
q
( xy + 2xz + 2yz) ≥ 3 xy · 2xz · 2yz = 3 4x2 y2 z2 .
p
3

3
Supondo xyz = 32 (que é dado no problema), resulta que 3 4x2 y2 z2 = 3 22 · (25 )2 = 212 = 16. Então, multipli-
p p

cando por 3 dos dois lados, xy + 2xz + 2yz ≥ 48.

(c) A igualdade entre as médias aritmética e geométrica ocorre se, e somente se, os termos são iguais. Neste caso,
quando xy = 2xz = 2yz. Como o volume é positivo, x, y, z têm que ser positivos, em particular não nulos. Então,
da equação 2xz = 2yz tiramos y = x, e da equação xy = 2yz tiramos z = 2x . Como xyz = 32 então x · x · x
2 = 32, isto
é, x3 = 64 = 26 , ou ainda x = 4. Então x = y = 4 e z = 2.

6
MA12 – Matemática Discreta – Prova 3 – 2011

Questão 1.
A sequência 0, 3, 7, 10, 14, 17, 21, . . . é formada a partir do número 0 somando-se alternadamente 3 ou 4 ao termo
anterior, isto é: o primeiro termo é 0, o segundo é 3 a mais que o primeiro, o terceiro é 4 a mais que o segundo, o
quarto é 3 a mais que o terceiro, o quinto é 4 a mais que o quarto e assim sucessivamente.

(0,5) (a) Qual é o centésimo termo dessa sequência?

(0,5) (b) Qual é a soma dos 100 primeiros termos dessa sequência?

(1,0) (c) Algum termo desta sequência é igual a 2000? Por quê?

Questão 2.
Seja Rn o número máximo de regiões determinadas no plano por n cı́rculos.

(0,5) (a) Quais são os valores de R1 e R2 ?

(0,5) (b) Explique por que Rn+1 = Rn + 2n, para todo n ≥ 1.

(1,0) (c) Mostre por indução que Rn = n2 − n + 2.

Questão 3.
Suponha que o dinheiro valha 10% ao mês para um comerciante que vende determinado produto por R$ 4200,00
à vista.

(1,0) (a) Se o comerciante deseja oferecer o produto para compra em duas prestações iguais, a primeira no ato da compra,
qual deve ser o valor dessas prestações?

(1,0) (b) Suponha que ele deseja oferecer o produto em 10 prestações iguais, a primeira no ato da compra. Escreva uma
expressão que permita calcular o valor da prestação.

Questão 4.
Uma senha de banco é formada por 4 digı́tos de 0 a 9.

(1,0) (a) Quantas são as senhas em que aparecem exatamente três dı́gitos diferentes?

(1,0) (b) Quantas são as senhas em que não há dı́gitos consecutivos iguais?

Questão 5.
João, ao partir para uma viagem, ficou de enviar um cartão postal para sua mãe. A probabilidade de que ele envie
o cartão é igual a 0,7. Por outro lado, a probabilidade de um cartão postal se extraviar é 0,1.

(1,0) (a) Qual é a probabilidade de que a mãe de João receba um cartão postal dele?

(1,0) (b) Se ela não receber um cartão de João, qual é a probabilidade de que ele o tenha enviado?
AV3 - MA 12 - 2011

Questão 1.
A sequência 0, 3, 7, 10, 14, 17, 21, . . . é formada a partir do número 0 somando-se alternadamente 3 ou 4 ao termo
anterior, isto é: o primeiro termo é 0, o segundo é 3 a mais que o primeiro, o terceiro é 4 a mais que o segundo, o
quarto é 3 a mais que o terceiro, o quinto é 4 a mais que o quarto e assim sucessivamente.

(0,5) (a) Qual é o centésimo termo dessa sequência?

(0,5) (b) Qual é a soma dos 100 primeiros termos dessa sequência?

(1,0) (c) Algum termo desta sequência é igual a 2000? Por quê?

UMA SOLUÇÃO

(a) Chamemos de a1 , a2 , a3 , . . . os termos dessa sequência. A sequência dos termos com índices ímpares a1 , a3 , a5 , . . .
é uma progressão aritmética com termo inicial 0 e passo (ou razão) 7. A sequência dos termos com índices pares
a2 , a4 , a6 , . . . é uma progressão aritmética com termo inicial 3 e passo 7. O centésimo termo é o 50o da sequência dos
pares. Então a100 = 3 + (50 − 1) · 7 = 3 + 343 = 346.

(b) Há maneiras diferentes de se fazer isso. Podemos agrupar a soma assim:

( a1 + a100 ) + ( a2 + a99 ) + ( a3 + a98 ) + . . . + ( a50 + a51 ) .

Veja que de a1 para a2 há um acréscimo de 3 e de a99 para a100 também. Então os dois primeiros termos são iguais.
Do segundo para o terceiro há um aumento e um decréscimo de 4, logo o terceiro termo é igual ao segundo. E assim
por diante. Então todos os termos entre parênteses são iguais ao primeiro, que vale 0 + 346 = 346. Como são 50
termos, a soma dá 50 · 346 = 17300.
Outro jeito de fazer é somar separadamente as sequências com índices ímpares e pares. No segundo caso (pares),
são 50 termos da progressão aritmética de razão 7 começando em 3 e terminando em 346. A soma dessa progressão

3 + 346
50 · = 25 · 349 = 8725 .
2
No primeiro caso (ímpares), são 50 termos, mas todos 3 unidades menores do que os termos da série par. Então a
soma desses é 8725 subtraído de 50 · 3 = 150, isto é, dá 8575. Juntando as duas, ficamos com 17300.
Obs. Essa segunda soma também sairia da mesma forma como a outra, pois a PA tem primeiro termo igual a 0,
último termo igual a 343, totalizando 50 termos, logo soma
0 + 343
50 · = 25 · 343 = 8575 .
2

(c) Observe primeiro que se n é ímpar então an é múltiplo de 7, e se n é par então an − 3 é múltiplo de 7 (de fato,
valem as recíprocas, mas não precisaremos disso).
Como nem 2000 = 7 · 285 + 5 nem 1997 = 7 · 285 + 2 são múltiplos de 7, então 2000 não pode ser um an nem para
n par nem para n ímpar.

1
AV3 - MA 12 - 2011

Questão 2.
Seja Rn o número máximo de regiões determinadas no plano por n círculos.

(0,5) (a) Quais são os valores de R1 e R2 ?

(0,5) (b) Explique por que Rn+1 = Rn + 2n, para todo n ≥ 1.

(1,0) (c) Mostre por indução que Rn = n2 − n + 2.

UMA SOLUÇÃO

(a) Um único círculo no plano determina exatamente duas regiões (dentro e fora). Então R1 = 2. Agora colocamos
um segundo círculo no plano e olhamos para várias possibilidades: (i) se ele for idêntico ao primeiro, continuamos
com duas regiões; (ii) se um dos círculos está inteiramente contido numa das regiões delimitadas pelo outro, então
ficam delimitadas 3 regiões (mesma coisa se apenas se tangenciam); (iii) se eles se intersectam sem se tangenciarem,
ficam delimitadas 4 regiões. Esse é o máximo possível, então R2 = 4.

(b) Primeiro verifiquemos se a fórmula está compatível com a resposta anterior. Pela fórmula, deveríamos ter
R2 = R1 + 2 · 1. De fato, R1 + 2 · 1 = 2 + 2 = 4.
Agora imaginemos que n círculos já estão desenhados, definindo um certo número de regiões. Então desenha-
mos um novo círculo (diferente dos anteriores, pois neste caso a divisão de regiões permaneceria a mesma), que
intersectará os círculos anteriores em um certo número de pontos. Como o novo círculo só pode intersectar cada
um dos outros círculos em no máximo 2 pontos, ele terá no máximo 2n intersecções. Essas intersecções dividirão o
círculo em arcos de círculo, que serão no máximo 2n (e no mínimo 1, que é quando o círculo não intersecta nenhum
dos círculos já desenhados). Chamemos de k o número de arcos de círculo obtidos.
Agora suponha que numeremos esses k arcos de círculo, e vamos desenhar o n + 1-ésimo círculo arco por arco,
contando qual é o máximo acréscimo de regiões em cada etapa. O primeiro arco está inteiramente contido em uma
das regiões previamente delimitadas, e a divide em duas regiões. Isso acrescenta uma unidade na contagem de
regiões. Como o segundo arco só pode intersectar os círculos anteriores e o primeiro arco em seus extremos, ele
também está inteiramente contido em uma das regiões, incluindo as novas regiões formadas pela introdução do
primeiro arco. Ele dividirá essa região em duas, acrescentando mais uma unidade na contagem. Esse raciocínio
pode ser repetido de forma indutiva até chegarmos no k-ésimo arco. No total, serão acrescentadas k regiões à
contagem.
Como k ≤ 2n, então são acrescentadas no máximo 2n regiões à contagem, quando se passa de n círculos para
n + 1 círculos. Portanto, se n círculos não podem dividir o plano em mais do que Rn regiões, então n + 1 círculos
não poderão dividir o plano em mais do que Rn + 2n regiões. Isso define o valor de Rn+1 .
Observação. A rigor, dever-se-ia mostrar que, para cada n, alguma configuração de círculos divide o plano em
Rn regiões, para se dizer que Rn é o máximo (e não apenas uma cota superior). Para tanto, em vista do que foi
feito acima, basta achar uma lista de círculos C1 , C2 , C3 , . . . tal que, para qualquer n ≥ 1, o círculo Cn+1 intersecta

2
cada círculo C1 , . . . , Cn em 2 pontos, produzindo ao todo 2n pontos de intersecção distintos entre si. Isso pode ser
realizado por
1
Ci = {( x, y); ( x − )2 + y2 = 1} , i = 1, 2, 3, . . . ,
i
isto é, Ci é o círculo de raio 1 e centro em ( 1i , 0). Uma conta simples mostra que Cn+1 intersecta Ci nos dois pontos
 s 
2
1 1 1 1 1 1
  
 + , ± 1− − .
2 n+1 i 2 i n+1

1
Como os valores de i são distintos para i = 1, 2, . . . , n, os 2n pontos de intersecção são todos distintos entre si.

(c) A fórmula vale para n = 1, pois 12 − 1 + 2 = 2 = R1 . Agora, supondo que ela vale para n, isto é, supondo
Rn = n2 − n + 2 verdadeira, queremos mostrar que também vale para n + 1, isto é, queremos mostrar que Rn+1 =
(n + 1)2 − (n + 1) + 2. Ora, a relação de recorrência nos dá Rn+1 = Rn + 2n; valendo a hipótese de que Rn =
n2 − n + 1, então

Rn+1 = Rn + 2n
= (n2 − n + 2) + 2n
= n2 + n + 2
= [(n + 1)2 − 2n − 1] + n + 2
= ( n + 1)2 − n − 1 + 2
= ( n + 1)2 − ( n + 1) + 2 .

3
AV3 - MA 12 - 2011

Questão 3. Suponha que o dinheiro valha 10% ao mês para um comerciante que vende determinado produto por
R$ 4200,00 à vista.

(1,0) (a) Se o comerciante deseja oferecer o produto para compra em duas prestações iguais, a primeira no ato da
compra, qual deve ser o valor dessas prestações?

(1,0) (b) Suponha que ele deseja oferecer o produto em 10 prestações iguais, a primeira no ato da compra. Escreva uma
expressão que permita calcular o valor da prestação.

UMA SOLUÇÃO

x 1 11×4200
(a) Se x for o valor da prestação, ele quer x + 1,1 = 4200. Isso dá x (1 + 1,1 ) = 4200. Então x = 21 = 2200.

(b) Pelo mesmo raciocínio, ele quer x tal que


x x x
x+ + +...+ = 4200 .
1, 1 1, 12 1, 19
Ou seja,  
x 1 + 1, 1−1 + 1, 1−2 + . . . + 1, 1−9 = 4200

e
1 − 1, 1−10
 
x = 4200 .
1 − 1, 1−1
Logo
1
1− 1,1 4200 1
x = 4200 · 1
= · .
1− 11 1 − 110
1,110 1,1

4
AV3 - MA 12 - 2011

Questão 4.
Uma senha de banco é formada por 4 digítos de 0 a 9.

(1,0) (a) Quantas são as senhas em que aparecem exatamente três dígitos diferentes?

(1,0) (b) Quantas são as senhas em que não há dígitos consecutivos iguais?

UMA SOLUÇÃO

(a) Se há exatamente 3 dígitos diferentes, então há dois dígitos iguais e mais dois outros, diferentes dele e diferentes
entre si. Há 10 possibilidades para o dígito que aparece repetido. Escolhido esse dígito, precisamos de 2 dígitos
entre os 9 restantes. Temos C92 = 36 escolhas para os dígitos restantes. Portanto, na escolha dos dígitos em que o
dígito repetido está determinado, temos 360 possibilidades.
Falta agora ver de quantas maneiras diferentes eles podem ser dispostos. Primeiro escolhemos a disposição dos
dois dígitos que não se repetem. Como há C42 possibilidades de escolha de duas entre quatro posições, temos um
total de 6 disposições possíveis.
Fixada as posições dos dígitos repetidos, temos 2 maneiras de colocar os outros dois dígitos.
Então cada uma das 360 escolhas dos 3 dígitos (com o dígito que se repete determinado) pode ser arranjada de 12
maneiras distintas, o que dá um total de 360 × 12 = 4320 senhas com exatamente 3 dígitos diferentes.

(b) Há 10 possibilidades para o primeiro dígito. Como o segundo só não pode ser igual ao primeiro, há 9 possibi-
lidades para o segundo (para cada escolha do primeiro). Mais uma vez, há 9 possibilidades para o terceiro (para
cada escolha dos dois primeiros) e 9 para o quarto (para cada escolha dos três primeiros). Então são 10 × 93 = 7290
possibilidades.

5
AV3 - MA 12 - 2011

Questão 5.
João, ao partir para uma viagem, ficou de enviar um cartão postal para sua mãe. A probabilidade de que ele envie
o cartão é igual a 0,7. Por outro lado, a probabilidade de um cartão postal se extraviar é 0,1.

(1,0) (a) Qual é a probabilidade de que a mãe de João receba um cartão postal dele?

(1,0) (b) Se ela não receber um cartão de João, qual é a probabilidade de que ele o tenha enviado?

UMA SOLUÇÃO

(a) A probabilidade de que um cartão não extravie, dado que foi enviado, é de 1 − 0, 1 = 0, 9. Portanto a probabi-
lidade de que a mãe de João receba um cartão de seu filho é igual à probabilidade de que seja enviado e não seja
extraviado (dado que foi enviado), isto é 0, 9 × 0, 7 = 0, 63.

(b) A probabilidade de a mãe não receber o cartão é igual a 1 − 0, 63 = 0, 37. A probabilidade de a mãe não receber
o cartão por não ter sido enviado é igual a 1 − 0, 7 = 0, 3 e a probabilidade de a mãe não receber o cartão por ter
se extraviado é 0, 1 × 0, 7 = 0, 07. Portanto, se for dado que ela não recebeu o cartão, a probabilidade de que ele o
tenha enviado é de 0, 07/0, 37 = 0, 7/3, 7 = 7/37.

6
MA12
-
2012
MA12 – Matemática Discreta – AV1 – 2012

Atenção: esta prova deve ser feita sem o uso de calculadoras!

Questão 1.

Uma venda imobiliária envolve o pagamento de 12 prestações mensais iguais a R$ 10.000,00, a primeira no ato
da venda, acrescidas de uma parcela final de R$ 100.000,00, 12 meses após a venda. Suponha que o valor do
dinheiro seja de 2% ao mês.
(a) (1,0) Se o comprador preferir efetuar o pagamento da parcela final junto com a última prestação, de quanto
deverá ser o pagamento dessa parcela?
(b) (1,0) Se o comprador preferir efetuar o pagamento à vista, qual deverá ser o valor desse pagamento único?

São dados alguns valores aproximados de 1, 02n :

n 1, 02n
-12 0,788
-1 0,980
12 1,268

Questão 2.

A figura abaixo mostra uma linha poligonal que parte da origem e passa uma vez por cada ponto do plano
cujas coordenadas são números inteiros e não negativos.
(a) (1,0) O conjunto dos pares de números inteiros e não negativos tem a mesma cardinalidade que os números
naturais? Por quê?
(b) (1,0) Mostre que o comprimento da linha poligonal da origem até o ponto (n, n) é n2 + n, para qualquer inteiro
não negativo n.
(c) (0,5) Qual é o comprimento da linha poligonal da origem até o ponto (10, 13)?

1
MA12 – Matemática Discreta – AV1 – 2012

Questão 3.

(1,5) Mostre, por indução finita, que se n é um inteiro positivo então 7n − 1 é divisı́vel por 6.

Questão 4.

Considere a recorrência xn+2 − 4xn = 9n, com as condições iniciais x0 = x1 = 0.


(a) (0,5) Encontre a solução geral da recorrência homogênea xn+2 − 4xn = 0.
(b) (0,5) Determine os valores de A e B para os quais xn = A + nB é uma solução da recorrência xn+2 − 4xn = 9n.
(c) (1,0) Encontre a solução da recorrência original.

Questão 5.

Para todo número natural n ≥ 2, considere o número N formado por n − 1 algarismos iguais a 1, n algarismos
iguais a 2 e um algarismo igual a 5, nesta ordem.
(a) (1,0) Mostre que o número N pode ser escrito na forma

A · 102n + B · 10n + C
,
9
onde A, B e C são constantes independentes de n. Indique os valores de A, B e C.
(b) (0,5) Mostre que N é um quadrado perfeito.

(c) (0,5) Quantos algarismos tem N ? Diga quais são esses algarismos.

2
AV1 - MA 12 - 2012

Questão 1.

Uma venda imobiliária envolve o pagamento de 12 prestações mensais iguais a R$ 10.000,00, a primeira no
ato da venda, acrescidas de uma parcela final de R$ 100.000,00, 12 meses após a venda. Suponha que o valor
do dinheiro seja de 2% ao mês.
(a) Se o comprador preferir efetuar o pagamento da parcela final junto com a última prestação, de quanto
deverá ser o pagamento dessa parcela?
(b) Se o comprador preferir efetuar o pagamento à vista, qual deverá ser o valor desse pagamento único?

São dados alguns valores aproximados de 1, 02n :

n 1, 02n
-12 0,788
-1 0,980
12 1,268

UMA SOLUÇÃO

(a) O valor de R$100.000,00 trazido um mês para trás é igual a

1
100.000, 00 × ≃ 0, 980 × 100.000, 00 = 98.000, 00 .
1, 02

(b) Trazendo os valores para a data de compra, o comprador pagará

10.000, 00 10.000, 00 10.000, 00 100.000, 00


10.000, 00 + + +···+ + .
1, 02 1, 022 1, 0211 1, 0212
Isso é igual a

10.000, 00 × (1 + 1, 02−1 + 1, 02−2 + . . . + 1, 02−11 ) + 100.000, 00 × 1, 02−12


1 − 1, 02−12
= 10.000, 00 × + 100.000, 00 × 1, 02−12
1 − 1, 02−1
1 − 0, 788
≃ 10.000, 00 × + 100.000, 00 × 0, 788
1 − 0, 980
= 106.000, 00 + 78.800, 00 = 184.800, 00 .

Portanto, se o dinheiro vale 2% ao mês, pagar o esquema de prestações do enunciado equivale a pagar (aproxima-
damente) R$ 184.800,00 à vista.

1
AV1 - MA 12 - 2012

Questão 2.

A figura abaixo mostra uma linha poligonal que parte da origem e passa uma vez por cada ponto do plano
cujas coordenadas são números inteiros e não negativos.
(a) O conjunto dos pares de números inteiros e não negativos tem a mesma cardinalidade que os números
naturais? Por quê?
(b) Mostre que o comprimento da linha poligonal da origem até o ponto (n, n) é n2 + n, para qualquer inteiro
não negativo n.
(c) Qual é o comprimento da linha poligonal da origem até o ponto (10, 13)?

UMA SOLUÇÃO

(a) Chamemos de Z ≥0 o conjunto dos inteiros não negativos. Então o conjunto dos pontos de R2 com coordenadas
inteiras e não negativas é o produto cartesiano Z2≥0 = Z ≥0 × Z ≥0 .
Imaginemos que a linha é percorrida com velocidade 1 a partir do instante 1 em (0, 0). A figura mostra que se
no instante k a curva está num ponto de Z2≥0 então no instante k + 1 ela estará em um outro ponto de Z2≥0 . Por
indução, estabelece-se uma função f : N → Z2≥0 em que f (k) é o ponto de Z2≥0 alcançado no instante k.
Como todos os pontos são atingidos, f é sobrejetiva. Como a linha não passa mais do que uma vez em cada
ponto, f é injetiva. Assim, existe uma bijeção entre N e Z2≥0 , mostrando que N e Z2≥0 têm a mesma cardinalidade.

(b) Por inspeção a afirmação é verdadeira para n = 0, pois n2 + n = 0 e realmente são 0 passos para chegar no
ponto de partida (n, n) = (0, 0). Agora suponhamos que a afirmação é válida para (n, n), isto é, que realmente são
n2 + n passos até se chegar em (n, n) (hipótese de indução). Queremos mostrar que a afirmação é válida quando
aplicada para n + 1, isto é, que são (n + 1)2 + (n + 1) passos até se chegar em (n + 1, n + 1).
De (n, n) até (n + 1, n + 1) são necessários: n passos (para encontrar um dos eixos; mais especificamente, para
encontrar a abscissa, se n é par, e para encontrar a ordenada, se n é ímpar) mais 1 passo (para avançar nesse eixo)
mais n + 1 passos (para voltar à diagonal, que é o conjunto dos pontos da forma ( x, x )). Assim, são necessários
n + 1 + (n + 1) = 2n + 2 passos para ir-se de (n, n) a (n + 1, n + 1). Pela hipótese de indução, já foram n2 + n passos
para se chegar em (n, n). Portanto são (n2 + n) + (2n + 2) passos até (n + 1, n + 1). Mas

(n2 + n) + (2n + 2) = (n2 + 2n + 1) + (n + 1) = (n + 1)2 + (n + 1) ,

2
como queríamos demonstrar.

Solução alternativa 1. Para se chegar ao ponto (n, n), é preciso percorrer todos os pontos de coordenadas inteiras
do quadrado [0, n] × [0, n], exceto os situados em um dos lados. Existem (n + 1)2 pontos de coordenadas inteiras
no quadrado, dos quais n não são visitados. Logo, o comprimento da poligonal é (n + 1)2 − 1 − n = n2 + n.

Solução alternativa 2. A linha poligonal da origem até o ponto (n, n) é formada por n segmentos de comprimento
1, por segmentos de comprimento 2k, para k variando de 1 a n − 1 e um segmento de comprimento n. Logo, seu
comprimento é
( n − 1) n
n + 2(1 + 2 + ... + n − 1) + n = n + 2 · + n = n2 + n .
2

(c) Primeiro, investiga-se se (10, 13) ocorre a 3 passos de distância (para mais ou para menos) de (10, 10) ou de
(13, 13), no trajeto definido pela curva. Vemos que (10, 13) está 3 unidades verticalmente acima de (10, 10) e 3
unidades horizontalmente à esquerda de (13, 13). Quando (n, n) é par, como é o caso de n = 10, a linha poligonal
prossegue na vertical para baixo, portanto no sentido contrário ao que esperaríamos se fosse encontrar (10, 13) em
3 passos. Quando (n, n) é ímpar, como é o caso de n = 13, a linha poligonal prossegue horizontalmente para a
esquerda. Neste caso, encontrará (10, 13) após 3 passos.
Portanto, como são 132 + 13 = 169 + 13 = 182 passos até (13, 13) e mais 3 passos até (10, 13), então são 185 passos
até (10, 13).

3
AV1 - MA 12 - 2012

Questão 3.

Mostre, por indução finita, que se n é um inteiro positivo então 7n − 1 é divisível por 6.

UMA SOLUÇÃO

Para n = 1, 7n − 1 = 7 − 1 = 6, que é divisível por 6. Então a afirmação vale para n = 1. Suponhamos que a
afirmação seja válida para n, isto é, suponha que 7n − 1 seja múltiplo de 6. Vamos mostrar, com essa hipótese, que
7n+1 − 1 também é múltiplo de 6.
Ora, 7n+1 − 1 = 7n+1 − 7n + 7n − 1 = 7n (7 − 1) + (7n − 1) = 6 · 7n + (7n − 1). O primeiro termo é múltiplo de
6, porque tem um fator 6, e o segundo também é, pela hipótese de indução. Então a soma é múltiplo de 6 e temos
demonstrado o que queríamos.

4
AV1 - MA 12 - 2012

Questão 4.

Considere a recorrência xn+2 − 4xn = 9n, com as condições iniciais x0 = x1 = 0.


(a) Encontre a solução geral da recorrência homogênea xn+2 − 4xn = 0.
(b) Determine os valores de A e B para os quais xn = A + nB é uma solução da recorrência xn+2 − 4xn = 9n.
(c) Encontre a solução da recorrência original.

UMA SOLUÇÃO

(a) Se xn+2 − 4xn = 0 então xn+2 = 4xn . Então x2m = 4m x0 , para todo m ≥ 0, e x2m+1 = 4m x1 , para todo n ≥ 0.
Escrevendo de outra maneira, a solução é

x0 , x1 , 4x0 , 4x1 , 42 x0 , 42 x1 , 43 x0 , 43 x1 , . . .

1
Também pode-se dizer que xn = 2n x0 , para n ≥ 0 par, e xn = 2 · 2n x1 , para n ≥ 0 ímpar.

(b) Se xn = A + nB então xn+2 = A + (n + 2) B. Se, além do mais, ( xn ) é solução de xn+2 − 4xn = 9n, então

9n = xn+2 − 4xn = A + (n + 2) B − 4A − 4Bn = −3A + 2B − 3nB .

Para que −3A + 2B − 3nB seja igual a 9n basta que −3A + 2B = 0 (primeira equação) e que −3B = 9 (segunda
equação). Da segunda equação sai imediatamente que B = −3, e, colocando esse valor na primeira, que A = −2.
Então xn = −2 − 3n é uma solução da equação não homogênea.

(c) Agora vamos combinar a solução geral da homogênea com a solução particular da não homogênea para obter
a solução de xn+2 − 4xn = 9n com x0 = x1 = 0. Seja xen = −2 − 3n a solução calculada em (b), que satisfaz
xen+2 − 4e
xn = 9n. Essa solução não satisfaz as condições iniciais pedidas, pois xe0 = −2 e xe1 = −5. Então seja
( xbn ) solução da homogênea satisfazendo xb0 = +2 e xb1 = +5. Vamos verificar que ( xn ) definida por xn = xen + xbn
satisfaz ao mesmo tempo as condições iniciais e a relação de recorrência não homogênea.
Ora, x0 = xe0 + xb0 = −2 + 2 = 0 e x1 = xe1 + xb1 = −5 + 5 = 0. Além disso,

xn+2 − 4xn = ( xen+2 − 4e


xn ) + ( xbn+2 − 4b
xn ) = 9n + 0 = 9n .

Assim, a solução do problema proposto é a sequência dada por xn = 2 · 2n − 3n − 2 = 2n+1 − 3n − 2, para n par,
5
xn = 2 · 2n − 3n − 2, para n ímpar.

5
OUTRA SOLUÇÃO

Esta é a solução que muitos esperavam, que usa equação característica.


(a) A equação característica é r2 − 4 = 0, cujas raízes são −2 e +2. Logo a solução geral da recorrência é xn =
C · 2n + D · (−2)n .

Obs. Note que, embora esta resposta seja diferente da resposta (a) da solução anterior, ambas estão corretas, mas
estão expressas em termo de outras constantes.

(b) Idêntica à resposta (b) da solução anterior.

(c) Somando as duas, obtemos a solução geral

xn = C · 2n + D · (−2)n − 2 − 3n .

Usando as condições iniciais x0 = 0 e x1 = 0, temos

C+D−2 = 0
2C − 2D − 2 − 3 = 0

9
Resolvendo o sistema, obtemos C = 4 e D = − 41 . Logo, a solução é

9 n 1
xn = · 2 − · (−2)n − 2 − 3n .
4 4
Não é difícil verificar que as duas soluções apresentadas são a mesma, mas escritas de formas diferentes.

6
AV1 - MA 12 - 2012

Questão 5.

Para todo número natural n ≥ 2, considere o número N formado por n − 1 algarismos iguais a 1, n algarismos
iguais a 2 e um algarismo igual a 5, nesta ordem.
(a) Mostre que o número N pode ser escrito na forma

A · 102n + B · 10n + C
,
9
onde A, B e C são constantes independentes de n. Indique os valores de A, B e C.
(b) Mostre que N é um quadrado perfeito.

(c) Quantos algarismos tem N? Diga quais são esses algarismos.

UMA SOLUÇÃO

(a) Usando a expansão na base decimal, podemos escrever N como

N = 102n−1 + 102n−2 + . . . + 10n+1 + 2 · 10n + 2 · 10n−1 + . . . + 2 · 101 + 5 .

Então
N = 10n+1 (1 + 10 + . . . + 10n−2 ) + 2 · 10 · (1 + 10 + . . . + 10n−1 ) + 5 .

Somando as duas PGs entre parênteses,

10n−1 − 1 10n − 1
N = 10n+1 · + 20 · +5
10 − 1 10 − 1
102n − 10n+1 + 20 · 10n − 20 + 45
=
9
102n + 10 · 10n + 25
= .
9
Portanto A = 1, B = 10 e C = 25.

Obs: Outra forma de fazer é multiplicar N por 9 usando o algoritmo de multiplicação e ver que fica o número
10 . . . 010 . . . 025, onde o bloco de zeros mais à esquerda tem n − 2 elementos e o bloco de zero mais à direita tem
n − 1 elementos.

(b) Queremos saber se N = p2 , com p ∈ N. Como 102n + 10 · 10n + 25 = (10n + 5)2 , então
 2
10n + 5
N= .
3

Resta saber se 10n + 5 é divisível por 3. Mas isso é verdade, porque como 10n + 5 = 10 . . . 05, com um bloco de n − 1
zeros, a soma dos algarismos desse número é igual a 6.

7
10n +5
(c) A raiz de N é o número p = 3 . Como 10n + 5 = 10 . . . 05, com um bloco de n − 1 zeros, então tem n + 1
algarismos. Ao dividir por 3, passa a ter n algarismos. Então p tem n algarismos.
Para saber qual é o número, podemos escrever

10n − 1 6
p= + .
3 3
O termo da esquerda é 33 . . . 3 (n vezes) e o da direita é igual a 2. Então p = 3 . . . 35, onde 3 aparece repetido n − 1
vezes.

8
MA12 – Matemática Discreta – AV2 – 2012

Nesta prova, todos os itens têm mesmo peso.

Questão 1.

Num porta-CDs, cabem 10 CDs colocados um sobre o outro, formando uma pilha vertical. Tenho 3 CDs de
MPB, 5 de rock e 2 de música clássica.
(a) De quantos modos diferentes posso empilhá-los de modo que todos os CDs de rock fiquem juntos?
(b) De quantos modos posso escolher 4 CDs para levar em uma viagem, de modo que eu leve pelo menos um
CD de cada tipo de música?

Questão 2.

Em uma caixa há 10 bolas idênticas, numeradas de 1 a 10. O número de cada bola corresponde a um dos
pontos da figura, os quais dividem a circunferência em 10 partes iguais. Nos itens a seguir, considere que as
bolas são retiradas ao acaso, uma a uma e sem reposição.
(a) Se forem retiradas duas bolas, qual é a probabilidade de que os pontos correspondentes sejam extremidades
de um diâmetro?
(b) Se forem retiradas três bolas, qual é a probabilidade de que os pontos correspondentes sejam vértices de
um triângulo isósceles?

1
MA12 – Matemática Discreta – AV2 – 2012

Questão 3.

Em uma caixa foram colocados um cartão no qual está escrito o número 1, dois cartões nos quais está escrito
o número 2, três cartões com o número 3 e assim por diante, até dez cartões com o número 10.
(a) Qual é o número mı́nimo de cartões que devem ser retirados da caixa, sem olhar, de modo que se tenha
certeza de que haja, entre os cartões retirados, 5 deles com o mesmo número?
(b) Qual é o número mı́nimo de cartões que devem ser retirados da caixa, sem olhar, de modo que se tenha
certeza de que haja, entre os cartões retirados, pelo menos um par de cartões com diferença maior do que
5?

Questão 4.

A média aritmética de 10 números positivos é igual a 1. Os números são agrupados aos pares e os números de
cada par somados, resultando daı́ um conjunto de 5 números positivos.
(a) O que se pode dizer sobre a média aritmética desses 5 números?
(b) Mostre que o produto desses 5 números é menor ou igual a 32.

Questão 5.

Uma moeda, com probabilidade 0,6 de dar cara, é lançada duas vezes.
(a) Qual é a probabilidade de que se observem resultados iguais no primeiro e segundo lançamentos?
(b) Dado que os resultados observados no primeiro e segundo lançamentos são iguais, qual é a probabilidade
condicional de que o resultado observado neles seja cara?

2
AV2 - MA 12 - 2012

Questão 1.

Num porta-CDs, cabem 10 CDs colocados um sobre o outro, formando uma pilha vertical. Tenho 3 CDs de
MPB, 5 de rock e 2 de música clássica.
(a) De quantos modos diferentes posso empilhá-los de modo que todos os CDs de rock fiquem juntos?
(b) De quantos modos posso escolher 4 CDs para levar em uma viagem, de modo que eu leve pelo menos
um CD de cada tipo de música?

UMA SOLUÇÃO

(a) Vamos fixar as posições dos CDs atribuindo números de 1 a 10 a suas posições, contando de baixo para cima.
Se todos os 5 CDs de rock ficam juntos, o primeiro pode ficar nas posições de 1 a 6, portanto são 6 escolhas para a
posição do bloco de CDs de rock. Os 5 CDs de rock podem ser arrumados de 5! = 120 maneiras dentro do bloco. As
posições restantes são 5 e os demais CDs também podem ser ordenados de 120 maneiras nessas posições restantes
(não importa que o bloco de CDs de rock interrompa a sequência). Portanto são 6 · 120 · 120, isto é, 86400 maneiras.

(b) Para escolher 4 CDs com pelo menos um para cada tipo de música, podemos escolher, primeiro, um de cada
tipo. Temos 3 possibilidades para MPB, 5 para rock e 2 para música clássica, perfazendo 3 · 5 · 2 = 30 possibilidades.
Depois dessa escolha, podemos pegar qualquer um dos 7 CDs restantes. São, portanto, 30 · 7 = 210 escolhas. No
entanto, temos que dividir por 2 esse valor, já que os dois CDs de mesmo gênero, digamos A e B, podem aparecer
com A na primeira escolha e B na segunda, ou vice-versa.
Outra maneira de resolver (mais complicada, mas que evita a divisão por dois no final): dos 4 CDs, dois são do
mesmo gênero (e os outros dois dos dois outros gêneros restantes). Se os dois de gênero repetido forem de música
clássica, são todos os disponíveis para esse gênero, de forma que restam 3 · 5 escolhas para os outros dois; são,
portanto, 15 possibilidades para se ter 2 CDs repetidos de música clássica. Se os dois de gênero repetido forem
de MPB, há C3,2 = 3 escolhas para eles; para cada uma delas, restam 2 · 5 escolhas dos outros dois; portanto, são
30 maneiras para se ter dois CDs de MPB. Finalmente, se os dois de gênero repetido forem de rock, há C5,2 = 10
escolhas para os dois repetidos, e 2 · 3 escolhas para os outros dois, perfazendo 6 · 10 = 60 possibilidades com dois
CDs de rock. No total, são 60 + 30 + 15 = 105 possibilidades.

1
AV2 - MA 12 - 2012

Questão 2.

Em uma caixa há 10 bolas idênticas, numeradas de 1 a 10. O número de cada bola corresponde a um dos
pontos da figura, os quais dividem a circunferência em 10 partes iguais. Nos itens a seguir, considere que as
bolas são retiradas ao acaso, uma a uma e sem reposição.
(a) Se forem retiradas duas bolas, qual é a probabilidade de que os pontos correspondentes sejam extremi-
dades de um diâmetro?
(b) Se forem retiradas três bolas, qual é a probabilidade de que os pontos correspondentes sejam vértices de
um triângulo isósceles?

UMA SOLUÇÃO

(a) A primeira bola pode ser qualquer uma. Das 9 restantes, apenas uma será diametralmente oposta a essa
primeira. Portanto a probabilidade de isso ocorrer é de 1/9.

(b) Escolhidos 3 pontos da figura, ficam definidos também 3 intervalos entre eles. Lados iguais de um triângulo
ocorrem se, e somente se, os correspondentes intervalos entre os pontos são iguais. Em particular, (i) os pontos
formam um triângulo isósceles se, e somente se, pelo menos dois desses intervalos são iguais; (ii) nenhum triângulo
equilátero pode ser formado, já que 10 não é divisível por 3.
Nunca havendo 3 intervalos iguais, definimos de forma unívoca o “ponto do meio´´ de um triângulo isósceles
àquele ladeado pelos dois intervalos iguais. Há 10 possibilidades para esse ponto do meio. Os intervalos iguais que
ladeiam esse ponto do meio podem ter os tamanhos: 1/10, 2/10, 3/10 e 4/10. Portanto são 10 · 4 = 40 maneiras de
tomar 3 desses 10 pontos como vértices de um triângulo isósceles.
Por outro lado, há C10,3 = 10 · 9 · 8/6 = 120 maneiras de se escolher 3 entre as 10 bolas. Portanto, dessas 120
escolhas, 40 levarão a um triângulo isósceles, e a probabilidade de isso ocorrer será de 40/120 = 1/3.

2
AV2 - MA 12 - 2012

Questão 3.

Em uma caixa foram colocados um cartão no qual está escrito o número 1, dois cartões nos quais está escrito
o número 2, três cartões com o número 3 e assim por diante, até dez cartões com o número 10.
(a) Qual é o número mínimo de cartões que devem ser retirados da caixa, sem olhar, de modo que se tenha
certeza de que haja, entre os cartões retirados, 5 deles com o mesmo número?
(b) Qual é o número mínimo de cartões que devem ser retirados da caixa, sem olhar, de modo que se tenha
certeza de que haja, entre os cartões retirados, pelo menos um par de cartões com diferença maior do que
5?

UMA SOLUÇÃO

(a) Os 5 cartões de mesmo número não podem ser os cartões numerados de 1 a 4. Esses cartões são 10. Há 6
números que são candidatos a terem 5 cartões repetidos: 5, 6, 7, 8, 9 e 10. Então, pelo Princípio das Gavetas, é
suficiente retirar 10 + 4 · 6 + 1 = 35 cartões.

(b) Não aparecem dois números com diferença maior do que 5 enquanto todos os cartões retirados tiverem todos
os números dentro de uma mesma sequência de 6 números consecutivos (um “bloco de 6”). Os blocos de 6 possíveis
são: 1-2-3-4-5-6, 2-3-4-5-6-7, 3-4-5-6-7-8, 4-5-6-7-8-9 e 5-6-7-8-9-10. O bloco com mais cartões é o último: ele tem
5 + 6 + 7 + 8 + 9 + 10 = 45 cartões. Então, pelo Princípio das Gavetas, com 46 cartões retirados não é possível que
todos eles estejam num mesmo bloco de 6, ou seja, certamente existirá um par com diferença maior do que 5.

3
AV2 - MA 12 - 2012

Questão 4.

A média aritmética de 10 números positivos é igual a 1. Os números são agrupados aos pares e os números
de cada par somados, resultando daí um conjunto de 5 números positivos.
(a) O que se pode dizer sobre a média aritmética desses 5 números?
(b) Mostre que o produto desses 5 números é menor ou igual a 32.

UMA SOLUÇÃO

(a) Sejam x1 , x2 , x3 , x4 , x5 , x6 , x7 , x8 , x9 , x10 os 10 números positivos. A primeira informação é de que


x1 + x2 + x3 + x4 + x5 + x6 + x7 + x8 + x9 + x10
= 1.
10
Ao agruparmos aos pares esses números e somarmos, obteremos a mesma soma do numerador. Mas, ao tirar a
média dos 5, dividiremos por 5, e não por 10. Portanto o resultado será igual a 2.

(b) Sejam y1 , y2 , y3 , y4 , y5 os 5 números positivos aos quais se refere o enunciado. A média geométrica dos 5
números é menor ou igual a sua média aritmética, isto é,
√ y1 + y2 + y3 + y4 + y5
5 y1 · y2 · y3 · y4 · y5 ≤ .
5
Acabamos de concluir, no item (a), que o lado direito é igual a 2. Daí resulta

y1 · y2 · y3 · y4 · y5 ≤ 25 = 32 .

4
AV2 - MA 12 - 2012

Questão 5.

Uma moeda, com probabilidade 0,6 de dar cara, é lançada duas vezes.
(a) Qual é a probabilidade de que se observem resultados iguais no primeiro e segundo lançamentos?
(b) Dado que os resultados observados no primeiro e segundo lançamentos são iguais, qual é a probabilidade
condicional de que o resultado observado neles seja cara?

UMA SOLUÇÃO

(a) Evidentemente está-se supondo que os lançamentos são independentes. Para aparecerem resultados iguais
nos dois primeiros lançamentos, ou ocorrem duas caras, com probabilidade 0, 6 × 0, 6 = 0, 36, ou duas coroas,
com probabilidade 0, 4 × 0, 4 = 0, 16. Sendo cara-cara e coroa-coroa dois eventos disjuntos (se um deles ocorre o
outro não ocorre), a probabilidade total de ocorrerem dois resultados iguais nos dois primeiros lançamentos é de
0, 36 + 0, 16 = 0, 52.

(b) Se já se sabe que vão sair dois resultados iguais, a probabilidade de que seja cara-cara é de

0, 36 36 9
= = .
0, 52 52 13

5
MA12 – Matemática Discreta – AV3 – 2012

Questão 1.
1
Uma moeda, com probabilidade 3 de dar cara, é lançada 40 vezes.
(a) Explique por que a probabilidade pk de se obter k caras nos 40 lançamentos é dada por
 k  40−k
1 2
pk = C40,k ,
3 3

para k = 0, 1, 2, . . . , 40.
(b) Calcule para que valores de k tem-se pk+1 > pk .
(c) Utilize (b) para obter o valor de k para o qual a probabilidade de se obter k caras é máxima.

Questão 2.

A soma dos n primeiros termos de uma progressão aritmética é dada por Sn = 2n2 − 15n.
(a) Determine o décimo termo da progressão.
(b) Encontre o primeiro termo positivo da progressão.

Questão 3.

Um comerciante, para quem o dinheiro vale 3% ao mês, oferece determinado produto por 3 prestações mensais
iguais a R$ 100,00, a primeira paga um mês após a compra.
(a) Que valor o comerciante deve cobrar por este produto, no caso de pagamento à vista?
(b) Se um consumidor desejar pagar o produto em três prestações mensais iguais, sendo a primeira paga no
ato da compra, qual deve ser o valor das parcelas?

Utilize, se desejar, os seguintes valores para as potências de 1,03: 1, 032 = 1, 0609, 1, 033 = 1, 0927, 1, 03−1 = 0, 9709,
1, 03−2 = 0, 9426, 1, 03−3 = 0, 9151.

1
MA12 – Matemática Discreta – AV3 – 2012

Questão 4.

(a) Mostre, por indução finita, que


1 1 1 7
+ + ... + ≥
n+1 n+2 2n 12
para todo número natural n ≥ 2.
1 1 1
(b) Use este fato para explicar por que a soma 1 + 2 + 3 + 4 + . . . cresce sem limite.

Questão 5.

Cada bolinha nas figuras abaixo deve ser colorida com uma das cores azul, branca, vermelha ou preta, de modo
que as bolinhas ligadas por um segmento tenham cores diferentes.

(a) De quantos modos se pode colorir a figura da esquerda?


(b) De quantos modos se pode colorir a figura da direita?

2
AV3 - MA 12 - 2012

Questão 1.
1
Uma moeda, com probabilidade 3 de dar cara, é lançada 40 vezes.
(a) Explique por que a probabilidade pk de se obter k caras nos 40 lançamentos é dada por
 k  40−k
1 2
pk = C40,k ,
3 3
para k = 0, 1, 2, . . . , 40.
(b) Calcule para que valores de k tem-se pk+1 > pk .
(c) Utilize (b) para obter o valor de k para o qual a probabilidade de se obter k caras é máxima.

UMA SOLUÇÃO

(a) A probabilidade de saírem k caras e 40 − k coroas em 40 lançamentos, numa ordem específica, é a probabilidade
de sair cara elevada à potência k vezes a probabilidade de sair coroa elevada à potência 40 − k. Neste caso,
 k  40−k
1 2
.
3 3
Mas o número de maneiras (ou ordens) que podem sair as k caras é o número de maneiras de se escolher k elementos
entre 40, ou seja, C40,k . Por isso a fórmula do enunciado.

(b) Lembramos que


40!
C40,k = .
k!(40 − k )!
Então pk+1 > pk se, e somente se,
 k+1  40−k−1  k  40−k
40! 1 2 40! 1 2
> .
(k + 1)!(40 − k − 1)! 3 3 k!(40 − k)! 3 3
Ou seja, se, e somente se,
1 1 1 2
· > · ,
k+1 3 40 − k 3
38
cancelando os fatores comuns nos dois lados. Portanto pk+1 > pk se, e somente se, 40 − k > 2k + 2, isto é, k < 3 .
Como k é inteiro, isto é equivalente a k ≤ 12.

(c) De (b) vale


p0 < p1 < p2 < . . . < p12 < p13 .

Vale, também,
p13 ≥ p14 ≥ . . . ≥ p40 ,

embora valham, de fato, as desigualdades estritas, se for aplicado raciocínio análogo àquele feito em (b). O valor
máximo, ocorre, portanto, em k = 13.

1
AV3 - MA 12 - 2012

Questão 2.

A soma dos n primeiros termos de uma progressão aritmética é dada por Sn = 2n2 − 15n.
(a) Determine o décimo termo da progressão.
(b) Encontre o primeiro termo positivo da progressão.

UMA SOLUÇÃO

(a) O décimo termo é S10 − S9 , isto é,

(2 · 102 − 15 · 10) − (2 · 92 − 15 · 9) = 23 .

(b) Queremos saber para quais valores de n o n-ésimo termo, isto é, a expressão Sn − Sn−1 , é maior do que zero.
Temos
(2n2 − 15n) − (2(n − 1)2 − 15(n − 1)) = 4n − 17 ,

logo o primeiro termo positivo ocorre para o primeiro n tal que 4n − 17 > 0, isto é, para n = 5.

2
AV3 - MA 12 - 2012

Questão 3.

Um comerciante, para quem o dinheiro vale 3% ao mês, oferece determinado produto por 3 prestações mensais
iguais a R$ 100,00, a primeira paga um mês após a compra.
(a) Que valor o comerciante deve cobrar por este produto, no caso de pagamento à vista?
(b) Se um consumidor desejar pagar o produto em três prestações mensais iguais, sendo a primeira paga no
ato da compra, qual deve ser o valor das parcelas?

Utilize, se desejar, os seguintes valores para as potências de 1,03: 1, 032 = 1, 0609, 1, 033 = 1, 0927, 1, 03−1 = 0, 9709,
1, 03−2 = 0, 9426, 1, 03−3 = 0, 9151.

UMA SOLUÇÃO

(a) Trazendo os valores das prestações a valor presente e somando, obtemos o valor para o pagamento à vista:

100 100 100 100


+ 2
+ = (1 + 1, 03−1 + 1, 03−2 )
1, 03 1, 03 1, 033 1, 03
100 1 − 1, 03−3
= ·
1, 03 1 − 1, 03−1
1 − 0, 9151 8, 49
≃ 100 · = 100 · = 283 .
0, 03 3
O resultado pode ser um pouco diferente dependendo de como são usados os arredondamentos.

(b) Não é preciso saber o valor à vista. Basta trazer em 1 mês cada uma das prestações de 100 reais. Ou seja, cada
uma deve ser de 100 · 1, 03−1 , que é aproximadamente igual a 97,09 reais.

3
AV3 - MA 12 - 2012

Questão 4.

(a) Mostre, por indução finita, que


1 1 1 7
+ +...+ ≥
n+1 n+2 2n 12
para todo número natural n ≥ 2.
1
(b) Use este fato para explicar por que a soma 1 + 2 + 31 + 14 + . . . cresce sem limite.

UMA SOLUÇÃO

1
+ 41 , que é igual a 12
(a) Primeiro vejamos que a desigualdade vale para n = 2. O lado esquerdo, neste caso, é 7
.3
Agora suponhamos que a desigualdade valha para um certo n ≥ 2. Iremos mostrar que a correspondente desi-
gualdade também vale para n + 1. Isto é, supondo que vale

1 1 1 7
+ +...+ ≥
n+1 n+2 2n 12
(hipótese de indução), mostraremos que vale

1 1 1 7
+ +...+ ≥ .
( n + 1) + 1 ( n + 1) + 2 2( n + 1) 12

Ora, mas o lado esquerdo pode ser escrito como

1 1 1 1 1 1
 
+ +...+ + + − .
n+1 n+2 2n 2n + 1 2n + 2 n + 1
7
Pela hipótese de indução, a soma entre parênteses é maior do que ou igual a 12 . Então basta mostrar que

1 1 1
+ − ≥ 0.
2n + 1 2n + 2 n + 1
Mas isso é verdade porque
1 1 2 1
+ > = .
2n + 1 2n + 2 2n + 2 n+1

(b) A soma mencionada pode ser agrupada assim:

1 1 1 1 1 1 1
     
1+ + + + +...+ + +...+ +...
2 3 4 5 8 9 16
1 1
Cada agrupamento entre parênteses é da forma + . . . + 2n
n +1 , com n = 2, 4, 8, . . ., isto é, n = 2k para k = 1, 2, 3, . . ..
7
E cada um deles é maior do que ou igual a 12 , conforme demonstrado em (a). Assim, a soma parcial até o termo
k 1 7
2 · 2 é maior do que ou igual a 1 + 2 + k · 12 .
7k
Como a sequência das somas parciais é crescente e para os valores de n = 2k+1 a soma parcial é maior do que 12 ,
então para qualquer valor real M > 0 existirá um n tal que a soma parcial até o n-ésimo termo supera o valor M.
Isso mostra que a série cresce sem limite.

4
AV3 - MA 12 - 2012

Questão 5.

Cada bolinha nas figuras abaixo deve ser colorida com uma das cores azul, branca, vermelha ou preta, de
modo que as bolinhas ligadas por um segmento tenham cores diferentes.

(a) De quantos modos se pode colorir a figura da esquerda?


(b) De quantos modos se pode colorir a figura da direita?

UMA SOLUÇÃO

(a) Na figura da esquerda, há 4 cores possíveis para a bolinha na posição mais alta. Uma vez fixada essa cor, a
bolinha na altura intermediária à esquerda tem 3 possibilidades, e, fixada esta, a da direita tem duas possibilida-
des. Para a bolinha inferior sobram duas possibilidades, ou a cor da bolinha superior ou a cor que não entrou em
nenhuma das 3 bolinhas mais acima. Então são 4 · 3 · 2 · 2 = 48 maneiras.

(b) A figura da direita poderia ser desenhada como a da esquerda, mas sem a ligação entre as duas bolinhas que
estão na posição intermediária. Se essas duas bolinhas têm a mesma cor, então são 4 cores para a bolinha superior,
3 para as intermediárias iguais, e 3 para a inferior (a inferior só não pode ser igual às intermediárias iguais). Isso
dá 4 · 3 · 3 = 36. Se as bolinhas intermediárias têm cores diferentes aí caímos no caso anterior, onde encontramos 48
maneiras. Então o número total de maneiras é 36 + 48 = 84.

5
MA12
-
2013
Sociedade Brasileira de Matemática
Mestrado Profissional em Matemática em Rede Nacional

MA12 – Matemática Discreta


Avaliação 1 - MA 12
27 de abril de 2013

1. (valor 3,0)
Paulo economizou durante muitos anos e tem, hoje, R$ 500.000,00 aplicados em um investi-
mento que rende juros de 1% ao mês. A partir do próximo mês, ele pretende fazer uma retirada
mensal de R$ 1.000,00.
a) Seja sn o saldo que resta da aplicação, após fazer a n-ésima retirada. Exprima sn+1 em
termos de sn . Dê também a condição inicial da recorrência obtida. (pontuação parcial 0,5)
b) Obtenha uma expressão para sn em função de n. (pontuação parcial 1,5)
c) Qual é a retirada mensal máxima que Paulo pode fazer de modo que o saldo da aplicação
nunca se torne negativo? (pontuação parcial 1,0)

2. (valor 2,5)
a) Para que valores de b existe uma progressão geométrica para a qual a soma dos n primeiros
termos é igual a 3n+1 + b, para todo n natural? (pontuação parcial 1,0)
b) Quais são o primeiro termo e a razão dessa progressão? (pontuação parcial 1,5)

3. (valor 2,0)
Prove, por indução finita, que

1 1 1 1 1 n
1+ + + + + . . . n−1 > ,
2 3 4 5 2 2

para todo n natural.


4. (valor 1,5)
Na figura abaixo temos uma espiral formada por infinitos semicírculos cujos centros pertencem
ao eixo das abscissas. Se o raio do primeiro semicírculo (o maior) é igual a 1 e o raio de cada
semicírculo é igual à metade do semicírculo anterior, determine:
a) o comprimento total da espiral. (pontuação parcial 0,75)
b) a abscissa do ponto P assintótico da espiral. (pontuação parcial 0,75)

1 2 x

5. (valor 1,0)
a) Se (an ) é uma progressão geométrica de termos positivos, prove que (bn ) definida por
bn = log an é uma progressão aritmética. (pontuação parcial 0,5)
b) Se (an ) é uma progressão aritmética, prove que (bn ) definida por bn = ean é uma progressão
geométrica. (pontuação parcial 0,5)
Sociedade Brasileira de Matemática
Mestrado Profissional em Matemática em Rede Nacional

MA12 – Matemática Discreta


Avaliação - GABARITO AV 1 - MA 12
27 de abril de 2013

1. (valor 3,0)
Paulo economizou durante muitos anos e tem, hoje, R$ 500.000,00 aplicados em um investi-
mento que rende juros de 1% ao mês. A partir do próximo mês, ele pretende fazer uma retirada
mensal de R$ 1.000,00.
a) Seja sn o saldo que resta da aplicação, após fazer a n-ésima retirada. Exprima sn+1 em
termos de sn . Dê também a condição inicial da recorrência obtida. (pontuação parcial 0,5)
b) Obtenha uma expressão para sn em função de n. (pontuação parcial 1,5)
c) Qual é a retirada mensal máxima que Paulo pode fazer de modo que o saldo da aplicação
nunca se torne negativo? (pontuação parcial 1,0)

Uma solução:
a) sn+1 = 1, 01.sn − 1 000, com s0 = 500 000 (ou s1 = 504 000).
b) Uma primeira solução pode ser feita resolvendo-se a recorrência acima. Uma solução da
equação homogênea associada an+1 = 1, 01.an é an = 1, 01n−1 .
1 000
Fazendo a substituição sn = an yn , obtemos yn+1 = yn − 1,01 n . Usando a recorrência recem

encontrada e somando os termos, encontramos

1 1
yn = y0 − 1 000(1 + 1
+ ... + )=
1, 01 1, 01n−1
1
1− 1,01n 1
= y0 − 1 000. 1 = y0 − 100 000.1, 01.(1 − )
1 − 1,01 1, 01n
.
1
Daí, sn = an yn = 1, 01n−1 y0 − 100 000.1, 01n .(1 − 1,01 n ) = 1, 01
n−1
y0 − 100 000(1, 01n − 1).

1
Finalmente, de s0 = a0 y0 , obtemos 500 000 = y,
1,01 0
ou seja y0 = 500 000.1, 01. Logo, a
expressão de sn é

sn = 500 000.1, 01n − 100 000(1, 01n − 1) = 400 000.1, 01n + 100 000.
Uma segunda solução desta questão pode ser feita utilizando-se o teorema sobre o valor de
1
1− n
uma série uniforme. O valor das n retiradas, no mês anterior à primeira retirada é 1 000. 0,01
1,01
=
1
100 000(1 − 1,01n ).
Para obter o valor dessas retiradas no mês da n-ésima retirada, devemos multiplicar o valor
no instante inicial por 1, 01n , obtendo 100 000(1, 01n − 1). O valor inicial do investimento, nesta
mesma época, é igual a 500 000 × 1, 01n . Portanto, o saldo restante após a n-ésima retirada é

500 000 × 1, 01n − 100 000(1, 01n − 1) = 400 000 × 1, 01n + 100 000.

c) A maior retirada possível é o rendimento mensal, igual a 0, 01 × 500 000 = 5 000. Podemos
chegar a este resultado resolvendo o item b) para uma retirada genérica p, para a qual obteremos,
após a n-ésima retirada, o valor (50 0000 − 100p) × 1, 01n + 100 000.
Para que este valor nunca se torne negativo, devemos ter 500 000 − 100p ≥ 0, ou seja,
p ≤ 5 000.

2. (valor 2,5)
a) Para que valores de b existe uma progressão geométrica para a qual a soma dos n primeiros
termos é igual a 3n+1 + b, para todo n natural? (pontuação parcial 1,0)
b) Quais são o primeiro termo e a razão dessa progressão? (pontuação parcial 1,5)

Uma solução:
A soma dos n primeiros termos da progessão geométrica de primeiro termo a1 e razão q é

qn − 1 a1 q n a1
S n = a1 = −
q−1 q−1 q−1
a1
Esta expressão deve ser idêntica a 3.3n + b. Devemos ter, portanto, q = 3 e q−1
= 3. Daí,
a1 = 6 e o valor de b é −a
q−1
1
= −3.

3. (valor 2,0)
Prove, por indução finita, que

1 1 1 1 1 n
1+ + + + + . . . n−1 > ,
2 3 4 5 2 2

para todo n natural.

Uma solução:
Seja P (n) a sentença 1 + 12 + 13 + 14 + . . . 2n−1
1
> n2 .
1
P (1) é verdadeira, já que 21−1
= 1 > 12 .
Suponhamos P (n) verdadeira para algum n > 1, ou seja, 1 + 12 + 13 + 14 + . . . 2n−1
1
> n2 , n > 1.
Daí, 1+ 12 + 13 + 14 +· · ·+ 2n−1
1
+ 2n−11 +1 +· · ·+ 21n > n2 + 2n−11 +1 +. . . 21n > n2 + 2 2n = n2 + 12 = n+1
n−1
2
.
Logo, P (n + 1) também é verdadeira. Portanto, pelo Princípio da Indução Finita, P (n) é
verdadeira para todo n natural.

4. (valor 1,5)
Na figura abaixo temos uma espiral formada por infinitos semicírculos cujos centros pertencem
ao eixo das abscissas. Se o raio do primeiro semicírculo (o maior) é igual a 1 e o raio de cada
semicírculo é igual à metade do semicírculo anterior, determine:
a) o comprimento total da espiral. (pontuação parcial 0,75)
b) a abscissa do ponto P assintótico da espiral. (pontuação parcial 0,75)

1 2 x
Uma solução:

a) O comprimento total da espiral é π.1 + π. 12 + π. 14 + · · · = π 1−1 1 = 2π


2

1 1 1
b) A abscissa do ponto P é 2 − 1 + − + ... = 2 −
2 4 1−(−( 12 ))
= 43 .

5. (valor 1,0)
a) Se (an ) é uma progressão geométrica de termos positivos, prove que (bn ) definida por
bn = log an é uma progressão aritmética. (pontuação parcial 0,5)
b) Se (an ) é uma progressão aritmética, prove que (bn ) definida por bn = ean é uma progressão
geométrica. (pontuação parcial 0,5)

Uma solução:
a) Como bn+1 − bn = log an+1 − log an = log an+1an
= log q = constante, sendo q a razão da
progressão geométrica (an ), então (bn ) é uma progressão aritmética.
b) Como bn+1
an+1

bn
= e ean = ean+1 −an = er = constante, sendo r a razão da progressão aritmética
(an ), então (bn ) é uma progressão geométrica.
Sociedade Brasileira de Matemática
Mestrado Profissional em Matemática em Rede Nacional

MA12 – Matemática Discreta


Avaliação - AV 2 - MA 12
29 de junho de 2013

1. (2,0) Penélope quer distribuir 6 presentes entre seus sobrinhos Alfredo, Bruno, Carlos e
Daniel, de modo que cada um receba pelo menos um presente. Todos os presentes devem ser
distribuídos.

a) (0,5) Supondo que todos os presentes sejam iguais, de quantos modos ela pode distribuir
os presentes?

b) (1,5) Resolva novamente o item a), supondo agora que todos os presentes sejam diferentes.

2. (2,0) Sejam R o raio da base e h a altura de um cilindro circular reto.

a) (0,5) Calcule a média aritmética e a média geométrica dos valores Rh, Rh e 2R2 .

b) (1,5) Use a desigualdade das médias para calcular qual é a menor área total possível para
um cilindro circular reto com um volume V dado. Que relação deve existir entre o raio da base
e a altura desse cilindro para que ele tenha essa menor área possível?

3. (2,0) João tem dois dados. O dado A tem três faces vermelhas e três azuis. O dado B
tem duas faces vermelhas e quatro azuis. Ele escolhe um dos dados ao acaso e o lança. Se a
face que sai é azul, ele lança a seguir o dado A; se é vermelha, ele lança o dado B.

a) (0,5) Qual é a probabilidade de que o segundo dado lançado seja o dado B?

b) (0,5) Qual é a probabilidade de que saia uma face vermelha no segundo lançamento?

c) (1,0) Se a face que sai no segundo lançamento é vermelha, qual é a probabilidade de que
o primeiro dado lançado tenha sido o A?
4. (2,0) Em uma reunião há 26 pessoas, com idades variando entre 16 e 65 anos.

a) (1,0) Mostre que há na reunião pelo menos um par de pessoas cujas datas de nascimento
estejam espaçadas por menos de 2 anos.

b) (0,5) Existe um mês do ano em que pelo menos k pessoas dentre as presentes na reunião
fazem aniversário. Qual é o maior valor de k para o qual esta sentença é necessariamente
verdadeira?

c) (0,5) Considere a afirmação: Existe um mês em que pelo menos quatro pessoas do mesmo
sexo dentre as presentes na reunião fazem aniversário. Quantas pessoas a mais, no mínimo,
devem chegar à reunião para que se tenha certeza de que esta afirmativa seja verdadeira?

5. (2,0) No sorteio da Mega-Sena, são sorteados, consecutivamente e sem reposição, 6 nú-


meros de 1 a 60.

a) (1,0) Qual é a probabilidade de que o número 23 seja um dos sorteados?

b) (1,0) Qual é a probabilidade de que o último número sorteado seja o maior dos 6 números
que foram sorteados?
Sociedade Brasileira de Matemática
Mestrado Profissional em Matemática em Rede Nacional

MA12 – Matemática Discreta


Avaliação - GABARITO AV 2 - MA 12
29 de junho de 2013

1. (2,0) Penélope quer distribuir 6 presentes entre seus sobrinhos Alfredo, Bruno, Carlos e
Daniel, de modo que cada um receba pelo menos um presente. Todos os presentes devem ser
distribuídos.
a) (0,5) Supondo que todos os presentes sejam iguais, de quantos modos ela pode distribuir
os presentes?
b) (1,5) Resolva novamente o item a), supondo agora que todos os presentes sejam diferentes.

Uma solução:
a) Uma vez tendo distribuído um presente para cada um dos sobrinhos, sobram 2 presentes,
para distribuir para 4 crianças. O número de modos de fazê-lo é igual ao número de soluções
não negativas de x1 + x2 + x3 + x4 = 2, que é igual a CR4,2 = C5,2 = 10.
É também fácil enumerar todas as possibilidades: há 4 modos de uma das crianças receber
dois presentes adicionais e C4,2 = 6 modos de duas delas receberem um presente adicional cada.
Aqui estão as possibilidades:

ABCD ABCD ABCD ABCD


3111 1311 1131 1113

ABCD ABCD ABCD ABCD ABCD ABCD


2211 2121 2112 1221 1212 1122
b) Primeiro, devemos decidir qual é a quantidade de presentes que cada sobrinho vai receber.
As possibilidades são as seguintes:
• um dos sobrinhos recebe 3 presentes e os demais 1.
O sobrinho a receber os 3 presentes pode ser escolhido de 4 modos. Os presentes dos demais
podem ser escolhidos de 6 × 5 × 4 = 120 modos (os que restarem ficam com o sobrinho que
recebe 3 presentes). O número total de possibilidades é 4 × 120 = 480.
• exatamente dois sobrinhos recebem dois presentes.
Esses sobrinhos podem ser escolhidos de C4,2 = 6 modos. Os presentes dos outros sobrinhos
podem ser escolhidos de 6×5 = 30 modos. Os presentes de um dos sobrinhos a receber 2 presentes
podem ser escolhidos de C4,2 = 6 modos. O número total de possibilidades é 6 × 30 × 6 = 1080.
O número total de modos de distribuir os presentes é 480 + 1080 = 1560.
2. (2,0) Sejam R o raio da base e h a altura de um cilindro circular reto.
a) (0,5) Calcule a média aritmética e a média geométrica dos valores Rh, Rh e 2R2 .
b) (1,5) Use a desigualdade das médias para calcular qual é a menor área total possível para
um cilindro circular reto com um volume V dado. Que relação deve existir entre o raio da base
e a altura desse cilindro para que ele tenha essa menor área possível?

Uma solução:
1
a) A média aritmética é A = (2Rh + 2R2 )/3 e a geométrica é G = (2R4 h2 ) 3 .
b) A área total do cilindro é S = 2πRh + 2πR2 , enquanto que seu volume é V = πR2 h.
Logo

S 2V 2
A= e G3 = 2
3π π
Daí, pela desigualdade das médias, A ≥ G, e portanto

S 2V 2 1 1
≥ ( 2 )3 =⇒ S ≥ 3(2πV 2 ) 3
3π π
1
Assim, a menor área total possível para um volume V fixo é S = 3(2πV 2 ) 3 .
Para que valha a igualdade (e portanto para que o cilindro tenha área total mínima), os
elementos que compõem as médias devem ser todos iguais, logo deve-se ter Rh = 2R2 , ou seja,
h = 2R.

3. (2,0) João tem dois dados. O dado A tem três faces vermelhas e três azuis. O dado B
tem duas faces vermelhas e quatro azuis. Ele escolhe um dos dados ao acaso e o lança. Se a
face que sai é azul, ele lança a seguir o dado A; se é vermelha, ele lança o dado B.
a) (0,5) Qual é a probabilidade de que o segundo dado lançado seja o dado B?
b) (0,5) Qual é a probabilidade de que saia uma face vermelha no segundo lançamento?
c) (1,0) Se a face que sai no segundo lançamento é vermelha, qual é a probabilidade de que
o primeiro dado lançado tenha sido o A?

Uma solução:
a) P (2o dado é B) = P (1o dado é A).P (2o dado é B|1o dado é A)+
+P (1o dado é B).P (2o dado é B|1o dado é B) = 1
2
× 12 + 12 × 1
3
= 5
12
.
É claro que P (1o dado é A) = 7
12
; isto será usado no próximo ítem.

b) P (2a é Vermelha) = P (2o dado é A).P (Vermelha|A) + P (2o dado é B).P (Vermelha|B)
= 12
7
× 12 + 12
5
× 13 = 31
72
.
c) P (1o dado é A|2a é Vermelha) = P(1o dado é A e 2a é Vermelha)/P (2a é Vermelha).
Mas
P (1o dado é A e 2a é Vermelha) = P(1o dado é A e 2o dado é A e 2a é Vermelha)+
+P (1o dado é A e 2◦ dado é B e 2a é Vermelha) = 12 × 12 × 12 + 12 × 12 × 13 = 5
24
.
Logo, P (1o dado é A|2a é Vermelha) =
= P (1o dado é A e 2a é Vermelha)/P (2a é Vermelha) = ( 24
5
)/( 31
72
)= 15
31
.
Estes resultados podem ser obtidos também usando-se diretamente o diagrama:

Segundo
lançamento
Primeiro
lançamento

1/2 Azul

Face azul → A é o dado


selecionado para o segundo Vermelho
1/2 1/2
lançamento

2/3 Azul
Escolha do
Face vermelha → B é o
dado A
dado selecionado para o
1/2 Vermelho
1/2 segundo lançamento 1/3

1/2 Azul
2/3 Face azul → A é o dado
1/2 selecionado para o segundo
Escolha do Vermelho
lançamento 1/2
dado B

2/3 Azul
1/3 Face vermelha → B é o
dado selecionado para o
segundo lançamento Vermelho
1/3

4. (2,0) Em uma reunião há 26 pessoas, com idades variando entre 16 e 65 anos.


a) (1,0) Mostre que há na reunião pelo menos um par de pessoas cujas datas de nascimento
estejam espaçadas por menos de 2 anos.
b) (0,5) Existe um mês do ano em que pelo menos k pessoas dentre as presentes na reunião
fazem aniversário. Qual é o maior valor de k para o qual esta sentença é necessariamente
verdadeira?
c) (0,5) Considere a afirmação: Existe um mês em que pelo menos quatro pessoas do mesmo
sexo dentre as presentes na reunião fazem aniversário. Quantas pessoas a mais, no mínimo,
devem chegar à reunião para que se tenha certeza de que esta afirmativa seja verdadeira?

Uma solução:

a) Sejam x1 ≤ x2 ... ≤ x26 as idades em ordem não decrescente. Por hipótese temos que
x1 ≥ 16 e x26 ≤ 65. Como

(x2 − x1 ) + (x3 − x2 ) + · · · + (x26 − x25 ) = x26 − x1 ≤ 65 − 16 = 49,


então a média aritmética

(x2 − x1 ) + (x3 − x2 ) + · · · + (x26 − x25 )


25
é menor do que 49/25 e pelo menos uma das diferenças (xi+1 − xi ) é menor ou igual do que
49/25 e, portanto, menor do que 2.
Obs.: A rigor, x26 − x1 não é, necessariamente, igual a 49; isto ocorre quando discretizamos o
tempo em anos, mas, de fato, a pessoa com idade x1 pode ter acabado de completar 16 anos e
a pessoa com idade x26 pode estar prestes a fazer 66. Em qualquer caso certamente x26 − x1 é
estritamente menor que 50, o que faz com que a conclusão seja válida.
b) A resposta é k = 3. Como há 26 pessoas e 12 possibilidades para o mês de aniversário,
há um mês em que nasceram no mínimo 26/12 = 2, 1... pessoas. Portanto, há um mês em que
nasceram pelo menos três pessoas. Não se pode garantir que haja 4 pessoas em um mesmo mês
(basta distribuir as pessoas colocando duas em cada um de 10 meses e três em cada um dos
outros 2 meses).
c) Há 24 combinações (gavetas) possíveis de sexo e mês de aniversário. Logo, para que
se possa garantir que haja 4 pessoas em uma mesma gaveta, é preciso que haja pelo menos
24 × 3 + 1 = 73 objetos (pessoas). Portanto, é preciso que cheguem mais 73 − 26 = 47 pessoas.

5. (2,0) No sorteio da Mega-Sena, são sorteados, consecutivamente e sem reposição, 6 nú-


meros de 1 a 60.
a) (1,0) Qual é a probabilidade de que o número 23 seja um dos sorteados?
b) (1,0) Qual é a probabilidade de que o último número sorteado seja o maior dos 6 números
que foram sorteados?

Uma solução:
a) Primeira solução: A probabilidade que 23 saia em cada um dos números sorteados é 1/60.
Logo, a probabilidade de que saia em um deles é 6 × 1/60 = 1/10.
Segunda solução: O número de casos possíveis para o sorteio é 60 × 59 × 58 × 57 × 56 × 55.
O número de casos favoráveis é

6 × 59 × 58 × 57 × 56 × 55
(6 é o número de modos de se escolher a posição do 23 e o produto dos demais fatores dá o
número de modos de se escolher os outros 5 números)
A probabilidade de aparecer o 23 é, portanto,

(6 × 59 × 58 × 57 × 56 × 55)/(60 × 59 × 58 × 57 × 56 × 55) = 1/10

b) Como os números são sorteados ao acaso, todas as ordenações dos números sorteados são
igualmente prováveis. Logo, é igualmente provável que o maior número apareça em cada posição.
Assim, a probabilidade de que ele apareça na última posição é 1/6.
Sociedade Brasileira de Matemática
Mestrado Profissional em Matemática em Rede Nacional

MA12 – Matemática Discreta


Avaliação - AV 3 - MA 12
13 de julho de 2013

1. (2,0) Seja (an ) uma progressão aritmética e seja (bn ) a sequência definida por

bn = an + an+1 , ∀n ≥ 1

a) (0,5) Mostre que (bn ) também é uma progressão aritmética.


b) (1,5) Suponha que a soma dos n primeiros termos da sequência (an ) seja igual a 2n2 + 5n,
para todo natural n ≥ 1. Obtenha uma expressão para a soma dos n primeiros termos de (bn ).

2. (2,0) Seja (an ) uma sequência tal que a1 = 2, a2 = 5 e an+2 = an + an+1 , para todo
natural n ≥ 1. Mostre, por indução finita, que

a2 + a4 + a6 + ... + a2n = a2n+1 − 2,


para todo n ≥ 1.

3. (2,0) Considere a bandeira da figura abaixo, formada por seis regiões. Para colori-la, há
lápis de cor de quatro cores diferentes.
a) (0,5) De quantos modos ela pode ser colorida de modo que regiões adjacentes tenham
cores diferentes?
b) (1,5) Resolva o item a), supondo agora que todas as quatro cores sejam utilizadas para
pintar cada bandeira.

4. (2,0) João precisa comprar uma peça para seu carro, com o qual ele espera ficar por mais 3
anos. Ele pode comprar, por R$ 1400, 00, uma peça original, que vai durar todo este período, ou,
por R$ 500, 00, uma peça alternativa, que dura apenas 1 ano. Suponha que o valor do dinheiro
seja de 10% ao ano.
a) (1,0) Mostre que, apesar do desembolso total com a peça alternativa ser maior, ela é a
mais vantajosa para João.
b) (1,0) João acha que pode conseguir um desconto na peça original. A partir de que valor
vale a pena ele optar por ela?

5. (2,0) As faces de um dado honesto são numeradas de 1 a 3 (cada número aparece duas
vezes). Seja pn a probabilidade de que a soma das faces obtidas em n lançamentos seja par.
2 1
a) (1,0) Explique porque a sequência pn satisfaz a recorrência pn+1 = 3
− 3
· pn . Qual é o
valor de p1 ?
b) (1,0) Resolva a equação de recorrência em a) para obter uma expressão para pn .
[Sugestão: determine uma constante k tal que pn = k seja uma solução da recorrência e faça a
substituição pn = yn + k, para obter uma recorrência homogênea ou você também pode usar o método
geral visto no curso para resolver recorrências lineares não homogêneas.]
Sociedade Brasileira de Matemática
Mestrado Profissional em Matemática em Rede Nacional

MA12 – Matemática Discreta


Avaliação - GABARITO AV 3 - MA 12
13 de julho de 2013

1. (2,0) Seja (an ) uma progressão aritmética e seja (bn ) a sequência definida por

bn = an + an+1 , ∀n ≥ 1

a) (0,5) Mostre que (bn ) também é uma progressão aritmética.


b) (1,5) Suponha que a soma dos n primeiros termos da sequência (an ) seja igual a 2n2 + 5n,
para todo natural n ≥ 1. Obtenha uma expressão para a soma dos n primeiros termos de (bn ).

Uma solução:
a) bn+1 − bn = (an+1 + an+2 ) − (an + an+1 ) = an+2 − an = 2r, sendo que r é a razão de
(an ). Logo, (bn ) é uma P.A. de razão 2r.
b) Primeira solução:
b1 + b2 + ... + bn = (a1 + a2 ) + (a2 + a3 ) + ... + (an + an+1 ) =
= (a1 + a2 + ... + an ) + (a2 + a3 + ... + an+1 ) = Sn + Sn+1 − S1 =
= 2n2 + 5n + 2(n + 1)2 + 5(n + 1) − 7 = 4n2 + 14n
Segunda solução:
Podemos determinar primeiramente o primeiro termo e a razão de (an ).
A soma dos n primeiros termos de an é

n n n2 r
Sn = (a1 + an ). = (a1 + a1 + (n − 1)r). = r. + (a1 − ).n = 2n2 + 5n
2 2 2 2
r r
Logo, 2 = 2 e (a1 − 2 ) = 5. Daí, r = 4 e a1 = 7. Portanto, (bn ) é uma P.A. cujo primeiro
termo é b1 = a1 + a2 = 7 + 11 = 18 e cuja razão é 2 × 4 = 8.
(alternativamente, podemos observar que S1 = 7 e S2 = 18; daí, a1 = 7 e a2 = 11 e, portanto,
r = 4).
Portanto, a soma dos seus n primeiros termos de (bn ) é

n n
(b1 + bn ). = (18 + 18 + (n − 1).8). = 4n2 + 14n
2 2
.

2. (2,0) Seja (an ) uma sequência tal que a1 = 2, a2 = 5 e an+2 = an + an+1 , para todo
natural n ≥ 1. Mostre, por indução finita, que

a2 + a4 + a6 + ... + a2n = a2n+1 − 2,


para todo n ≥ 1.

Uma solução:
Seja p(n) a afirmação: a2 + a4 + a6 + ... + a2n = a2n+1 − 2.
i) Para n = 1, temos a2 = 5 e a3 = 2 + 5 = 7. Logo, de fato temos a2 = a3 − 2, o que
verifica a validade de p(1 ).
ii) Suponhamos que p(n) seja válida, ou seja, que a2 + a4 + a6 + ... + a2n = a2n+1 − 2. Daí:
a2 + a4 + a6 + ... + a2n + a2n+2 = a2n+1 + a2n+2 − 2 = a2n+3 − 2, o que mostra que p(n + 1 )
é válida.
Logo, pelo princípio da indução finita, p(n) é válida para todo n natural.

3. (2,0) Considere a bandeira da figura abaixo, formada por seis regiões. Para colori-la, há
lápis de cor de quatro cores diferentes.

a) ((0,5) De quantos modos ela pode ser colorida de modo que regiões adjacentes tenham
cores diferentes?
b) (1,5) Resolva o item a), supondo agora que todas as quatro cores sejam utilizadas para
pintar cada bandeira.

Uma solução:
B
A
C
D
F
E

a) Colorindo as regiões na ordem indicada, o número de possibilidades é 4×3×2×3×3×2 =


432
b) Nas condições exigidas, não é possível pintar a bandeira com apenas uma cor ou com
duas cores apenas, mas é possível pintá-la com três cores (sem que regiões adjacentes tenham a
mesma cor).
Colorindo as regiões na ordem indicada com apenas 3 cores, o número de possibilidades é
3×2×1×2×2×1 = 24. Mas as 3 cores a serem usadas podem ser escolhidas de 4 modos. Logo,
o número de modos de colorir a bandeira usando todas as quatro cores é 432 − 4 × 24 = 336.

4. (2,0) João precisa comprar uma peça para seu carro, com o qual ele espera ficar por mais 3
anos. Ele pode comprar, por R$ 1400, 00, uma peça original, que vai durar todo este período, ou,
por R$ 500, 00, uma peça alternativa, que dura apenas 1 ano. Suponha que o valor do dinheiro
seja de 10% ao ano.
a) (1,0) Mostre que, apesar do desembolso total com a peça alternativa ser maior, ela é a
mais vantajosa para João.
b) (1,0) João acha que pode conseguir um desconto na peça original. A partir de que valor
vale a pena ele optar por ela?

Uma solução:

a) Devemos comparar os seguintes esquemas de pagamento:


R$ 1400

0 1 2 3 anos

R$ 500 R$ 500 R$ 500

1 2 3 anos
0

Levando ambos para a data 2:


Peça original: 1400 × (1, 1)2 = 1400 × 1, 21 = 1694
Peça alternativa: 500 × (1, 1)2 + 500 × 1, 1 + 500 = 605 + 550 + 500 = 1655.
Logo, o custo com a peça alternativa é inferior.
b) Para que os esquemas sejam equivalentes, o preço p da peça original deve ser tal que
p × 1, 21 = 1655, ou seja, p = 1655
1,21
≈ 1367, 77 reais.

5. (2,0) As faces de um dado honesto são numeradas de 1 a 3 (cada número aparece duas
vezes). Seja pn a probabilidade de que a soma das faces obtidas em n lançamentos seja par.
2 1
a) (1,0) Explique porque a sequência pn satisfaz a recorrência pn+1 = 3
− 3
· pn . Qual é o
valor de p1 ?
b) (1,0) Resolva a equação de recorrência em a) para obter uma expressão para pn .
[Sugestão: determine uma constante k tal que pn = k seja uma solução da recorrência e faça a
substituição pn = yn + k, para obter uma recorrência homogênea ou você também pode usar o método
geral visto no curso para resolver recorrências lineares não homogêneas.]

Uma solução:

a) pn+1 = P (soma par em n + 1 lançamentos) =


= P (soma par em n lançamentos).P (sai par no lançamento n + 1) +
P ( soma ímpar em n lançamentos).P ( sai ímpar no lançamento n + 1)
Logo
1 2 2 1
pn+1 = pn · + (1 − pn ) · = − · pn
3 3 3 3
O valor de p1 é 1/3.

2
b) Seguindo a sugestão: k = 3 − 13 · k, o que resulta em k = 12 . Fazendo a substituição pn = yn + 21 ,
obtemos

1 2 1 1
yn+1 + = − (yn + ),
2 3 3 2
ou seja yn+1 = (− 31 )yn .
A solução geral desta recorrência homogênea é yn = C(− 13 )n. Logo, pn = 12 + C(− 31 )n . Vamos
determinar C. Substituindo n = 1, obtemos p1 = 13 = 12 + C(− 13 ), que fornece C = 21 . Portanto, a
solução da recorrência é

1 1 1
pn = + · (− )n
2 2 3
.
Vejamos agora como solucionar a mesma questão usando o método geral para resolver recorrências
não homogêneas de primeira ordem.
Uma solução da equação homogênea pn+1 = (− 31 )pn é an = (− 13 )n . Fazendo a substituição
pn = an yn , temos

1 2 1 1
(− )n+1 yn+1 = − (− )n yn ,
3 3 3 3
ou seja, yn+1 − yn = 32 · (−3)n+1 .
Escrevendo esta igualdade para n variando de 1 a n − 1 e somando:

2   (P G) 3 
yn − y1 = ( ) (−3)2 + ... + (−3)n ) = · 1 − (−3)n−1

3 2
.
3
Daí yn = y1 + 2 · [1 − (−3)n−1 ]. Mas p1 = 13 . Logo − 31 · y1 = 13 , ou seja y1 = −1. Logo,

3 3 1 1
yn = −1 + − (−3)n−1 = + (−3)n
2 2 2 2
Daí, finalmente:
1 1 1 1
pn = (− )n yn = · (− )n +
3 2 3 2
RELATÓRIO
DE
DESEMPENHO

MA11
E
MA12
RELATORIO MA11
Questão 1 Questão 2 Questão 3 Questão 4 Questão 5
1,0 1,0 1,0 1,0 1,0 1,0 0,5 0,5 1,0 1,0 1,0 NOTA
9,43 9,40 7,77 9,28 5,20 3,63 6,08 3,02 2,05 4,59 2,45 5,83
9,42 8,52 4,41 3,30 3,52

25
%
0 2 2 0,45
1 2 0 0 20
2 6 4 0,9
3 21 15 3,36
15
4 75 54 12,11
5 177 102 22,87
6 260 83 18,61 Coluna E
10
7 335 75 16,82
8 386 51 11,43
5
9 424 38 8,52
10 446 22 4,93
17 POLOS 0
Linha 31 Linha 35 Linha 39
Linha 29 Linha 33 Linha 37

Página 1
MA11 — Números, conjuntos e funções elementares – Prova 1 – 2011

Questão 1.
Um pequeno barco a vela, com 7 tripulantes, deve atravessar o oceano em 42 dias. Seu suprimento de água potável
permite a cada pessoa dispor de 3,5 litros de água por dia (e é o que os tripulantes fazem). Após 12 dias de viagem,
o barco encontra 3 náufragos numa jangada e os acolhe. Pergunta-se:

(1.0) (a) Quantos litros de água por dia caberão agora a cada pessoa se a viagem prosseguir como antes?

(1.0) (b) Se os 10 ocupantes de agora continuarem consumindo 3,5 litros de água cada um, em quantos dias, no máximo,
será necessário encontrar uma ilha onde haja água?

Questão 2.

(1.0) (a) Quais são os valores de y para os quais existe uma função quadrática f : R → R tal que f (1) = 3, f (2) = 5 e
f (3) = y?

(1.0) (b) Tome y = 9 e determine a função quadrática correspondente. Justifique seus argumentos.

Questão 3.

(1.0) (a) Seja f : A → B uma função. Dê as definições de f (X) e f −1 (Y ), para X ⊂ A e Y ⊂ B. Se f : R → R é dada
por f (x) = 2x2 + 3x + 4, determine os conjuntos f (R) e f −1 (3).

(1.0) (b) Seja f : A → B uma função. Prove que f (X ∪ Y ) = f (X) ∪ f (Y ), quaisquer que sejam X, Y ⊂ A. Dê um
exemplo em que f (X ∩ Y ) 6= f (X) ∩ f (Y ).

Questão 4.

(0.5) (a) Se r 6= 0 é um número racional, prove que r 2 é irracional.

(0.5) (b) Dado qualquer número real ǫ > 0, prove que existe um número irracional α tal que 0 < α < ǫ.

(1.0) (c) Mostre que todo intervalo [a, b], com a < b, contém algum número irracional.

Questão 5.
Sejam m e n números naturais primos entre si.

(1.0) (a) Mostre que m


n é equivalente a uma fração decimal (isto é, com denominador potência de 10) se, e somente se,
n não tem fatores primos diferentes de 2 ou 5.

(1.0) (b) Mostre que se n tem outros fatores primos além de 2 ou 5 então a expansão decimal é infinita e, a partir de
um certo ponto, periódica.
RELATORIO MA12
Questão 1 Questão 2 Questão 3 Questão 4 Questão 5 NOTA
0,5 0,5 1 1 1 0,4 0,8 0,8 1 1 0,5 1 0,5
9,78 9,71 6,80 9,61 8,15 8,80 5,59 6,88 9,61 6,90 7,29 4,38 5,17 7,49
8,27 8,88 6,75 8,26 5,31

20
%
18
0 0 0 0
16
1 0 0 0
2 3 3 1,1 14
3 10 7 2,57 12
4 29 19 6,99 10
5 56 27 9,93 Coluna E
8
6 94 38 13,97
7 141 47 17,28 6
8 184 43 15,81 4
9 223 39 14,34 2
10 272 49 18,01
0
11 POLOS
Linha 26 Linha 30 Linha 34
Linha 24 Linha 28 Linha 32

Página 1
MA12 – Matemática Discreta – Prova 1 – 2011

Questão 1.
Considere a sequência (an )n≥1 definida como indicado abaixo:

a1 = 1
a2 = 2+3
a3 = 4+5+6
a4 = 7 + 8 + 9 + 10
...

(0.5) (a) O termo a10 é a soma de 10 inteiros consecutivos. Qual é o menor e o qual é o maior desses inteiros?

(0.5) (b) Calcule a10 .

(1.0) (c) Forneça uma expressão geral para o termo an .

Questão 2.
Um comerciante, para quem o dinheiro vale 5% ao mês, oferece determinado produto por 3 prestações mensais
iguais a R$ 100,00, a primeira paga no ato da compra.

(1.0) (a) Que valor o comerciante deve cobrar por esse produto, no caso de pagamento à vista?

(1.0) (b) Se um consumidor desejar pagar o produto em três prestações mensais iguais, mas sendo a primeira paga um
mês após a compra, qual deve ser o valor das parcelas?

Utilize, se desejar, os seguintes valores para as potências de 1, 05: 1, 052 = 1, 1025; 1, 05−1 = 0, 9524; 1, 05−2 = 0, 9070.

Questão 3.
Considere o conjunto dos números escritos apenas com os algarismos 1, 2 e 3, em que o algarismo 1 aparece uma
quantidade par de vezes (por exemplo, 2322 e 12123). Seja an a quantidade desses números contendo exatamente n
algarismos.

(0.4) (a) Liste todos esses números para n = 1 e n = 2, indicando os valores de a1 e a2 .

(0.8) (b) Explique por que an satisfaz a equação de recorrência an+1 = (3n − an ) + 2an , para n ≥ 1 (note que 3n é o
número total de números com n algarismos iguais a 1, 2 ou 3).

(0.8) (c) Resolva a equação de recorrência em (b).


MA12 – Matemática Discreta – Prova 1 – 2011

Questão 4.

(1.0) (a) Mostre, por indução finita, que

(2n − 1)3n + 1
1 · 30 + 2 · 31 + 3 · 32 + . . . + n · 3n−1 = .
4

(1.0) (b) Seja (an )n≥1 progressão geométrica com termo inicial a1 positivo e razão r > 1, e Sn a soma dos n primeiros
termos da progressão. Prove, por indução finita, que Sn ≤ r−1 an ,
r
para qualquer n ≥ 1.

Questão 5.
Seja (xn )n≥0 sequência definida pela relação de recorrência xn+1 = 2xn + 1, com termo inicial x0 ∈ R.

(0.5) (a) Encontre x0 tal que a sequência seja constante e igual a um número real a.

(1.0) (b) Resolva a recorrência com a substituição xn = yn + a, em que a é valor encontrado em (a).

(0.5) (c) Para que valores de x0 a sequência é crescente? Justifique.


OUTROS
MATERIAIS
MA12
MATEMÁTICA
DISCRETA
UNIVERSIDADE FEDERAL DE MATO GROSSO

INSTITUTO DE CIÊNCIAS EXATAS E DA TERRA

MESTRADO PROFISSIONAL EM MATEMÁTICA – PROFMAT

MATEMÁTICA DISCRETA

CUIABÁ – 04/2012

1) Prove que a função N  N  N definida por f(m,n) = 2 .3  1 é injetiva.


m n

Solução:

Devemos mostrar que dados (m1 , n1 ) e (m2 , n2 ) , tais que: f (m1 , n1 )  f (m2 , n2 ) , então
(m1 , n1 )  (m2 , n2 )

De fato!

Sejam (m1 , n1 ) e (m2 , n2 ) pertencente a N x N, tais que f (m1 , n1 )  f (m2 , n2 ) . Então


temos que:

2m1 .3n1  1  2m2 .3n2  1 , que nos leva a 2m1 .3n1  2m2 .3n2 .

Como temos potências com expoentes naturais, então podemos aplicar a divisão de
um mesmo termo em ambos os lados. Logo:

2m1 m2  3n2 n1

Esta igualdade vem das propriedades da potenciação. Como 2 e 3 são primos entre
si, temos que esta igualdade só será satisfeita quando o expoente for igual a zero,
isto é:

m1  m2  0  n2  n1 , mas isto só acontece se (m1 , n1 )  (m2 , n2 ) . c.q.d

2) Quando dividimos a soma 1! + 2! + 3! + 4! + ... + 50! Por 15, qual é o resto?

Solução:

O resto é 3.
Com efeito!

Observe que a partir de 5! temos a seguinte situação: 5! = 5x4x3x2x1 = 15q, para


algum q natural.

Logo podemos de maneira análoga arrumar 6!, ..., 50! Como múltiplos de 15. Logo
temos que:

1! + 2! + 3! + 4! + ... + 50! = 1 + 2 + 6 + 24 + 15j, para algum j natural.

1! + 2! + 3! + 4! + ... + 50! = 3 + 30 + 15j = 3 + 15w, para algum w natural.

Portanto a divisão desta soma por 15 é igual a 3.

3) Os números 210, 301 e 352 escrito na base b, estão em PA. Determine b.

De maneira geral, um número natural N pode ser representado em uma base b


através do polinômio:

N  a0b0  a1b1  ...  anbn , com n natural e 0  ai  b  1 .

Sabendo disso temos que a representação destes números na base b é dada por:

210  2b2  b , 301  3b2  1 e 352  3b2  5b  2 . Como estes números estão em PA

temos que: 301 – 210 = 352 – 301, logo: 3b  1  (2b  b)  3b  5b  2  (3b  1) .


2 2 2 2

Resultando em: b²- b + 1 = 5b + 1, implicando em: b² - 6b = 0 = b(b - 6). Desta


forma b = 0 ou b = 6, logo temos que b = 6.

Transformando os números para a base decimal temos que: 210 = 78 na base dez;
301 = 109 na base 10 e 352 = 140 na base 10. Logo de fato a base é 6, pois 109 – 78
= 140 – 109 = 31. Logo r = 31 e a PA = {...,78,109,140,...}.

4) Uma quantidade de 500 reais foi investida em uma conta remunerada a


uma taxa de juros compostos anual de 10%:
Dados: i = 0,1 ao ano, Capital = R$ 500,00.
Pelo Teorema 2 da página 3 da unidade 9 temos que o montante daqui a n anos é

dado por: Cn  500(1.1) .


n

a) Escreva a definição recursiva da quantia P(n) na conta no início do (n)-


ésimo ano.

Chamando de Cn  P(n) , temos que a definição recursiva da quantia na conta no

início do (n)-ésimo ano é: P(n)  500(1.1) .


n

b) Depois de quantos anos a quantia investida excederá 700 reais.


Fazendo a iteração na definição da letra a, geramos a seguinte seqüência:
P(n)  {550,605,665.5,732.05,805.255,...} , logo observamos que depois de

quatro anos a quantia chegará a R$ 732,05 excedendo o valor de R$ 700,00.

5) Escreva a função recursiva de cada uma das seqüências:


a) a, b, a + b, a + 2b, 2a + 3b, .........
Observe que:
x1  a
x2  b
x3  a  b  x2  x1
x4  a  2b  x3  x2
x5  2a  3b  x4  x3

Logo concluímos que a função recursiva é:


xn2  xn1  xn , com x1  a e x2  b .

b) p, q, p – q, p + q, p – 2q, p + 2q, p – 3q, ...........


Observe que a partir do terceiro termo temos expressões parecidas, alterando
apenas o sinal. Desta forma podemos dividir em duas subseqüências: para n
par e para n ímpar. Do terceiro termo em diante observamos a seguinte
relação entre os termos e o q da expressão: (Termo * coeficiente de q): 3 * 1,
5 * 2, 7 * 3,..., n * ((n – 1): 2) e 4 * 1, 6 * 2, 8 * 3, ...., n * (n – 2): 2). Logo
temos a seguinte situação:
x1  p x2  q
 3 1   4 2 
x3  p  q  p   q x4  p  q  p   q
 2   2 
 5 1   6 2 
x5  p  2q  p   q x6  p  2q  p   q
 2   2 
 5 1   8 2 
x7  p  3q  p   q x8  p  3q  p   q
 2   2 

 n 1   n 2 
xn  ímpar  p   q xn  par  p   q
 2   2 
Logo concluímos que a função recursiva é dada por:
  n 1 
n  ímpar : xn  p   2  q
  
xn  
n  par : x  p   n  2  q
  
, com x1  p e x2  q .
n
 2 

6) Calcular:
n

k
k 1
4

a)
Solução:
A idéia para solucionar este exercício é dada na Unidade 5 – página 9 – Somas
polinomiais.
Devemos partir de:
(k  1)5  k 5  5k 4  10k3  10k2  5k  1
Aplicando o somatório em ambos os lados e juntamente as propriedades, temos que:
n n n n n n

[(k  1)5  k 5 ]  5 k 4  10 k3  10 k2  5 k  1


k 1 k 1 k 1 k 1 k 1 k 1

Da teoria e dos exercícios temos que:


n
n(n  1)(2n  1) n
n(n  1) n n

k
k 1
2

6
k 
k 1 2 ,
1  n
k 1
k
k 1
3

, . Não temos a expressão , todavia

usando a mesma idéia inicial, isto é: (k  1)  k , chegamos à seguinte expressão:


4 4

2
n
n4  2n3  n2  n(n  1) 

k 1
k 
3

4

 2 

Logo temos que:


n n n n n n

[(k  1)
k 1
5
 k 5 ]  5 k 4  10 k 3  10 k 2  5 k  1
k 1 k 1 k 1 k 1 k 1
  n(n  1) 2 
n
 n(n  1)(2n  1)   n(n  1) 
(n  1)  1  5 k  10  
5 4
   10    5 n

k 1  2    6   2 

Desenvolvendo e simplificando esta igualdade chegamos em:


n
n 4
k
k 1
4

30
(6n  15n3  10n2  1)

n
1
k
k 2
2
1
b)

Solução:
n
1 n
 1 
k
k 2
2
 
 1 k 2  (k  1).(k  1)  . Vamos tentar encontrar A e B tais que:

Observe que:
n
 1  n
 A B 
  (k  1).(k  1)     k  1  k  1 
k 2 k 2

Resolvendo um sistema de equações encontramos A = -1/2 e B = 1/2. Logo temos que:


n
1 n
 1  n  A B  n  1/2 1/2  n  1 1 
k
k 2
 
2              
 1 k 2  (k  1).(k  1)  k 2  k  1 k  1  k 2  k  1 k  1  k 2  2(k  1) 2(k  1) 
Desenvolvendo este somatório temos que:
n
 1 1  1 1 1 1 1 1 1 1 1 1 1 1
   2(k  1)  2(k  1)    6  2  8  4  10  6  ...  2(n  1)  2(n  3)  2n  2(n  2)  2(n  1)  2(n  1)
k 2

Desta forma observamos que sobrarão os dois primeiros termos positivos e os dois últimos
termos negativos, isto é:

n
1 1 1 1 1 3  2(n  1)  2n  3n2  n  2

k 2 k  1
2
       
2 4 2n 2(n  1) 4  4n(n  1)  4n(n  1)
UNIVERSIDADE FEDERAL DE MATO GROSSO

INSTITUTO DE CIÊNCIAS EXATAS E DA TERRA

MESTRADO PROFISSIONAL EM MATEMÁTICA – PROFMAT

MATEMÁTICA DISCRETA

1) Prove que:
1 1 1
(a)1    ...  2 n
2 3 n , para todo n maior ou igual a um.

Solução:

Para n = 1 temos verdadeiro, pois 1 < 2. OK.

Logo, suponha válido para n = k, isto é:

1 1 1
1   ...  2 k
2 3 k , para todo K maior ou igual a um.

Devemos mostrar verdadeiro para K + 1, isto é:

1 1 1 1
1   ...    2 k 1
2 3 k k 1 , para todo K maior ou igual a um.

De fato!

Veja que:

1 1 1  1   1 
1   ...   2 k  
2 3 k  k 1   k 1 

1
4 k  4K 
Agora como k é maior ou igual a 1, temos que k  1 . Logo temos que:

1 1
4 k  4K   4k  4k  4  4  
k 1 k 1
 k 1 
2
1
   1 
2
 4k  4(k  1)  2.2     2 k  2 k 1   
 k 1  k 1  k 1 
1 1
 2 k 2 k 1  2 k   2 k  1.
k 1 k 1
1 1 1 1 1
1   ...   2 k   2 k  1.
Portanto 2 3 k k 1 k 1 , como queríamos
demonstrar.

OBS: estas passagens são garantidas, pois k é maior ou igual a 1.

n3 2 2
(b)12  22  ...  (n  1)2   1  2  ...  n2
3 , para todo n maior ou igual a dois.

Solução:

n.  n  1 .  2n  1
12  22  ...  n2 
Afirmação 1: 6

De fato!

Para n = 1 OK, pois 1 = ((1.2.3)/6)

k.  k  1 . 2k  1
12  22  ...  k 2 
Suponha válido para n = k, isto é: 6 , devemos mostrar

12  22  ...  k 2   k  1 
2  k  1.  k  2. 2k  3
válido para k + 1, isto é: 6 .

Veja que:
k.  k  1 . 2k  1
12  22  ...  k 2   k  1    k  1
2 2

6
k.  k  1 . 2k  1  k  1  k  1  2k  3 . k  2 
  k  1  k. 2k  1  6  k  1     
2

6 6 6

12  22  ...  n2   n  1 
 n  1  2n  3 . n  2    n  1 .  n  2 . 2n  3
   
2

E, portanto: 6 6 .

Agora veja que (pela afirmação 1):

12  22  ...   n  1 
2  n  1 n . 2n  1  n.n.2n  2n3  n3
6 6 6 3 , pois n – 1 < n e 2n – 1 <

2n, para todo n maior ou igual a 2.

De mesmo modo, temos que:


n.  n  1 . 2n  1 n.n.2n 2n3 n3
12  22  ...   n  
2
  
6 6 6 3 , pois n + 1 > n e 2n + 1 > 2n,

para todo n maior ou igual a 1. Logo concluímos que:

n3 2 2
12  22  ...   n  1   1  2  ...   n 
2 2

3 , para todo n maior ou igual a 2.

1 1 1
(c)1    ...  n  2
2 4 2 , para todo n maior ou igual a um.

Solução:

Para n = 1 temos verdadeiro, pois 1 < 2. OK.

Logo, suponha válido para n = k, isto é:

1 1 1
1    ...  k  2
2 4 2 , para todo k maior ou igual a um.

Devemos mostrar verdadeiro para k + 1, isto é:

1 1 1
1    ...  k1  2
2 4 2 , para todo k maior ou igual a um.

De fato!

Veja que:

1 1 1
1    ...  k
2 4 2 , consiste na soma dos k primeiros termos de uma PG de razão ½.

n 1
1
an   
Seja 2 , então temos a seguinte sequencia = {1,1/2,1/4,...,1/k}.

  1 k 
 1    
2    2k  1 2 k
Sk  a1  a2  ...  ak  a1 
1
 1  k  2   2
1
1  2  2k  1 2k  1
2

Agora precisamos mostrar verdadeiro para Sk 1 . Veja que:


k
1 1 2k 1  1 2k 1
Sk 1  Sk  ak 1  2     2  k   k 2
2 2 2k 2

1 1 1 1
Sk 1  a1  a2  ...  ak  ak 1  1    ...  k  k 1  2
Portanto: 2 4 2 2 .

2) Suponha que 51 números são escolhidos dos números de 1 até 100. Mostre
que existem dois pares destes números que não tem divisor primo comum.

Solução:

Inicialmente observe que dois números consecutivos são primos entre si, pois,
necessariamente um será par e o outro ímpar. De 1 até 100 temos 100 números,
pois, 50 são pares: {2,4,6,...,100} e 50 são ímpares: {1,3,5,...,99}. Na pior das
hipóteses teremos 50 números pares e um ímpar ou 50 números ímpares e um par.

Caso 1) 50 números pares e um ímpar.

Se este impar for 1, temos (1,2) e (1,3), dois pares de números que não tem divisor
primo comum.

Se este impar for 99, temos que (98,99) e (99,100), dois pares de números que não
tem divisor primo comum.

De maneira análoga para qualquer outro impar mostramos o par de números que não
tem divisor primo comum.

OBS: Para todos os outros casos a análise é a mesma.

Caso 2) 50 números ímpares e um par

Se este par for 2, temos que (2,1) e (2,3) dois pares de números que não tem divisor
primo comum.

Se este par for 100, temos que (100,99) e (100,97) dois pares de números que não
tem divisor primo comum.

OBS: Para todos os outros casos a análise é a mesma.


3) Use a Boa-ordenação para provar que se X é um conjunto tal que n(X) = n
n
tem 2 subconjuntos.

Solução:

Uma análise inicial consiste na demonstração empírica, isto é, vejamos alguns


casos:

Se X = {1}. Logo n(X) = 1 e P(X )  {,1} , ou seja, P(X) tem 2 subconjuntos.

Se X = {1,2}. Logo n(X) = 2 e P(X)  {,1,2,(1,2)} , ou seja, P(X) tem 4


subconjuntos.

Se X = {1,2,3}. Logo n(X) = 3 e P(X)  {,1,2,3,(1,2),(1,3),(2,3),(1,2,3)} , ou seja, P(X)


tem 8 subconjuntos.

Intuitivamente observamos que se X possui n elementos, P(X) possui


2n subconjuntos. Daí, se acrescentamos um novo elemento k em X, este elemento é

unido a cada subconjunto de P(X), logo ele gera a mesma quantidade de elementos
que já existem em P(X), isto é, 2  2  2.2  2 .
n n n n1

Uma demonstração matemática é dada pela indução matemática. Veja:

Para n = 1 , OK, pois fazendo X={a}, temos como subconjuntos de X o  e o {a}.

Agora, supondo que a afirmação seja verdadeira para um conjunto X com n


elementos, daí para um conjunto Y com (n + 1) elementos. Fixando {a} pertencente
a Y e seja X = Y – {a}. Assim, há dois tipos de Subconjuntos de Y, aqueles que não
n
contêm a, ou seja, os conjuntos das partes de X, que somam 2 subconjuntos e
n
aqueles que contêm a, que também somam 2 subconjuntos, daí o total de
subconjuntos de Y é dado por 2  2  2.2  2 . Portanto, a afirmação é válida para
n n n n1

todo n natural.

4) Mostre que entre quaisquer quatro números podemos encontrar 2 números


de modo que a diferença entre eles é divisível por 3.
Solução:
A resolução deste exercício é feita mediante o Princípio das casas de Pombos, pois dado
um número inteiro qualquer, a divisão dele por 3 pode ser representada de três
maneiras: múltiplo de 3, resto 1 ou resto 2. Desta forma tendo 4 números (pombos) e
tendo apenas 3 casas (múltiplo de 3, resto 1 ou resto 2), necessariamente dois deles
ficarão na mesma casa, ou seja, a diferença entre eles seja um múltiplo de 3 e
automaticamente será divisível por 3.

Uma demonstração algébrica pode ser dada da seguinte maneira:


Sejam A, B, C e D números inteiros, tais que: A = 3k, B = 3w + 1, C = 3f + 2 e D
deixaremos variável.
Assim as possibilidades de D são: D = 3c, D = 3c + 1 ou D = 3c + 2.
Logo se:
D = 3c , então D – A = 3(c – k) e portanto será divisível por 3.
D = 3c + 1 , então D – B = 3(c – w) e portanto será divisível por 3.
D = 3c + 2 , então D – C = 3(c – f) e portanto será divisível por 3.

5) Mostre que entre quaisquer n + 1 números podemos encontrar 2 números
de modo que a diferença entre eles é divisível por n.
Este exercício é a generalização do exercício anterior e é solucionado pelo Principio das
casas de pombos, pois dado um numero inteiro qualquer, a divisão dele por n pode ser
representada de n maneiras: múltiplo de n, resto 1, resto 2, ... , resto n - 1. Desta forma
tendo n + 1 números (pombos) e tendo apenas n casas (múltiplo de n, resto 1, resto 2, ...
, resto n - 1), necessariamente dois deles ficarão na mesma casa, ou seja, a diferença
entre eles seja um múltiplo de n e automaticamente será divisível por n.

Uma demonstração algébrica pode ser dada da seguinte maneira:


Sejam A1, A2, A3, ... , An e D números inteiros, tais que: A1 = nk, A2 = nw + 1, A3 =
nf + 2, ... , An = nt + (n-1) e D deixaremos variável.
Assim as possibilidades de D são: D = nc, D = nc + 1, D = nc + 2, ... , D = nc + (n-1)
Logo se:
D = nc , então D – A1 = n(c – k) e portanto será divisível por n.
D = nc + 1 , então D – A2 = n(c – w) e portanto será divisível por n.
D = nc + 2 , então D – A3 = 3(c – f) e portanto será divisível por n.
.
.
.
D = nc + (n-1) , então D – An = n(c – t) e portanto será divisível por n.

6) Sejam A e B conjuntos com m e n elementos, respectivamente. Quantas
funções injetivas existem de A em B?
Solução:
Inicialmente vamos estudar o caso de m = n.
Sejam A e B conjuntos com m elementos.
1) Quantas funções existem de A em B?
Para cada elemento de A, temos m opções em B, logo o número de funções é: m x

m x ... x m (m vezes), isto é: m.m. m  m .


m

2) Quantas funções são injetivas?


Para o primeiro elemento de A, temos m opções. Para o segundo elemento de A,
temos (m – 1) opções. ... . Para o m – ésimo elemento temos uma opção. Logo
temos m . (m – 1).(m – 2). ... .1 = m!

Agora vamos analisar o caso de m diferente de n.


Sejam A e B conjuntos com m e n elementos, respectivamente.
* Caso 1) m < n
- Quantas funções existem de A e B?
Para cada elemento de A, temos n opções. Logo temos n x n x ... x n (m vezes), isto
m
é, n funções.
- Quantas funções são injetivas?
Para o primeiro elemento de A, temos n opções. Para o segundo elemento de A,
temos ( n – 1) opções. Para o terceiro elemento de A, temos (n – 2) opções. ... . Para
o m – ésimo elemento de A, temos (n – (m – 1)) opções. Logo temos n x (n – 1) x (n
– 2) x ... x (n – (m – 1), isto é, um arranjo.
n.  n  1 .  n  2  .....  n   m  1  .  n  m !
n.  n  1 .  n  2  .....  n   m  1  
n!
 A
 n  m !  n  m ! n,m
* Caso 2) m > n
- Quantas funções existem de A e B?
Para cada elemento de A, temos n opções. Logo temos n x n x ... x n (m vezes), isto
m
é, n funções.
- Quantas funções são injetivas?
Neste caso não há nenhuma função injetiva, pois como m > n, teremos elementos
distintos em A com imagens iguais!
MA13
AVALIAÇÕES

MA13

2011-2013
MA13
-
2011
MA13 – Geometria I – Avaliação 1 – 2011

Questão 1.
A figura abaixo mostra uma sequência de circunferências de centros C1 , C2 , . . ., Cn com raios r1 , r2 , . . ., rn ,
respectivamente, todas tangentes às retas s e t, e cada circunferência, a partir da segunda, tangente à anterior.

C1
C2
C3

Considere r1 = a e r2 = b.

(1,0) (a) Calcule r3 em função de a e b.

(1,0) (b) Calcule rn em função de a e b.

Questão 2.
Na figura abaixo, a circunferência de centro I é tangente em D ao lado BC do triângulo ABC e é tangente em E
e F aos prolongamentos dos lados AB e AC, respectivamente.

F
C

I
D

A
B E

(1,0) (a) Mostre que AE é igual ao semiperı́metro do triângulo ABC.

b é a metade do ângulo ACB.


(1,0) (b) Mostre que o ângulo AIB b
MA13 – Geometria I – Avalilação 1 – 2011

Questão 3.

(2,0) Dado um paralelogramo ABCD construa no seu exterior os triângulos equiláteros BCE e CDF . Mostre que
o triângulo AEF é equilátero.

Questão 4.

(2,0) No triângulo ABC, Bb = 68o e C


b = 40o , AD e BE são alturas, M é médio de BC e N é médio de AC. Calcule
os ângulos DNb M e E DN
b .

Questão 5.

(2,0) O triângulo equilátero ABC está inscrito em uma circunferência e P é um ponto qualquer do menor arco BC.
Prove que P A = P B + P C (isto é, que a distância de P ao ponto A é igual à soma das distâncias de P aos
pontos B e C).
Sugestão: Considere um ponto D sobre P A tal que P D = P B.
AV1 - MA 13 - 2011

Questão 1.
A figura abaixo mostra uma sequência de circunferências de centros C1 , C2 , . . ., Cn com raios r1 , r2 , . . ., rn , respec-
tivamente, todas tangentes às retas s e t, e cada circunferência, a partir da segunda, tangente à anterior.

C1
C2
C3

Considere r1 = a e r2 = b.

(1,0) (a) Calcule r3 em função de a e b.

(1,0) (b) Calcule rn em função de a e b.

UMA SOLUÇÃO

C1
a
b C2
A b x C3
B

(a) Todos os centros estão a igual distância das duas retas, portanto estão na bissetriz das retas s e t. Seja A o ponto
de intersecção entre a paralela à reta t passando por C2 e a perpendicular à reta t passando por C1 , e seja B o ponto
de intersecção entre a paralela à reta t passando por C3 e a perpendicular à reta t passando por C2 . Seja x = r3 .

1
Como os triângulos-retângulos AC1 C2 e BC2 C3 são semelhantes, temos

C1 A C B
= 2 ,
C1 C2 C2 C3
isto é,
a−b b−x
= ,
a+b b+x
b2
o que implica x = a .

(b) A relação obtida


r22
r3 =
r1
pode ser reformulada como
r3 r b
= 2 = ,
r2 r1 a
o que mostra que os três raios formam uma progressão geométrica de razão ba . Como a mesma situação ocorre para
b
quaisquer três circunferências consecutivas, a sequência r1 , r2 , . . ., rn , . . . é uma progressão geométrica de razão a e
termo inicial a. Assim   n −1
b b n −1
rn = a · = n −2 ,
a a
para n = 1, 2, 3, . . ..

2
AV1 - MA 13 - 2011

Questão 2.
Na figura abaixo, a circunferência de centro I é tangente em D ao lado BC do triângulo ABC e é tangente em E e
F aos prolongamentos dos lados AB e AC, respectivamente.

F
C

I
D

A
B E

(1,0) (a) Mostre que AE é igual ao semiperímetro do triângulo ABC.

(1,0) (b) Mostre que o ângulo A b b


IB é a metade do ângulo ACB.

UMA SOLUÇÃO

(a) Seja 2p o perímetro do triângulo ABC. Tem-se

2p = AB + BC + CA = AB + BD + DC + CA = AB + BE + CF + CA = AE + AF = 2AE .

Logo AE = p.

b =A
(b) No triângulo ABC, sejam B AC b e ACB
b = C.
b O ângulo externo de vértice B é D BE
b =A b + C.
b Seja A b
IB = θ.
b e D BE
Como AI e BI são bissetrizes dos ângulos C AB b então, no triângulo ABI, o ângulo externo I BE
b é tal que

b+C
A b b
D BE b
A
= = I BE b + Ab
b = I AB IB = +θ.
2 2 2
Logo
b
C
θ= .
2

3
AV1 - MA 13 - 2011

Questão 3.

(2,0) Dado um paralelogramo ABCD construa no seu exterior os triângulos equiláteros BCE e CDF. Mostre que o
triângulo AEF é equilátero.

UMA SOLUÇÃO

A D

α
B
C
F

Primeiro, vemos que BA = DF = CF. A segunda igualdade é consequência de CDF ser equilátero, enquanto a
primeira segue de que AB = CD (pois ABCD é paralelogramo) e CD = DF (pois CDF é equilátero).
Depois, vemos que AD = BE = EC. A segunda desigualdade segue de BCE ser equilátero. A primeira segue de
que AD = BC (pois ABDC é paralelogramo) e BC = BE (pois BCE é equilátero).
Finalmente, vamos mostrar que os ângulos A BE, b e A DF
b ECF b são iguais. Para isso vamos mostrar que todos
são iguais a α + 60o , em que α é o ângulo A BC.
b De fato, isso é evidente para A BE,
b pois BCE equilátero implica
b = 60o . O mesmo para A DF,
C BE b pois A DC b = 60o
b = α (ângulos opostos do paralelogramos são iguais) e C DF
(CDF é equilátero). Finalmente, em torno do ponto C tem-se

b + D CF
BCD b + F CE b = 360o ,
b + ECE

logo
(180o − α) + 60o + ECF
b + 60o = 360o

b = α + 60o , como queríamos demonstrar.


e, portanto, ECF
Portanto os triângulos ABE, FCE e FDA são congruentes, de onde concluímos que AE = EF = AF, isto é, AEF é
equilátero.

4
AV1 - MA 13 - 2011

Questão 4.

(2,0) No triângulo ABC, Bb = 68o e C


b = 40o , AD e BE são alturas, M é médio de BC e N é médio de AC. Calcule os
b M e E DN.
ângulos D N b

UMA SOLUÇÃO

A
E

B D M C

(A figura não foi desenhada com os ângulos prescritos no enunciado)

b = 180o − 68o − 40o = 72o . Segundo, como N é o ponto médio de AC, então é equidistante de
(a) Primeiro, B AC
A e D. Logo AND é isósceles e ND = N A. Pela mesma razão N A = NC, de onde resulta que NDC é isósceles.
b = 40o e que D NC
b = ACB
Disso resulta que N DC b = 180o − 40o − 40o = 100o . Terceiro, MN é paralelo a BA, logo
MNC é semelhante a BAC e, por conseguinte, M NC b isto é, 72o . Portanto, D N
b é igual a B AC, b M = D NC
b − M NC
b =
100o − 72o = 28o .

b = 180o − D NC
(b) ADN é isósceles e A ND b = 80o , logo A DN
b = 50o .
b = 90o = B EA,
Como B DA b então E e D pertencem à circunferência cujo diâmetro é AB. Logo, os ângulos A BE
b e
b inscritos nessa circunferência são iguais. Então A DE
A DE b = 90o − 72o = 18o .
b = A BE
b = A DN
Portanto E DN b = 50o − 18o = 32o .
b − A DE

5
AV1 - MA 13 - 2011

Questão 5.

(2,0) O triângulo equilátero ABC está inscrito em uma circunferência e P é um ponto qualquer do menor arco BC.
Prove que PA = PB + PC (isto é, que a distância de P ao ponto A é igual à soma das distâncias de P aos pontos
B e C).
Sugestão: Considere um ponto D sobre PA tal que PD = PB.

UMA SOLUÇÃO

B C

Seja D o ponto do segmento PA tal que PD = PB. Precisamos mostrar que AD = PC.
Como o arco AB mede 120o , então B PA
b = 60o . Então B PD
b = 60o (é o mesmo ângulo) e, como PB = PD, então
b = 60o e, por conseguinte,
PBD é equilátero, resultando que BD = PB. Também por PBD ser equilátero tem-se B DP
b = 120o .
B DA
b = 240o = 120o , logo B PC
Como o arco BAC mede 240o , então B PC b = B DA.
b Juntando essa informação com a
2
b b b = P BC.
igualdade B AP = BCP, que é evidente da simetria da construção, concluímos que A BD b
Por LAL os triângulos ABD e CBP são congruentes, resultando que AD = PC, como queríamos demonstrar.

6
MA13 – Geometria I – Avaliação 2 – 2011

Questão 1
(2,0) A figura abaixo mostra um triângulo equilátero e suas circunferências inscrita e
circunscrita. A circunferência menor tem raio 1.
Calcule a área da região sombreada.

Questão 2
O poliedro P que inspirou a bola da Copa de 70 é formado por faces
pentagonais e hexagonais, e é construído da seguinte forma:
• Considere um icosaedro regular de aresta a (Fig. 1 abaixo).
• A partir de um vértice e sobre cada uma das 5 arestas que concorrem nesse
a
vértice, assinale os pontos que estão a uma distância de desse vértice. Esses
3
5 pontos formam um pentágono regular (Fig. 2).
• Retirando a pirâmide de base pentagonal que ficou formada obtemos a Fig. 3.
• Repetindo a mesma operação para todos os vértices do icosaedro obtém-se o poliedro P.

Fig. 1 Fig. 2 Fig. 3

(0,5) (a) Determine quantas são as faces pentagonais e quantas são as faces hexagonais de P.

(0,7) (b) Determine os números de arestas, faces e vértices de P.

(0,8) (c) Sabendo que uma diagonal de um poliedro é todo segmento que une dois vértices que
não estão na mesma face, determine o número de diagonais de P.
MA13 – Geometria I – Avaliação 2 – 2011

Questão 3

Definição: Dado um segmento AB, o plano mediador desse segmento é o plano perpendicular a
AB que contém o seu ponto médio.

1ª Parte
(2,0) Prove que um ponto P equidista de dois pontos A e B se, e somente se, pertence ao plano
mediador de AB.

2ª Parte H
A figura abaixo mostra o cubo ABCD-EFGH de aresta G
a. E
Sejam M, N, P, Q, R e S os pontos médios das arestas F
AB, BF, FG, GH, HD e DA.

(0,5) (a) Mostre que esses seis pontos são coplanares. D


Sugestão: Mostre que qualquer um deles pertence ao plano
mediador da diagonal EC do cubo (a propriedade enunciada na C
primeira parte da questão pode ser utilizada mesmo que você não
A
a tenha demonstrado). B

(0,5) (b) Mostre que o hexágono MNPQRS é regular.

(1,0) (c) Calcule o volume da pirâmide de vértice E e base MNPQRS.

H
3ª Parte G
A figura abaixo mostra o cubo ABCD-EFGH de aresta a.
E
(1,0) (a) Mostre que as retas DB e EC são ortogonais. F

(1,0) (b) Calcule o comprimento da perpendicular


comum entre DB e EC. D
C

A
B
MA13 – Geometria I – Avaliação 2 – 2011
Gabarito

Questão 1
(2,0) A figura abaixo mostra um triângulo equilátero e suas circunferências inscrita e
circunscrita. A circunferência menor tem raio 1.
Calcule a área da região sombreada.

Uma solução:

X
O
Y
B M C

Seja O, o centro do triângulo equilátero ABC e seja M o ponto médio do lado BC como na
figura acima. Pela propriedade do baricentro do triângulo, OA 2 OM e como OM 1 temos
OA 2 .
A região cuja área se pede é formada por duas partes justapostas X e Y como mostra a figura.
Observando que 3 X 3Y é a área da coroa circular formada pelas duas circunferências temos
3( X Y ) 22 12 3 .
Logo, X Y .
Questão 2
O poliedro P que inspirou a bola da Copa de 70 é formado por faces
pentagonais e hexagonais, e é construído da seguinte forma:
•Considere um icosaedro regular de aresta a (Fig. 1 abaixo).
•A partir de um vértice e sobre cada uma das 5 arestas que concorrem nesse
a
vértice, assinale os pontos que estão a uma distância de desse vértice. Esses
3
5 pontos formam um pentágono regular (Fig. 2).
•Retirando a pirâmide de base pentagonal que ficou formada obtemos a Fig. 3.
•Repetindo a mesma operação para todos os vértices do icosaedro obtém-se o poliedro P.

Fig. 1 Fig. 2 Fig. 3

(0,5) (a) Determine quantas são as faces pentagonais e quantas são as faces hexagonais de P.

(0,7) (b) Determine os números de arestas, faces e vértices de P.

(0,8) (c) Sabendo que uma diagonal de um poliedro é todo segmento que une dois vértices que
não estão na mesma face, determine o número de diagonais de P.

Uma solução:

(a) Cada face pentagonal de P apareceu onde havia um vértice do icosaedro. Como o icosaedro
tem 12 vértices então P tem 12 faces pentagonais. Cada face (triangular) do icosaedro deu
origem a uma face hexagonal de P. Como o icosaedro tem 20 faces triangulares então P tem 20
faces hexagonais.

(b) Do item anterior temos F5 12 e F6 20


O número total de faces de P é F F5 F6 1
12 2
20 32 .
3
Contando as arestas temos: 2 A 5 F5 6 F6 5 12 6 20 1
180 , ou seja, A 9 .
90
Como P é convexo então vale a relação de Euler V A F 2 . Portanto, V 6 .
60

(c) Seja d n o número de diagonais de um polígono de n lados.


O número de diagonais de um pentágono é d 5 5 e o de um hexágono é d 6 9.
A soma dos números de diagonais de todas as faces é S F5 d 5 F6 d 6 12
1 5 20
2 240 .
2
Vamos agora construir todos os segmentos cujas extremidades são os V vértices do poliedro P.
A quantidade de diagonais de P é D CV2 A S .
6
60 59
Assim, D C602 9
90 2
240 9
90 240
2 1170
1 330
3 1440.
1
2
Questão 3

Definição: Dado um segmento AB, o plano mediador desse segmento é o plano perpendicular a
AB que contém o seu ponto médio.

1ª Parte
(2,0) Prove que um ponto P equidista de dois pontos A e B se, e somente se, pertence ao plano
mediador de AB.

Uma solução:

Seja M o ponto médio de AB e seja Π o plano mediador de AB.

A
(a) Suponha que P pertença a Π. Se P coincide com M então
Π equidista de A e B. Se não, como AB é perpendicular a Π então AB
P M é perpendicular a MP. Como M é médio de AB então os triângulos
retângulos MPA e MPB são congruentes.
Logo, PA P PB , ou seja, P equidista de A e B.
B

A (b) Suponha que P não pertença a Π. Imaginemos, por exemplo e


sem perda de generalidade, os pontos P e A no mesmo semiespaço
P Π determinado por Π. Como B está no semiespaço oposto a reta PQ
corta Π em um ponto Q. Como Q então, pela parte a),
Q
QA Q QB .
No triângulo PAQ tem-se: PA PQ P Q
QA P
PQ QB
Q PB .
P
B
Assim, P não equidista de A e B.
2ª Parte
A figura abaixo mostra o cubo ABCD-EFGH de aresta a. H
Sejam M, N, P, Q, R e S os pontos médios das arestas G
AB, BF, FG, GH, HD e DA. E
F
(0,5) (a) Mostre que esses seis pontos são coplanares.
Sugestão: Mostre que qualquer um deles pertence ao plano
mediador da diagonal EC do cubo (a propriedade enunciada na
primeira parte da questão pode ser utilizada mesmo que você não a D
tenha demonstrado). C

A
(0,5) (b) Mostre que o hexágono MNPQRS é regular. B

(1,0) (c) Calcule o volume da pirâmide de vértice E e base MNPQRS.

Uma solução:
(a) Tomemos o ponto M, médio da aresta AB. Os
triângulos AME e BMC são congruentes, pois AM B BM ,
0
E AE B BC e MAE MBC 90 9
Logo, ME M MC e, portanto, M pertence ao plano
mediador da diagonal EC.
Analogamente, cada um dos outros pontos: N, P, Q, R e S
C também estão nesse mesmo plano.
A
M B
(b) Cada lado do hexágono é a metade da diagonal de
H Q B
BG a 2
G uma face. Por exemplo, NP .
2 2
E P Seja O, o centro do cubo. Todos os vértices do
R F hexágono possuem mesma distância ao ponto O. A
O distância do centro do cubo a qualquer aresta é a
a 2
metade da diagonal de uma face, ou seja, .
N 2
S Portanto, cada um dos triângulos MON, NOP, ...,
C
SOM é equilátero e o hexágono é regular.
A
M
B
a2 3 3 3a 2
(c) A área do hexágono é 6 .
4 2
a 3
Como a altura da pirâmide é a metade da diagonal do cubo temos OE .
2
1 3 3a 2 a 3 3a 3
O volume da pirâmide é: V .
3 2 2 8
H
3ª Parte G
A figura abaixo mostra o cubo ABCD-EFGH de aresta a.
E
(1,0) (a) Mostre que as retas DB e EC são ortogonais. F

(1,0) (b) Calcule o comprimento da perpendicular


comum entre DB e EC. D
C

A
B

Uma solução:
H
G

Y
D
C
X
A
B

(a) Seja Π o plano diagonal AEGC.


Como AE é perpendicular ao plano ABCD então AE é ortogonal a BD. Mas AC é perpendicular
a BD (pois as diagonais de um quadrado são perpendiculares. Como BD é ortogonal a AE e AC
então BD é perpendicular a Π.
Como EC está contida em Π então BD é ortogonal a EC.

(b) Seja X o ponto onde BD fura o plano Π. O ponto X é o centro da face ABCD.
Sobre o plano Π tracemos XY perpendicular a EC.
Lembrando que BD é perpendicular a Π então BD é perpendicular a XY. Assim, XY é a
perpendicular comum entre BD e EC.

Os triângulos retângulos CYX e CAE são semelhantes. Logo,

XY C
CX XY a 2 2 a 6
→ → XY
AE CE
C a a 3 6
MA13 – Geometria I – Avaliação 3 – 2011

Questão 1
Considere um quadrado ABCD de lado a e seja E o ponto do lado CD tal que
AE BC CE .
(1,0) (a) Calcule o comprimento de CE.
(1,0) (b) Calcule o seno do ângulo CAˆ E .

Questão 2
Um trapézio ABCD tem altura h e bases AB a e CD b . Seja F o ponto de
interseção das diagonais.
(1,0) (a) Calcule as distâncias de F às duas bases.
(1,0) (b) Calcule as áreas dos triângulos ADF e BCF.

Questão 3
Seja ABC um triângulo qualquer. Desenhe exteriormente a ABC os triângulos
equiláteros ABD e ACE.
(1,0) (a) Mostre que DC = BE. Sugestão: use congruência de triângulos.
(0,5) (b) Sendo F o ponto de interseção de DC e BE, mostre que o quadrilátero ADBF
é inscritível.
(0,5) (c) Mostre que AFˆ B BFˆC CFˆA 1200 .

Questão 4
Seja um plano horizontal. A reta r é perpendicular a e seja A o ponto de
interseção de r e . A reta s está contida em e não passa por A. O ponto B da reta s
é tal que AB é perpendicular à reta s. Seja M um ponto de r e N um ponto de s.
Dados: AM a , BN b, AB c .
(0,5) (a) Faça um desenho da situação descrita no enunciado.
(0,5) (b) Calcule a distância entre os pontos M e N.
(0,5) (c) Calcule a tangente do ângulo que a reta MN faz com o plano .
(0,5) (d) Calcule a tangente do ângulo entre as retas AB e MN.

Questão 5
As bases de um tronco de pirâmide regular são quadrados de lados 12 e 4. Sabe-se
que a área lateral é igual à soma das áreas das bases.
(1,0) (a) Calcule a altura do tronco.
(1,0) (b) Calcule o volume do tronco.
MA13 – Geometria I – Avaliação 3 – 2011

Questão 1
Considere um quadrado ABCD de lado a e seja E o ponto do lado CD tal que
AE BC CE .
(1,0) (a) Calcule o comprimento de CE.
(1,0) (b) Calcule o seno do ângulo CAˆ E .

Questão 2
Um trapézio ABCD tem altura h e bases AB a e CD b . Seja F o ponto de
interseção das diagonais.
(1,0) (a) Calcule as distâncias de F às duas bases.
(1,0) (b) Calcule as áreas dos triângulos ADF e BCF.

Questão 3
Seja ABC um triângulo qualquer. Desenhe exteriormente a ABC os triângulos
equiláteros ABD e ACE.
(1,0) (a) Mostre que DC = BE. Sugestão: use congruência de triângulos.
(0,5) (b) Sendo F o ponto de interseção de DC e BE, mostre que o quadrilátero ADBF
é inscritível.
(0,5) (c) Mostre que AFˆ B BFˆC CFˆA 1200 .

Questão 4
Seja um plano horizontal. A reta r é perpendicular a e seja A o ponto de
interseção de r e . A reta s está contida em e não passa por A. O ponto B da reta s
é tal que AB é perpendicular à reta s. Seja M um ponto de r e N um ponto de s.
Dados: AM a , BN b, AB c .
(0,5) (a) Faça um desenho da situação descrita no enunciado.
(0,5) (b) Calcule a distância entre os pontos M e N.
(0,5) (c) Calcule a tangente do ângulo que a reta MN faz com o plano .
(0,5) (d) Calcule a tangente do ângulo entre as retas AB e MN.

Questão 5
As bases de um tronco de pirâmide regular são quadrados de lados 12 e 4. Sabe-se
que a área lateral é igual à soma das áreas das bases.
(1,0) (a) Calcule a altura do tronco.
(1,0) (b) Calcule o volume do tronco.
MA13 – Geometria I – Avaliação 3 – 2011
Gabarito

Questão 1 – Solução
D E C
(a) Seja CE x . Assim AE a x .
Traçando EF perpendicular a AB temos no triângulo AEF:
a
(a x) 2 (a x) 2 a2 o que dá x . a
4

(b) Seja AEˆ C .


a a 5a
Como CE e AE a temos, pela lei dos senos, A F B
4 4 4
a4 5a 4 2
o que dá sin
n .
sin 2 2 10

Questão 2 – Solução
D b C
(a) Sejam x e y as distâncias de F às bases AB e y
CD, respectivamente. Como os triângulos FAB e F
FCD são semelhantes, temos: h
a b a b x
x y h
ah bh A a B
Assim, x e y .
a b a b

(b) Os triângulos ADB e ACB têm mesma área porque possuem mesma base e mesma
altura. Os triângulos ADF e BCF têm mesma área porque
[ADF] = [ADB] – [AFB] = [ACB] – [AFB] = [BCF]

ah
a ax
a a ah abh
[ ADF ] [ BCF ] h
2 2 2 a b 2(a b)
Questão 3 – Solução

(a) Temos AD AB , AC AE e DAˆ C BAˆ E Aˆ 600 . Portanto, os triângulos


ADC e ABE são congruentes e DC = BE.

ˆF
(b) Pela congruência anterior, AD ABˆ F . Portanto D está na circunferência que
passa por A, B e F.

(c) Como ADBF é inscritível, seus ângulos são suplementares. Então


AFˆB 1800 AD ˆ B 1800 600 1200 . Analogamente, AECF é inscritível e
CFˆA 1200 . Consequentemente, BFˆC 1 1200 .

Questão 4 – Solução

(a) r

A
c P

B
b N s

b) No triângulo ABN, retângulo em B, AN 2 b 2 c 2 .


No triângulo MAN, retângulo em A, MN 2 a 2 AN 2 a2 b2 c 2 .
Então MN a2 b2 c2 .

AM a
c) O ângulo que MN faz com é MNˆ A . Assim, tan .
AN b 2
c2

d) Construa o retângulo ABNP.


AM é ortogonal a NP e AP é perpendicular a NP. Portanto, NP é perpendicular ao
plano AMP e, consequentemente, o ângulo NPA é reto.
O ângulo entre MN e BA é o ângulo entre MN e NP, MNˆ P .
P
PM a2 b2
Assim, tan .
NP
N c
Questão 5 – Solução

(a)
V
Sejam O e O os centros
das duas bases (maior e
menor) como mostra a O' N
figura acima. 4
Na reta OO está o h x
vértice V da pirâmide que
dou origem ao tronco.
A altura do tronco é
O P M
OO h .
Cada face lateral do 12
tronco é um trapézio
isósceles, e a altura de
um dos trapézios é o segmento MN que une os pontos médios das duas bases. Seja
MN x .
A área lateral do tronco é a soma das áreas dos quatro trapézios. Então,

(12 4) x
4 1 2
12 42
2

Isto dá x 5 . Trace agora NP perpendicular à OM como na figura acima. Temos


O O NP N h , ON OP 2 , OM 6 e, consequentemente, PN 4 . No triângulo
PMN retângulo em P temos h 3 .

(b) Seja VO y.
y 4 3
Utilizando a semelhança entre as duas pirâmides temos o que dá y .
y 3 12
1 2
3
9
A altura da pirâmide grande é OV 3 e o seu volume é
2
2
1 9
V1 122 216 .
2
3 2
1 2 3
O volume da pirâmide pequena é V2 4 8.
3 2
O volume do tronco é a diferença: V 216
2 8 2 208 unidades de volume.

Obs:
Pode-se também aplicar a fórmula do volume do tronco de pirâmide:
h
V ( S1 S 2 S1 S 2 ) onde S1 e S 2 são as áreas das duas bases e h é a altura do
3
tronco. Assim,
3 2
V (12 4 2 122 4 2 ) 1 144 1 16 48
4 208 .
2
3
MA13
-
2012
MA13 Geometria I

Avaliação 1 2012/2

NOME: −−−−−−−−−−−−−−−−−−−−−−−−−−−−−−−−−−−−−−−−−−−−−−−−−−−−−−−−−−−−−−−−−−−−−−−−−−−−−−−−−−−−−

Questão 1. (pontuação: 2)

O ponto D pertence ao lado AC do triângulo ABC. Sabe-se que AB = BC = CD e que o ângulo ABD mede 21o .
Determine a medida do ângulo ABC.

Questão 2. (pontuação: 2)

Quadrados foram construı́dos sobre os lados de um paralelogramo como mostra a figura abaixo. Mostre que os
centros desses quatro quadrados são vértices de outro quadrado.

Questão 3. (pontuação: 2)

No triângulo ABC de lados AB = 8, BC = 7 e AC = 9, os pontos M e N dos lados AB e AC, respectivamente,


são tais que o segmento M N é tangente à circunferência inscrita no triângulo ABC. Mostre que o perı́metro do
triângulo AM N é constante e calcule seu valor.
MA13 Geometria I Avaliação 1 2012

Questão 4. (pontuação: 2)

No trapézio ABCD os ângulos A e D são retos, AB = 12, CD = 4 e AD = 10. O ponto E pertence ao lado AD e o
ponto F pertence ao lado BC. Sabe-se que as retas EF e AB são paralelas e que o segmento EF fica dividido em
três partes iguais pelas diagonais do trapézio. Calcule a distância entre as retas AB e EF .

Questão 5. (pontuação: 2)

A figura abaixo mostra o triângulo acutângulo ABC inscrito na circunferência de centro O. A reta BD é perpendicular
em D a AC e encontra a circunferência em M . A reta CE é perpendicular em E a AB e encontra a circunferência
em N . As alturas BD e CE intersectam-se em H, ortocentro do triângulo.

a) Mostre que HD = DM .

b) Mostre que M N é perpendicular a OA.

M
A

N E
H O

B C
MA13 Geometria I Avaliação 1 2012

SOLUÇÕES

Questão 1. (pontuação: 2)
O ponto D pertence ao lado AC do triângulo ABC. Sabe-se que AB = BC = CD e que o ângulo ABD mede 21o .
Determine a medida do ângulo ABC.

21o

α α
A D C

Uma solução:
Como AB = BC, seja α = ∠BAC = ∠BCA. O ângulo BDC é externo do triângulo ABD. Então, ∠BDC =
21o + α = ∠DBC, pois BC = CD. No triângulo BDC temos 21o + α + 21o + α + α = 180o , ou seja, α = 46o . O
ângulo ABC mede 21o + 21o + α = 42o + 46o = 88o .

Questão 2. (pontuação: 2)
Quadrados foram construı́dos sobre os lados de um paralelogramo como mostra a figura abaixo. Mostre que os
centros desses quatro quadrados são vértices de outro quadrado.

H x D
y C
x β
α
A B F

E
Uma solução:

No paralelogramo ABCD os quadrados construı́dos sobre os lados AB, BC, CD e DA têm centros E, F , G e H,
respectivamente.
Os triângulos AEB, BF C, CGD e DHA são retângulos e isósceles. O primeiro e o terceiro são congruentes e o
segundo e o quarto são também congruentes.
Sejam ∠BAD = α e ∠ADC = β dois ângulos internos vizinhos do paralelogramo. Sabemos que α + β = 180o .
Observemos que ∠HAE = 45o + α + 45o = 90o + α e que ∠HDG = 360o − 45o − 45o − β = 270o − (180o − α) =
90o + α = ∠HAE.
Reunindo as informações anteriores concluı́mos que os triângulos HAE, HDG, F CG e F BE são todos congruentes
e, portanto, EH = HG = GF = F E e o quadrilátero EF GH possui os quatro lados iguais.
Da congruência dos triângulos HAE e HDG temos ∠AHE = ∠DHG = x e seja ∠EHD = y. Por um lado,
∠AHE + ∠EHD = x + y = 90o , pois o ângulo AHD é reto.
Por outro lado, ∠EHG = ∠DHG + ∠EHD = x + y = 90o .
Assim, o quadrilátero EF GH possui os quatro lados iguais e um ângulo reto. Logo, é um quadrado.

Questão 3. (pontuação: 2)
No triângulo ABC de lados AB = 8, BC = 7 e AC = 9, os pontos M e N dos lados AB e AC, respectivamente,
são tais que o segmento M N é tangente à circunferência inscrita no triângulo ABC. Mostre que o perı́metro do
triângulo AM N é constante e calcule seu valor.

Uma solução:

z
x
y N
M R
P

B Q C

Sejam AM = x, M N = y e N A = z os lados do triângulo AM N . Temos M B = 8 − x e CN = 9 − z . Como o


quadrilátero BCN M é circunscritı́vel temos, pelo teorema de Pitot (Unidade 7, Teorema 4), BC + M N = M B + N C
ou seja, 7+y = 8−x+9−z. Logo x+y +z = 10. Portanto o perı́metro do triângulo AM N é igual a 10, independente
da posição do segmento M N .

Outra solução:
A circunferência inscrita em ABC é uma circunferência exiscrita ao triângulo AM N . Sabemos que o semiperı́metro
do triângulo AM N é o segmento AP que é constante, ou seja, não depende da posição do segmento M N (Unidade
7, Proposição 22). Fazendo AP = AR = a, BP = BQ = b e CQ = CR = c, temos as equações:

 a+b=8


b+c=7

 c+a=9

Resolvendo, encontramos a = 5 que é o semiperı́metro do triângulo AM N . Logo, o perı́metro de AM N é 10.

Questão 4. (pontuação: 2)
No trapézio ABCD os ângulos A e D são retos, AB = 12, CD = 4 e AD = 10. O ponto E pertence ao lado AD e o
ponto F pertence ao lado BC. Sabe-se que as retas EF e AB são paralelas e que o segmento EF fica dividido em
três partes iguais pelas diagonais do trapézio. Calcule a distância entre as retas AB e EF .

Uma solução:
O problema tem duas soluções pois há duas possibilidades: quando EF está abaixo do encontro das diagonais do
trapézio e quando EF está acima do encontro das diagonais do trapézio. Qualquer uma das soluções está igualmente
correta.

Primeira situação:

D 4 C

10 – x

P Q
E F
m m m
10

12 B
A
Na figura acima, seja AE = x. Então, ED = 10 − x. Como as diagonais dividem EF em três partes iguais sejam
2x
EP = P Q = QF = m. Da semelhança dos triângulos AEP e ADC temos: m
4 = x
10 ⇒ m= 5 . Da semelhança
2m 10−x 3(10−x)
dos triângulos DEQ e DAB temos: 12 = 10 ⇒ m= 5 .
Igualando temos 2x = 3(10 − x), o que dá x = 6.

Segunda situação:

D 4 C
P Q
E m m m F

x
10

A 12 B

Na figura acima, seja AE = x. Então, ED = 10 − x. Como as diagonais dividem EF em três partes iguais sejam
2m 4
EP = P Q = QF = m. Da semelhança dos triângulos QEA e CDA temos: x = 10 ⇒ m= 5.
x
Da semelhança
10−x 10 12(10−x)
dos triângulos DEP e DAB temos: m = 12 ⇒ m= 10 .
60
Igualando obtemos x = 7 .

Questão 5. (pontuação: 2)
A figura abaixo mostra o triângulo acutângulo ABC inscrito na circunferência de centro O. A reta BD é perpendicular
em D a AC e encontra a circunferência em M . A reta CE é perpendicular em E a AB e encontra a circunferência
em N . As alturas BD e CE intersectam-se em H, ortocentro do triângulo.
a) Mostre que HD = DM .
b) Mostre que M N é perpendicular a OA.
Uma solução:

M
A

N E
β
β O
H α
α
α
B C

a) Considerando a figura acima, sejam ∠DCH = α e ∠DHC = β. Como o ângulo HDC é reto então α e β
são complementares. Temos ∠EHB = β (oposto pelo vértice de DHC) e ∠HBE = α pois o ângulo BEH é reto.
Escrevemos ∠ABM = ∠HBE = α. Como os ângulos inscritos ABM e ACM subtendem o mesmo arco AM , então
são iguais, ou seja, ∠ACM = α.
Os triângulos retângulos CDH e CDM são congruentes. Assim HD = DM , como querı́amos demonstrar.

b)

A M

N
O
α
α
C

Os arcos AM e AN são iguais porque ∠ACM = ∠ACN = α. Como arcos iguais subtendem cordas iguais o ponto
A equidista dos pontos M e N . Entretanto o ponto O, centro da circunferência também equidista de M e N . Assim,
A e O são pontos da mediatriz do segmento M N o que significa dizer que a reta AO é a mediatriz do segmento M N .
Logo, OA é perpendicular a M N .
MA13 Geometria I - Avaliação 2 - 2012/2

NOME: −−−−−−−−−−−−−−−−−−−−−−−−−−−−−−−−−−−−−−−−−−−−−−−−−−−−−−−−−−−−−−−−−−−−−−−−−−−−−−−−−−−−−

Questão 1. (pontuação: 2)

No setor AOB de centro O, raio OA = 3 e ângulo AOB = 60o está inscrita uma circunferência como mostra a figura.

a) Calcule o raio dessa circunferência.

b) Calcule a área da região sombreada.

O A

Questão 2. (pontuação: 2)

O Teorema das Três Perpendiculares tem o seguinte enunciado:


“A reta r é perpendicular ao plano α no ponto A. A reta s está contida em α e não passa por A. O ponto B da
reta s é tal que AB é perpendicular a s. Então, se P é qualquer ponto de r, P B é perpendicular a s.”

a) Faça uma figura que descreva o enunciado do Teorema.

b) Demonstre o Teorema.

Questão 3. (pontuação: 2)

Em um cubo, ABCD e EF GH são faces opostas e AE, BF , CG e DH são arestas paralelas. Sejam M e N os
pontos médios das arestas BC e DH, respectivamente.

a) Se a aresta do cubo mede 2, calcule a distância entre os pontos M e N .

b) Calcule o cosseno do ângulo entre as retas AB e N M .

Questão 4. (pontuação: 2)

O trapézio ABCD tem bases AB e CD. A altura do trapézio mede 8. As bases medem AB = 10 e CD = 6. As
diagonais AC e BD do trapézio dividiram o trapézio em quatro triângulos. Calcule as áreas dos quatro triângulos
em que o trapézio ficou dividido.
Questão 5. (pontuação: 2)

No cubo ABCDA′ B ′ C ′ D′ de aresta a, os pontos M , N , P e Q são médios das arestas A′ B ′ , B ′ C ′ , C ′ D′ e A′ D′ ,


respectivamente. Foram feitas as seções pelos planos AM Q, BN M , CP N e DQP . Retirando-se os quatro tetraedros
formados, resultou o poliedro P com vértices em A, B, C, D, M, N, P e Q, como ilustrado na Figura 1. O poliedro P
possui duas bases paralelas e faces laterais triangulares. Ele é um prismatóide.
a) Calcule o volume do poliedro P em função de a.

D' P
C'
Q
N
A' M
B'

D
C

A
B
Figura 1

Observe agora a Figura 2; pelo ponto médio X da aresta AA′ foi traçado um plano paralelo à face ABCD que
determinou em P uma seção octogonal. A forma dessa seção equidistante das bases do poliedro P, que é chamada
se seção média, está ilustrada na Figura 3.
No poliedro P, representaremos a área da base ABCD por S, a área da base M N P Q por s, a área da seção média
por Sm e a distância entre as bases por h.

b) Calcule a área da seção média e calcule o volume de P usando a fórmula do volume dos prismatóides:
V = h6 (S + s + 4Sm ).

D' P
C'
Q N
A' M
B'

X
D
C

A B X
Figura 2 Figura 3
MA13 Geometria I - GABARITO DA AVALIAÇÃO 2 - 2012/2

Questão 1. (pontuação: 2)

No setor AOB de centro O, raio OA = 3 e ângulo AOB = 60o está inscrita uma circunferência como mostra a figura.

a) Calcule o raio dessa circunferência.

b) Calcule a área da região sombreada.

Uma solução

r M
C
r
O D A

a) Seja M o ponto médio do arco AB. O raio OM passa pelo centro C da circunferência inscrita no setor. Seja CD
perpendicular a OA como mostra a figura acima e seja r = CD = DM o raio da circunferência. Como AOM = 30o
então, no triângulo ODC tem-se OC = 2r e, portanto, OM = 3r = 3, ou seja, r = 1 .

b) A área sombreada (S) é igual à área do setor AOM subtraı́da da área do triângulo ODC e da área do setor
DCM do cı́rculo de centro C.
π32 3π
A área do setor AOM é 12 = 4 .
√ √
OD.OC 3
Como CD = 1 e OC = 2, então OD = 3 e a área do triângulo ODC é 2 = 2 . Por outro lado, o ângulo
o
DCM mede 120 e, portanto, o setor DCM do cı́rculo de centro C tem área igual à terça parte da área do cı́rculo
√ √
π 3π 3 π 5π−6 3
de centro C, ou seja, 3. Assim, a área sombreada é S = 4 − 2 − 3 , ou seja, S = 12 .

Questão 2. (pontuação: 2)

O Teorema das Três Perpendiculares tem o seguinte enunciado:


“A reta r é perpendicular ao plano α no ponto A. A reta s está contida em α e não passa por A. O ponto B da
reta s é tal que AB é perpendicular a s. Então, se P é qualquer ponto de r, P B é perpendicular a s.”

a) Faça uma figura que descreva o enunciado do Teorema.

b) Demonstre o Teorema.
Uma solução
a)

r
β
P

α A
B

b) Como r é perpendicular a α então r é ortogonal a qualquer reta de α , portanto r é ortogonal a s. Porém,


AB é perpendicular a s. Assim, s é ortogonal a duas retas concorrentes: AB e r. Logo s é perpendicular ao plano
determinado por AB e r, que chamaremos de plano β. Como P e B são pontos de β então s é perpendicular a P B,
como querı́amos demonstrar.

Questão 3. (pontuação: 2)

Em um cubo, ABCD e EF GH são faces opostas e AE, BF , CG e DH são arestas paralelas. Sejam M e N os
pontos médios das arestas BC e DH, respectivamente.

a) Se a aresta do cubo mede 2, calcule a distância entre os pontos M e N .

b) Calcule o cosseno do ângulo entre as retas AB e N M .

Uma solução

H
G

E
N F
O

D
C

θ M
A B


a) Observe a figura acima. No triângulo CDN , retângulo em D, CD = 2 e DN = 1. Consequentemente, N C = 5.
2 2 2
Como a aresta BC é perpendicular à face DCGH, o triângulo M CN é retângulo em C. Daı́, M N = N C + M C =

5 + 1 = 6, ou seja M N = 6.
b) Façamos uma translação do segmento M N de forma que o ponto M concida com o ponto B. Nessa translação,
o ponto N coincidirá com o ponto O, centro da face ADHE. O ângulo entre as retas reversas AB e N M é o ângulo
entre as concorrentes AB e OB, ou seja, o ângulo ABO = θ.

No triângulo ABO, temos AB = 2, OB = N M = 6 e AO é a metade da diagonal do quadrado ADHE, ou seja,

AO = 2.
A lei dos cossenos no triângulo ABO fornece:

√ √ √
( 2)2 = 22 + ( 6)2 − 2.2. 6.cos θ

6
Daı́, encontramos cos θ = 3 . Isto também pode ser obtido notando-se que o triângulo ABO é retângulo em A e
usando-se diretamente a definição de cosseno.

Questão 4. (pontuação: 2)

O trapézio ABCD tem bases AB e CD. A altura do trapézio mede 8. As bases medem AB = 10 e CD = 6. As
diagonais AC e BD do trapézio dividiram o trapézio em quatro triângulos. Calcule as áreas dos quatro triângulos
em que o trapézio ficou dividido.

Uma solução

D 6 C
x

P
8

8–x

A B
10

Notação: (XY Z) representa a área do triângulo de vértices X, Y e Z.


Seja P o ponto de interseção das diagonais e seja x a distância de P à base menor do trapézio. Como os triângulos
6 x 6.3 10.5
P AB e P CD são semelhantes temos 10 = 8−x , o que dá x = 3 . Assim, (P CD) = 2 = 9 e (ABC) = 2 = 25.

10.8
(DAP ) = (DAB) − (P AB) = − 25 = 40 − 25 = 15
2

(CP B) = (CAB) − (P AB) = 40 − 25 = 15


As áreas dos quatro triângulos estão na figura abaixo:

15 15

25
Questão 5. (pontuação: 2)

No cubo ABCDA′ B ′ C ′ D′ de aresta a, os pontos M , N , P e Q são médios das arestas A′ B ′ , B ′ C ′ , C ′ D′ e A′ D′ ,


respectivamente. Foram feitas as seções pelos planos AM Q, BN M , CP N e DP Q. Retirando-se os quatro tetraedros
formados, resultou o poliedro P ilustrado na Figura 1. O poliedro P possui duas bases paralelas e faces laterais
triangulares. Ele é um prismatóide.
a) Calcule o volume do poliedro P .

D' P
C'
Q
N
A' M
B'

D
C

A
B
Figura 1

Observe agora a Figura 2; pelo ponto médio X da aresta AA′ foi traçado um plano paralelo à face ABCD que
determinou em P uma seção octogonal. A forma dessa seção equidistante das bases do poliedro P , que é chamada
se seção média, está ilustrada na Figura 3.
No poliedro P , representaremos a área da base ABCD por S, a área da base M N P Q por s, a área da seção média
por Sm e a distância entre as bases por h.

b) Calcule a área da seção média e calcule o volume de P usando a fórmula do volume dos prismatóides: V =
h
6 (S + s + 4Sm ) .

D' P
C'
Q N
A' M
B'

X
D
C

A B X
Figura 2 Figura 3
Uma solução

a
a) Um dos tetraedros retirados é AA´M Q. Sua base é o triângulo retângulo A´M Q de catetos A´M = A´Q = 2 e
3
altura AA´= a . O volume desse tetraedro é 31 . (a/2)(a/2)
2 .a = a
24 . Como quatro desses tetraedros foram retirados, o
3 a3 3 a3 5a3
volume do poliedro P é V = a − 4. 24 = a − 6 = 6 .

b) A seção média é obtida de um quadrado XY ZW , congruente com ABCD retirando-se quatro triângulos
retângulos isósceles congruentes. O plano da seção média corta a aresta BB´do cubo em Y e corta as arestas M A e
M B do poliedro P em E e F , respectivamente.

X E a/2 F Y

AB a
Temos EF = 2 = 2 (já que a reta XY une pontos médios de lados do triângulo AM B) e, consequentemente,
a2
XE = F Y = a4 . Assim, cada um dos pequenos triângulos retângulos tem área 12 . a4 . a4 = 32 e a área da seção média é
2 2
2
Sm = a − 4. a32 = 7a8 .
Aplicando a fórmula do volume do prismatóide temos:

a 2 a2 7a2 a a2 7a2 a 10a2 5a3


V =
(a + + 4. ) = (a2 + + )= . =
6 2 8 6 2 2 6 2 6
o que coincide com o resultado do item a).
MA13 Geometria I - Avaliação 3 - 2012/2

NOME: −−−−−−−−−−−−−−−−−−−−−−−−−−−−−−−−−−−−−−−−−−−−−−−−−−−−−−−−−−−−−−−−−−−−−−−−−−−−−−−−−−−−−

Questão 1. (pontuação: 2)

A figura abaixo mostra as semirretas perpendiculares r e s, três circunferências pequenas cada uma com raio igual
a 1 e uma circunferência grande de centro O. Uma das circunferências pequenas é tangente a r e a s, cada uma das
outras duas é tangente a ela e a uma das semirretas, e a circunferência grande é tangente às semirretas e a duas das
circunferências pequenas.

Calcule o raio da circunferência grande.

Questão 2. (pontuação: 2)

No triângulo ABC a bissetriz do ângulo BAC encontra o lado BC em D.


DB AB
a) Prove que DC = AC (teorema da bissetriz interna).
b) Use o teorema acima e a figura abaixo para calcular a tangente de 15o .

15o x
15o
MA13 Geometria I - Avaliação 3 - 2012/2

Questão 3. (pontuação: 3)

O losango ABCD tem lado 3 e ângulo  = 60o . Os pontos M , N , P e Q pertencem aos lados AB, BC, CD e DA,
respectivamente e são tais que AM = BN = CP = DQ = 1 .

a) Justifique, de forma breve, porque o quadrilátero M N P Q é um paralelogramo.

b) Calcule a área do quadrilátero M N P Q.

c) Calcule a distância entre os pontos M e P .

Questão 4. (pontuação: 1)

O icosaedro regular é o poliedro formado por 20 faces triangulares equiláteras. Determine quantas diagonais do
icosaedro não passam pelo seu centro.

Questão 5. (pontuação: 2)

Considere o paralelepı́pedo retângulo de bases ABCD e EF GH e com arestas laterais AE, BF , CG e DH. As
medidas são AB = 6, AD = AE = 4 e M é o ponto médio da aresta EF . São feitas as seções pelos planos M HA e
M BG. Retirando-se os tetraedros EM HA e F M BG resulta o poliedro P.

a) Faça um desenho do poliedro P e calcule seu volume.

b) Determine o cosseno do ângulo entre as retas AH e M G.


GABARITO MA13 Geometria I - Avaliação 3 - 2012/2

A área de um triângulo ABC será denotada por (ABC).

Questão 1. (pontuação: 2)

A figura abaixo mostra as semirretas perpendiculares r e s, três circunferências pequenas cada uma com raio igual
a 1 e uma circunferência grande de centro O. Uma das circunferências pequenas é tangente a r e a s, cada uma das
outras duas é tangente a ela e a uma das semirretas, e a circunferência grande é tangente às semirretas e a duas das
circunferências pequenas.

Calcule o raio da circunferência grande.

Uma solução:
Na figura a seguir, A é o centro da circunferência pequena que tangencia r e a circunferência grande, OT é
perpendicular a r e a reta BC passa por A e é paralela a r.

s R

C B
A
r T
Seja R o raio da circunferência grande. No triângulo retângulo ABO temos OA = R+1, OB = R−1 e AB = R−3.
O teorema de Pitágoras conduz à equação R2 − 10R + 9 = 0 cujas raı́zes são 1 e 9.
Devido às caracterı́sticas do problema, a menor raiz é o raio da circunferência pequena tangente às duas semirretas
e a maior raiz é o raio da circunferência grande.
O raio da circunferência grande é igual a 9.

Questão 2. (pontuação: 2)

No triângulo ABC a bissetriz do ângulo BAC encontra o lado BC em D.


DB AB
a) Prove que DC = AC (teorema da bissetriz interna).

b) Use o teorema acima e a figura abaixo para calcular a tangente de 15o .

15o x
15o

Uma solução:
a) Se dois triângulos têm mesma altura, então a razão entre suas áreas é igual à razão entre suas bases. Assim,

(ADB) DB
=
(ADC) DC

E d
d
B D C
Sejam DE e DF perpendiculares a AB e AC como na figura anterior. Como todo ponto da bissetriz de um ângulo
equidista dos lados desse ângulo, então DE = DF = d. Assim,

(ADB) (1/2).AB.d AB DB AB
= = , portanto = , c.q.d.
(ADC) (1/2).AC.d AC DC AC

b) Como o triângulo ABC da figura é retângulo em B e tem ângulo B ÂC = 30o e hipotenusa AC = 2, então

BC = 1 e AB = 3.

1–x
2
D
15o x
15o
A 3 B


DB AB x 3
O teorema da bissetriz interna aplicado a esse triângulo fornece: DC = AC , ou seja, 1−x = 2 .
Aplicando propriedades das proporções podemos escrever:

x 1−x x + (1 − x) 1
√ = = √ = √ .
3 2 3+2 2+ 3
Porém, observando o triângulo ABD, vemos que √x3 é a tangente do ângulo de 15o . Assim,

1 √
tan15o = √ = 2 − 3.
2+ 3

Questão 3. (pontuação: 3)

O losango ABCD tem lado 3 e ângulo  = 60o . Os pontos M , N , P e Q pertencem aos lados AB, BC, CD e DA,
respectivamente e são tais que AM = BN = CP = DQ = 1 .

a) Justifique, de forma breve, porque o quadrilátero M N P Q é um paralelogramo.

b) Calcule a área do quadrilátero M N P Q.

c) Calcule a distância entre os pontos M e P .


Uma solução:

D
Q
P
A C
M
N
B

a) Os triângulos AM Q e CP N são congruentes (caso LAL). Daı́, M Q = N P . Os triângulos BN M e DQP são con-
gruentes (caso LAL); daı́, M N = QP . Assim, o quadrilátero M N P Q possui dois pares de lados opostos congruentes.
Logo, é um paralelogramo.
√ √
3 3
b) (AM Q) = 12 .AM.AQ.sen 60o = 12 .1.2. 2 = 2 .
Como os ângulos de 60o e 120o possuem mesmo seno, concluı́mos que os triângulos AM Q, BN M , CP N e DQP

3
possuem todos a mesma área, igual a 2 . √ √
3 9 3
A área do losango é igual a AB.AD.sen 60o = 3.3. 2 = 2 .
A área do paralelogramo é √ √ √
9 3 3 5 3
S= −4 = .
2 2 2

c) Seja R o ponto médio de P D. Como AM é paralelo a DR e ambos têm comprimento 1 então AM RD é um


paralelogramo e M R = AD = 3. Além disso, RP = 1 e M R̂P = 120o .

D
R
P
A C
M

No triângulo M RP a lei dos cossenos fornece:

1 √
M P 2 = M R2 + RP 2 − 2.M R.RP.cos 120o = 32 + 12 − 2.3.1.(− ) = 13 =⇒ M P = 13.
2
Questão 4. (pontuação: 1)

O icosaedro regular é o poliedro formado por 20 faces triangulares equiláteras. Determine quantas diagonais do
icosaedro não passam pelo seu centro.

Uma solução:

O icosaedro possui 20 faces triangulares. Como cada aresta é lado de exatamente duas faces, o número de arestas
20.3
do icosaedro é A = 2 = 30.

O número de vértices pode ser calculado pela relação de Euler V − A + F = 2, de onde V = 12.
Cada segmento que une dois vértices do icosaedro ou é aresta ou é diagonal. Assim, denotando por CV2 o número
de escolhas de subconjuntos com dois elementos do conjunto de todos os vértices do icosaedro, o número de diagonais
do icosaedro é

12!
2
D = CV2 − A = C12 −A= − 30 = 66 − 30 = 36
10!2!

O icosaedro possui 6 pares de vértices diametralmente opostos e cada diagonal que une dois vértices diametralmente
opostos passa pelo centro do icosaedro. Essas são as únicas diagonais que passam pelo centro. Então, o número de
diagonais que não passam pelo centro é 36 − 6 = 30.

Questão 5. (pontuação: 2)

Considere o paralelepı́pedo retângulo de bases ABCD e EF GH e com arestas laterais AE, BF , CG e DH. As
medidas são AB = 6, AD = AE = 4 e M é o ponto médio da aresta EF . São feitas as seções pelos planos M HA e
M BG. Retirando-se os tetraedros EM HA e F M BG resulta o poliedro P.

a) Faça um desenho do poliedro P e calcule seu volume.

b) Determine o cosseno do ângulo entre as retas AH e M G.


Uma solução:

H
G
E M
F
D
C
A
B

O desenho de P está acima.


Se pensarmos o tetraedro EM HA com base EM H e altura EA, podemos calcular seu volume do seguinte modo:

1 EM.EH 1 3.4
v=. .EA = . .4 = 8
3 2 3 2
O tetraedro F M BG tem também volume v = 8 porque é congruente com EM HA.
O volume de P é o volume do paralelepı́pedo subtraı́do dos volumes dos tetraedros, ou seja,

V = 6.4.4 − 2.8 = 96 − 16 = 80

b) O ângulo entre as retas AH e M G é o ângulo entre BG e M G, ou seja, o ângulo θ = B ĜM . Como F M = 3 e



F B = F G = 4 temos, pelo teorema de Pitágoras, BG = 4 2 e M B = M G = 5.

H
G
5
M θ
E
F 22 

5 N
D
C
A
B

O triângulo M BG é isósceles. Então, assinalando o ponto N médio do lado BG, temos que M N é perpendicular
a BG. Assim, no triângulo M N G,

NG 2 2
cos θ = = .
MG 5
MA13
-
2013
MA13 - AV3 - Avaliação Final 1o semestre - 2013

Questão 1. (pontuação: 2)

No triângulo isósceles ABC tem-se AB = AC. Os pontos M , N e P dos lados AB, BC e CA são tais que P M = P N .
Sendo P M̂ A = α, N P̂ C = β e M N̂ B = θ mostre que

α+β
θ=
2

Questão 2. (pontuação: 2)

Considere o triângulo ABC, retângulo em A, sendo BC = a e AC = b. Seja K1 a circunferência de centro C que


passa por A. A circunferência K2 tem centro P sobre o lado BC, é tangente externamente à K1 e é tangente ao lado
AB.

a) (1,0) Descreva como se pode determinar com régua e compasso o ponto P .

b) (1,0) Determine o raio da circunferência K2 em função de a e b.

Obs.: os itens acima podem ser resolvidos de maneira independente.

Questão 3. (pontuação: 2)

A figura a seguir mostra duas semicircunferências com mesmo centro O e com raios OD = r e OA = 2r. Na semi-
circunferência maior foi assinalado um ponto B e ângulo AÔB mede α radianos. O raio OB cortou a circunferência
menor em C e a região R é a que está sombreada (delimitada pelo arco AB, segmento BC, arco CD e segmento
DA) na figura.

a) (1,0) Calcule o perı́metro de R em função de r e α.

b) (1,0) Calcule a área de R em função de r e α.


Questão 4. (pontuação: 2)

A aresta da base de uma pirâmide reta de base quadrada mede 2 unidades e a esfera inscrita nessa pirâmide tem
raio r (0 < r < 1).

D
C
A
B

a) (1,0) Calcule o volume da pirâmide em função de r.

b) (1,0) Se, para cada valor de r (0 < r < 1 ), o volume da pirâmide é V (r), faça um esboço do gráfico dessa
função.

Questão 5. (pontuação: 2)

Um copo de plástico rı́gido e espessura muito fina tem a forma de um tronco e cone com 8 cm de diâmetro na
boca, 6 cm de diâmetro no fundo e 12 cm de altura.

a) (1,0) Determine um valor aproximado para o volume do copo (ou seja, o número inteiro de cm3 que melhor
aproxima o volume).

b) (1,0) Determine um valor aproximado para a área externa total do copo (ou seja, o número inteiro de cm2 que
melhor aproxima a área).
GABARITO MA13 - Avaliação Final 1o semestre - 2013

Questão 1. (pontuação: 2)

No triângulo isósceles ABC tem-se AB = AC. Os pontos M , N e P dos lados AB, BC e CA são tais que P M = P N .
Sendo P M̂ A = α, N P̂ C = β e M N̂ B = θ mostre que

α+β
θ=
2

Uma solução:

Sejam AB̂C = AĈB = x e P M̂ N = P N̂ M = y.


O ângulo AM̂ N é externo do triângulo M BN . Logo, α + y = x + θ.
O ângulo B N̂ P é externo do triângulo P N C. Logo, β + x = y + θ.
α+β
Somando, temos α + β = 2θ, ou seja, θ = 2 , cqd.

Questão 2. (pontuação: 2)

Considere o triângulo ABC, retângulo em A, sendo BC = a e AC = b. Seja K1 a circunferência de centro C que


passa por A. A circunferência K2 tem centro P sobre o lado BC, é tangente externamente à K1 e é tangente ao lado
AB.

a) (1,0) Descreva como se pode determinar com régua e compasso o ponto P .

b) (1,0) Determine o raio da circunferência K2 em função de a e b.

Obs.: os itens acima podem ser resolvidos de maneira independente.


Uma solução:
a)

Seja D o ponto onde K1 corta BC. A perpendicular a BC por D é tangente a K1 e corta AB em E. A bissetriz
do ângulo DÊB corta BC em P . De fato, pela construção acima, P é equidistante das retas ED e EB. Logo, a
circunferência de centro P que passa por D é tangente a K1 e ao lado AB.

b)

Tracemos P G, perpendicular a AB e P F perpendicular a AC como na figura acima. Sejam P D = P G = x. Da


semelhança dos triângulos CF P e CAB temos:

b−x b
=
b+x a
Assim,

b(a − b)
x=
a+b
.
Questão 3. (pontuação: 2)

A figura a seguir mostra duas semicircunferências com mesmo centro O e com raios OD = r e OA = 2r. Na semi-
circunferência maior foi assinalado um ponto B e ângulo AÔB mede α radianos. O raio OB cortou a circunferência
menor em C e a região R é a que está sombreada (delimitada pelo arco AB, segmento BC, arco CD e segmento
DA) na figura.

a) (1,0) Calcule o perı́metro de R em função de r e α.

b) (1,0) Calcule a área de R em função de r e α.

Uma solução:
a) O comprimento de um arco de circunferência é igual a medida do ângulo central em radianos multiplicada pelo
raio. Assim o perı́metro de R é:

P = 3r + α.2r + r + (π − α)r = (α + π + 4)r

αr 2
b) A área de um setor de ângulo central α em radianos em um cı́rculo de raio r é 2 . Assim a área da região R é:

α(2r)2 (π − α)r2 (3α + π)r2


A= + =
2 2 2

Questão 4. (pontuação: 2)

A aresta da base de uma pirâmide reta de base quadrada mede 2 unidades e a esfera inscrita nessa pirâmide tem
raio r (0 < r < 1).
E

D
C
A
B

a) (1,0) Calcule o volume da pirâmide em função de r.

b) (1,0) Se, para cada valor de r (0 < r < 1 ), o volume da pirâmide é V (r), faça um esboço do gráfico dessa
função.

Uma solução:

a)

Seja O o centro da base ABCD da pirâmide de vértice E como mostra a figura ao lado. Seja M o ponto médio
da aresta BC. Seja K o ponto sobre a altura OE o centro da esfera inscrita na pirâmide. Assim, traçando KT
perpendicular à face EBC temos KO = KT = r. Seja h = OE a altura da pirâmide. Da semelhança dos triângulos
ET K e EOM temos

KT KE
=
OM EM
ou seja,

r h−r
=√
1 h2 + 1
2r
Dessa relação determinamos a altura da pirâmide h = 1−r 2 .
O volume da pirâmide é
1 2 2r 8r
V = .2 . =
3 1 − r2 3(1 − r2 )
.
b) A função que associa r e V é crescente. Quando r se aproxima de 0, temos que V se aproxima de 0. Porém,
quando r se aproxima 1 temos que V tende a +∞. Logo, o gráfico de V (r) tem o seguinte aspecto:

Questão 5. (pontuação: 2)

Um copo de plástico rı́gido e espessura muito fina tem a forma de um tronco e cone com 8 cm de diâmetro na
boca, 6 cm de diâmetro no fundo e 12 cm de altura.

a) (1,0) Determine um valor aproximado para o volume do copo (ou seja, o número inteiro de cm3 que melhor
aproxima o volume).

b) (1,0) Determine um valor aproximado para a área externa total do copo (ou seja, o número inteiro de cm2 que
melhor aproxima a área externa).

Uma solução:
a) Os raios das bases são 4 cm, e 3 cm e a fórmula do volume do tronco de cone fornece o resultado:

πh π.12
V = .(R2 + r2 + Rr) = (16 + 9 + 12) ≈ 3, 14.4.37 ≈ 465 cm3
3 3

b) A geratriz do tronco de cone é igual a 122 + 1 que é aproximadamente igual a 12. Nesse copo, a altura é quase
igual à geratriz.
A área do copo é a soma da área lateral com a área da base, isto é:

A = π(R + r)g + πr2 ≈ 3, 14.[(4 + 3).12 + 32 ] ≈ 292 cm2


MA14
AVALIAÇÕES

MA14

2011-2013
MA14
-
2011
MA14 – Aritmética I – Avaliação 1 – 2011

Questão 1.

(1,0) (a) Determine o maior número natural que divide todos os produtos de três números naturais consecutivos.

(1,0) (b) Responda à mesma questão no caso do produto de quatro números naturais consecutivos.

Em ambos os itens, justifique a sua resposta.

Questão 2.

(1,0) (a) Determine os possı́veis restos da divisão de a3 por 7, onde a é um número natural.

(1,0) (b) Prove que se a e b são naturais e a3 + 2b3 é divisı́vel por 7, então a e b são divisı́veis por 7.

Questão 3.

(1,0) (a) Determine todos os valores possı́veis para (n + 1, n2 + 4).

(1,0) (b) Sabendo que o resto da divisão de n por 5 não é 4, determine [n + 1, n2 + 4].

Questão 4.

(1,5) Determine todos os números naturais que, quando divididos por 18, deixam resto 6 e, quando divididos por 14,
deixam resto 4.

Questão 5.
Sejam p e q dois números naturais, com 1 < p < q e (p, q) = 1. Sabemos que existem números naturais não nulos
u e v tais que up − vq = 1.

(1,0) (a) Mostre que existem dois números naturais p1 e q1 , não nulos, com p1 < p tais que q1 p − p1 q = 1. Conclua
que (p1 , q1 ) = 1 e que q1 < q. Sugestão: Divida v por p, usando o algoritmo da divisão, para encontrar p1 .

(0,5) (b) Mostre que n1 = qq1 é tal que


p 1 p1
= + .
q n1 q1
Conclua que p1 < q1 .

(1,0) (c) Prove que para quaisquer números naturais p e q com 1 < p < q e com (p, q) = 1, existe um número natural
r > 0 e números naturais n1 > n2 > · · · > nr > 1 tais que
p 1 1 1
= + + ··· + .
q n1 n2 nr
AV1 - MA 14 - 2011

Questão 1.

(1,0) (a) Determine o maior número natural que divide todos os produtos de três números naturais consecutivos.

(1,0) (b) Responda à mesma questão no caso do produto de quatro números naturais consecutivos.

Em ambos os itens, justifique a sua resposta.

DUAS SOLUÇÕES

Uma solução

(a) Sendo 1 · 2 · 3 = 6, segue-se que o maior número natural que divide todo produto de três naturais consecutivos
é um divisor de 6. Vamos mostrar que é exatamente 6. De fato, dados três inteiros consecutivos, exatamente um é
múltiplo de 3 e pelo menos um é múltiplo de 2, logo o seu produto é múltiplo de 6. Isto mostra que o maior natural
que divide o produto de quaisquer três naturais consecutivos é 6.

(b) Sendo 1 · 2 · 3 · 4 = 24, segue-se que o maior número natural que divide todo produto de quatro naturais
consecutivos é um divisor de 24. Vamos mostrar que este número é exatamente 24. De fato, dado o produto de
4 números consecutivos a( a + 1)( a + 2)( a + 3), pelo menos um desses é múltiplo de 3. Por outro lado, um deles é
múltiplo de 4. Digamos que seja a o múltiplo de 4, logo a + 2 é par. Se a + 1 é o múltiplo de 4, então a + 3 é par. Se
a + 2 é o múltiplo de 4, então a é par. Se a + 3 é o múltiplo de 4, então a + 1 é par. Isto mostra que n é múltiplo de
3 · 4 · 2 = 24, o que prova o resultado.

Outra solução

Mais geralmente, sabemos da Combinatória que é natural o número

a+r−1 ( a + r − 1) ! ( a + r − 1)( a + r − 2) · · · ( a + 1) a
 
= = ,
r ( a − 1)!r! r!

para quaisquer a, r ∈ N \ {0}. Logo o produto dos r números consecutivos ( a + r − 1)( a + r − 2) · · · ( a + 1) a é


divisível por r!. E, quando a = 1, esse é o máximo divisor possível.

1
AV1 - MA 14 - 2011

Questão 2.

(1,0) (a) Determine os possíveis restos da divisão de a3 por 7, onde a é um número natural.

(1,0) (b) Prove que se a e b são naturais e a3 + 2b3 é divisível por 7, então a e b são divisíveis por 7.

UMA SOLUÇÃO

(a) Podemos escrever a = 7k + r, onde r = 0, 1, 2, 3, 4, 5, 6. Temos que

a3 = (7k + r )3 = 7(72 k3 + 3 · 7k2 r + 3kr2 ) + r3 .

Portanto, o resto da divisão de a3 por 7 é igual ao resto da divisão de r3 por 7. Como 03 = 0 · 7 + 0, 13 = 0 · 7 + 1,


23 = 8 = 1 · 7 + 1, 33 = 27 = 3 · 7 + 6, 43 = 64 = 9 · 7 + 1, 53 = 125 = 17 · 7 + 6, 63 = 216 = 30 · 7 + 6, segue-se que
os possíveis restos de r3 por 7 são 0, 1 ou 6. Além disso, o único caso em que o resto de r3 por 7 dá zero é quando
r = 0, isto é, quando a é múltiplo de 7. Então a3 múltiplo de 7 implica a múltiplo de 7.

(b) Os possíveis restos da divisão de 2b3 por 7 são os possíveis restos de 2r3 por 7, em que r é o resto da divisão
de b por 7. Como os possíveis restos de r3 são 0, 1 e 6, pelo item anterior, multiplicamos por 2 cada um deles
(2 · 0 = 0 · 7 + 0, 2 · 1 = 0 · 7 + 2 e 2 · 6 = 12 = 1 · 7 + 5) e concluímos que os possíveis restos de 2r3 por 7 são 0, 2 ou
5.
Observamos também que se 2b3 é múltiplo de 7, então o resto de 2r3 por 7 é zero, e isso só ocorre se o resto de r3
por 7 é zero, que só ocorre quando o resto de b3 por 7 é zero, que é o mesmo que ter b3 múltiplo de 7, que, pelo item
anterior, é o mesmo que ter b múltiplo de 7. Ou seja, 2b3 múltiplo de 7 implica b múltiplo de 7.
Somando as três possibilidades de restos de a3 com as três possibilidades de restos de 2b3 , num total de nove
possibilidades (0 + 0 = 0, 0 + 1 = 1, 0 + 5 = 5, 1 + 0 = 1, 1 + 2 = 3, 1 + 5 = 6, 6 + 0 = 6, 6 + 2 = 1 · 7 + 1,
6 + 5 = 11 = 1 · 7 + 4), a única que dá resto zero é quando a3 e 2b3 têm ambos resto zero, ou seja, quando a3 e 2b3
são múltiplos de 7, ou seja, quando a e b são múltiplos de 7. Concluímos que a3 + 2b3 é múltiplo de 7 se e somente
se a e b são ambos múltiplos de 7.

2
AV1 - MA 14 - 2011

Questão 3.

(1,0) (a) Determine todos os valores possíveis para (n + 1, n2 + 4).

(1,0) (b) Sabendo que o resto da divisão de n por 5 não é 4, determine [n + 1, n2 + 4].

UMA SOLUÇÃO

(a) Temos
(n + 1, n2 + 4) = (n + 1, n2 + 4 − (n − 1)(n + 1)) = (n + 1, 5) .

Daí segue-se que (n + 1, n2 + 4) só pode ser igual a 1 ou a 5.

(b) Se n não deixa resto 4 quando dividido por 5, temos que n + 1 não é múltiplo de 5. Neste caso, (n + 1, n2 + 4) = 1.
Logo,
[n + 1, n2 + 4] = (n + 1)(n2 + 4) .

3
AV1 - MA 14 - 2011

Questão 4.

(1,5) Determine todos os números naturais que, quando divididos por 18, deixam resto 6 e, quando divididos por
14, deixam resto 4.

UMA SOLUÇÃO

Temos que x = 18v + 6 e x = 14u + 4. Igualando, temos 14u − 18v = 2. Esta equação é equivalente a 7u − 9v = 1.
A menor solução particular é u0 = 4 e v0 = 3. Portanto, a solução geral é dada por u = 4 + 9t e v = 3 + 7t, com
t ∈ N. Daí segue-se que x = 60 + 126t, t ∈ N.

4
AV1 - MA 14 - 2011

Questão 5.
Sejam p e q dois números naturais, com 1 < p < q e ( p, q) = 1. Sabemos que existem números naturais não nulos
u e v tais que up − vq = 1.

(1,0) (a) Mostre que existem dois números naturais p1 e q1 , não nulos, com p1 < p tais que q1 p − p1 q = 1. Conclua
que ( p1 , q1 ) = 1 e que q1 < q. Sugestão: Divida v por p, usando o algoritmo da divisão, para encontrar p1 .

(0,5) (b) Mostre que n1 = qq1 é tal que


p 1 p
= + 1.
q n1 q1
Conclua que p1 < q1 .

(1,0) (c) Prove que para quaisquer números naturais p e q com 1 < p < q e com ( p, q) = 1, existe um número
natural r > 0 e números naturais n1 > n2 > · · · > nr > 1 tais que

p 1 1 1
= + +···+ .
q n1 n2 nr

UMA SOLUÇÃO

(a) Sejam dados u e v tais que up − vq = 1. Pela divisão euclidiana, temos que v = pb + p1 , com 0 < p1 < p, logo
(u − bq) p − p1 q = 1. Ponhamos u − bq = q1 , logo q1 p − p1 q = 1. Daí conclui-se que p1 e q1 são primos entre si. Por
outro lado, temos necessariamente que q1 < q, pois q1 p = p1 q + 1 < pq.

(b) Temos que


p 1 p
= + 1.
q qq1 q1
p p1
Como q < 1, segue-se que q1 < 1, o que mostra que p1 < q1 .

p1
(c) Aplique o resultado do ítem b) à fração q1 , etc. Como p > p1 > p2 > · · · > 0, para um certo r ter-se-á pr = 1
e daí o procedimento para. As desigualdades n1 > n2 > · · · > nr seguem de n1 = qq1 , n2 = q1 q2 , etc. e de
qq1 > q1 q2 > · · · .

5
MA14 – Aritmética I – Avaliação 2 – 2011

Questão 1.

(1,5) Sejam a e b dois números naturais tais que (a, b) = pq, em que p e q são dois números primos distintos. Quais
são os possı́veis valores de
(a) (a2 , b)?
(b) (a3 , b)?
(c) (a2 , b3 )?

Questão 2.

(2,0) Ache o resto da divisão por 17 do número

S = 116 + 216 + 316 + · · · + 8516 .

Questão 3.
 
2357
(1,5) É possı́vel repartir exatamente objetos entre 49 pessoas?
528
MA14 – Aritmética I – Avaliação 2 – 2011

Questão 4.

(2,0) Dispomos de uma quantia de x reais menor do que 3.000. Se distribuirmos essa quantia entre 11 pessoas, sobra
um real; se a distribuirmos entre 12 pessoas, sobram dois reais, e se a distribuirmos entre 13 pessoas, sobram
3 reais. De quantos reais dispomos?
Sugestão: Pode ser útil utilizar o seguinte fato: c é solução da congruência ay ≡ b mod m se, e somente se, c é
solução da congruência ry ≡ b mod m, onde r é o resto da divisão de a por m.

Questão 5.

(1,0) Sabendo que 74 = 2401, ache os algarismos da dezena e da unidade do número 799999 .

Questão 6.
Considere Zm para m > 2.

(0,5) (a) Mostre que Zm tem sempre um número par de elementos invertı́veis. Sugestão: Analise a paridade de ϕ(m),
quando m > 2.

(0,5) (b) Mostre que se [a] é invertı́vel em Zm , então −[a] = [m − a] é invertı́vel e [a] 6= −[a].

(0,5) (c) Mostre que a soma de todos os elementos invertı́veis de Zm é igual a 0.

(0,5) (d) Mostre que a soma de todos os elementos de um sistema reduzido qualquer de resı́duos módulo m é sempre
múltiplo de m.

Observação: em cada item, pode-se usar a afirmação cuja demonstração é pedida em um item anterior sem necessa-
riamente tê-la demonstrado.
AV2 - MA 14 - 2011

Questão 1.

(1,5) Sejam a e b dois números naturais tais que ( a, b) = pq, em que p e q são dois números primos distintos. Quais
são os possíveis valores de
(a) ( a2 , b)?
(b) ( a3 , b)?
(c) ( a2 , b3 )?

UMA SOLUÇÃO

Suponhamos que a = pr qs c e b = pu qv d, onde c e d são primos entre si e também com p e q. A hipótese ( a, b) = pq


implica que min{r, u} = 1 e min{s, v} = 1.

(a) a2 = p2r q2s c2 , onde c2 é primo com p, q e d. Logo, ( a2 , b) = pmin{2r,u} qmin{2s,v} . Tanto min{2r, u} como min{2s, v}
podem e só podem assumir os valores 1 e 2. Portanto, são possíveis ( a2 , b) = pq, ( a2 , b) = p2 q, ( a2 , b) = pq2 , ou
( a2 , b ) = p2 q2 .

(b) ( a3 , b) = pl qt , com l = min{3r, u} e t = min{3s, v}. Logo, l ∈ {1, 2, 3} e t ∈ {1, 2, 3}.

(c) ( a2 , b3 ) = pl qt , com l ∈ {2, 3} e t ∈ {2, 3}.

1
AV2 - MA 14 - 2011

Questão 2.

(2,0) Ache o resto da divisão por 17 do número

S = 116 + 216 + 316 + · · · + 8516 .

UMA SOLUÇÃO

Pelo Pequeno Teorema de Fermat temos que


(
16 1, se 17 não divide a
a ≡ mod 17
0, se 17 divide a

Como 85 = 17 × 5, temos que de 1 a 85 há 5 múltiplos de 17 e 85 − 5 = 80 não múltiplos de 17 (i.e., primos com 17),
logo
S ≡ 80 × 1 mod 17 ≡ 12 mod 17.

Portanto, o resto da divisão de S por 17 é 12.

2
AV2 - MA 14 - 2011

Questão 3.

2357
 
(1,5) É possível repartir exatamente objetos entre 49 pessoas?
528

UMA SOLUÇÃO

Temos
2357 2357!
 
a= = .
528 1829!528!
Portanto, o expoente da maior potência de 7 que divide a é dado por

E7 (2357!) − E7 (1829!) − E7 (528!).

Agora

2357 = 7 × 336 + 5, 336 = 7 × 48 + 0, 48 = 7 × 6 + 6.

1829 = 7 × 261 + 2, 261 = 7 × 37 + 2, 37 = 7 × 5 + 2.

528 = 7 × 75 + 3, 75 = 7 × 10 + 5, 10 = 7 × 1 + 3.

Assim,

E7 (2357!) = 336 + 48 + 6 = 390,

E7 (1829!) = 261 + 37 + 5 = 303 e

E7 (528!) = 75 + 10 + 1 = 86.

Logo,

E7 (2357!) − E7 (1829!) − E7 (528!) = 390 − 303 − 86 = 1.

Portanto, 49 = 72 não divide a e a resposta do problema é não.

3
AV2 - MA 14 - 2011

Questão 4.

(2,0) Dispomos de uma quantia de x reais menor do que 3000. Se distribuirmos essa quantia entre 11 pessoas, sobra
um real; se a distribuirmos entre 12 pessoas, sobram dois reais, e se a distribuirmos entre 13 pessoas, sobram
3 reais. De quantos reais dispomos?
Sugestão: Pode ser útil utilizar o seguinte fato: c é solução da congruência ay ≡ b mod m se, e somente se, c é
solução da congruência ry ≡ b mod m, onde r é o resto da divisão de a por m.

UMA SOLUÇÃO

O número x de Reais é uma solução do seguinte sistema de congruências:



 X ≡ 1 mod 11


X ≡ 2 mod 12

 X ≡ 3 mod 13

Com as notações do Teorema Chinês dos Restos, temos N = 11 × 12 × 13 = 1716, N1 = 12 × 13 = 156, N2 =


11 × 13 = 143 e N3 = 11 × 12 = 132. Precisamos determinar uma solução do sistema:

 N1 Y1 ≡ 1 mod 11


N2 Y2 ≡ 1 mod 12

 N Y ≡ 1 mod 13

3 3

Utilizando a sugestão, podemos resolver o sistema:



 2Y1 ≡ 1 mod 11


11Y2 ≡ 1 mod 12

 2Y ≡ 1 mod 13

3

que possui a solução (y1 , y2 , y3 ) = (6, 11, 7) (achada por inspeção). Assim, as soluções do sistema de congruências
são da forma

x ≡ N1 × y1 × 1 + N2 × y2 × 2 + N3 × y3 × 3 = 156 × 6 × 1 + 143 × 11 × 2 + 132 × 7 × 3 = 6854 mod 1716.

A menor solução é dada pelo resto da divisão de 6854 por 1716 que é 1706. A próxima solução é 1706 + 1716 = 3422,
que ultrapassa 3000. Portanto, a solução procurada é 1706.

Outra solução. Usando-se números negativos pode-se perceber, por inspeção, que −10 é solução do sistema de
congruências. Então basta somar N = 1716 para se obter a primeira solução positiva (igual a 1706) e a seguinte, que
ultrapassa 3000.

Recomendação aos professores. No material da disciplina optou-se pelo estudo de congruências sem a utilização
dos negativos. Pretende-se rever essa decisão para o ano que vem, visto que os negativos são úteis e perfeitamente
naturais na abordagem deste assunto. De qualquer forma, a banca entende que esta solução também deve ser
considerada correta.

4
AV2 - MA 14 - 2011

Questão 5.

(1,0) Sabendo que 74 = 2401, ache os algarismos da dezena e da unidade do número 799999 .

UMA SOLUÇÃO

Efetivamente, precisamos encontrar o resto da divisão de 799999 por 100.

Como 99999 = 4 × 24444 + 3 e 74 ≡ 1 mod 100, temos que

(74 )24444 ≡ 1 mod 100.

Assim,
799999 = (74 )24444 × 73 ≡ 1 × 73 mod 100 ≡ 43 mod 100.

Portanto, os algarismos são 4, da dezena, e 3, da unidade.

5
AV2 - MA 14 - 2011

Questão 6.
Considere Z m para m > 2.

(0,5) (a) Mostre que Z m tem sempre um número par de elementos invertíveis. Sugestão: Analise a paridade de
ϕ(m), quando m > 2.

(0,5) (b) Mostre que se [ a] é invertível em Z m , então −[ a] = [m − a] é invertível e [ a] 6= −[ a].

(0,5) (c) Mostre que a soma de todos os elementos invertíveis de Z m é igual a 0.

(0,5) (d) Mostre que a soma de todos os elementos de um sistema reduzido qualquer de resíduos módulo m é
sempre múltiplo de m.

Observação: em cada item, pode-se usar a afirmação cuja demonstração é pedida em um item anterior sem necessa-
riamente tê-la demonstrado.

UMA SOLUÇÃO

α −1
(a) Se m = p1 1 . . . prαr , então ϕ(m) = p1 1 . . . prαr −1 ( p1 − 1) . . . ( pr − 1), que é obviamente par se m > 2. Como o
α

número de elementos invertíveis de Z m é ϕ(m), o resultado segue.

(b) Se [b] é um inverso de [ a], é imediato ver que (−[ a])(−[b]) = [ a][b] = 1, logo −[ a] é invertível.
Se [ a] é invertível, então ( a, m) = 1. Suponhamos por absurdo que [ a] = −[ a], logo [2a] = 2[ a] = [0], o que implica
que 2a = tm, para algum t ∈ N. Como m divide 2a e (m, a) = 1, segue-se que m divide 2, o que implica que m = 2,
absurdo.

(c) Os elementos invertíveis se apresentam aos pares, um simétrico do outro, a soma é portanto zero.

(d) Se a1 , . . . , a ϕ(m) é um sistema reduzido de resíduos módulo m, temos que [ a1 ], . . . , [ a ϕ(m) ] são os elementos in-
vertíveis de Z m , logo
[ a1 + · · · + a ϕ ( m ) ] = [ a1 ] + · · · + [ a ϕ ( m ) ] = [0],

o que implica que a1 + · · · + a ϕ(m) é um múltiplo de m.

6
MA14 – Aritmética I – Avaliação 3 – 2011

Questão 1.

(1,0) (a) Descreva os números naturais que possuem 15 divisores naturais.

(1,0) (b) Determine o menor número natural com 15 divisores.

Questão 2.

(2,0) Determine a maior potência de 15 que divide 150!

Questão 3.

(2,0) Quando um macaco sobe uma escada de dois em dois degraus, sobra um degrau, quando sobe de três em três
degraus, sobram dois degraus e quando sobe de cinco em cinco degraus, sobram três degraus. Quantos degraus
possui a escada, sabendo que o número de degraus está entre 150 e 200 ?

Questão 4.

(1,0) (a) Determine os elementos invertı́veis de Z24 e mostre que cada um é o seu próprio inverso.

(0,5) (b) Calcule a soma de todos os elementos invertı́veis de Z24 .

(0,5) (c) Calcule o produto de todos os elementos invertı́veis de Z24 .

Questão 5.

(1,0) (a) Seja dado um número natural m = pα αr


1 . . . pr decomposto em fatores irredutı́veis. Seja n um número
1

natural tal que φ(pα n


i ) divide n, para todo i = 1, . . . , r. Mostre que m divide a − 1 para todo número natural
i

a primo com m.

(1,0) (b) Mostre que a12 − 1 é divisı́vel por 4095 sempre que (a, 1365) = 1.
AV3 - MA 14 - 2011

Questão 1.

(1,0) (a) Descreva os números naturais que possuem 15 divisores naturais.

(1,0) (b) Determine o menor número natural com 15 divisores.

UMA SOLUÇÃO

α
Dado o número n cuja decomposição em fatores primos é p1 1 . . . prαr , o número de divisores naturais de n é dado
pela fórmula d(n) = (α1 + 1) · · · (αr + 1).

(a) Se d(n) = 15, temos duas opções:

(i) r = 1 e α1 + 1 = 15, ou

(ii) r = 2, α1 + 1 = 3 e α2 + 1 = 5.

Portanto, temos duas possibilidades: n = p14 , ou n = p2 q4 , com p e q primos distintos.

(b) Os candidatos a menor número natural com 15 divisores naturais são: 214 e 32 24 , sendo o menor deles o número
32 24 .

1
AV3 - MA 14 - 2011

Questão 2.

(2,0) Determine a maior potência de 15 que divide 150!

UMA SOLUÇÃO

Se E3 (150!) = n e E5 (150!) = m, então o expoente da maior potência de 15 que divide 150! é E15 (150!) =
min{n, m}. Vamos determinar E3 (150!) e E5 (150!):

150 = 50 × 3 + 0, 50 = 16 × 3 + 2, 16 = 5 × 3 + 1 e 5 = 1 × 3 + 2,

150 = 30 × 5 + 0, 30 = 6 × 5 + 0, e 6 = 1 × 5 + 1.

Portanto, E3 (150!) = 50 + 16 + 5 + 1 = 72 e E5 (150!) = 30 + 6 + 1 = 37. Consequentemente, E15 (150!) = 37.

2
AV3 - MA 14 - 2011

Questão 3.

(2,0) Quando um macaco sobe uma escada de dois em dois degraus, sobra um degrau, quando sobe de três em
três degraus, sobram dois degraus e quando sobe de cinco em cinco degraus, sobram três degraus. Quantos
degraus possui a escada, sabendo que o número de degraus está entre 150 e 200 ?

UMA SOLUÇÃO

O número x de degraus é solução do seguinte sistema de congruências:



 X ≡ 1 mod 2


X ≡ 2 mod 3

 X ≡ 3 mod 5

Com as notações do Teorema Chinês dos Restos, temos N = 2 × 3 × 5 = 30, N1 = 15, N2 = 10 e N3 = 6. Seja
(y1 , y2 , y3 ) = (1, 1, 1) solução do sistema 
 15Y1 ≡ 1 mod 2


10Y2 ≡ 1 mod 3

 6Y ≡ 1 mod 5.

3

Portanto, toda solução do sistema original é da forma

x ≡ N1 y1 1 + N2 y2 2 + N3 y3 3 mod 30,

ou seja, x ≡ 53 mod 30. Assim, a solução entre 150 e 200 é 53 + 120 = 173.

Outra solução: Como n ≡ 3 mod 5 e n tem que ser ímpar, pois n ≡ 1 mod 2, ficamos apenas com as seguintes
possibilidades: 153, 163, 173, 183, 193. Então excluímos os múltiplos de 3 (153 e 183) e os "múltiplos de 3 +1"(163 e
193). Sobra 173.

3
AV3 - MA 14 - 2011

Questão 4.

(1,0) (a) Determine os elementos invertíveis de Z24 e mostre que cada um é o seu próprio inverso.

(0,5) (b) Calcule a soma de todos os elementos invertíveis de Z24 .

(0,5) (c) Calcule o produto de todos os elementos invertíveis de Z24 .

UMA SOLUÇÃO

(a) Os elementos invertíveis de Z24 são da forma [ a], onde a < 24 e tal que ( a, 24) = 1. Portanto, esses são [1], [5],
[7], [11], [13], [17], [19] e [23]. Agora,

12 ≡ 1 mod 24, 52 = 25 ≡ 1 mod 24, 72 = 2 × 24 + 1 ≡ 1 mod 24,

112 = 5 × 24 + 1 ≡ 1 mod 24, 132 = 7 × 24 + 1 ≡ 1 mod 24,

172 = 12 × 24 + 1 ≡ 1 mod 24, 192 = 15 × 24 + 1 ≡ 1 mod 24,

232 = 22 × 24 + 1 ≡ 1 mod 24.

Logo, [ a]2 = 1, para a = 5, 7, 11, 13, 17, 19, 23.

(b) Temos que


[1] + [5] + [7] + [11] + [13] + [17] + [19] + [23] = [96] = [4 × 24] = [0].

(c) Por outro lado,


[1] × [5] × [7] × [11] × [13] × [17] × [19] × [23] = 1,
pois [5] × [7] = [11], [13] × [17] = [5] e [19] × [23] = [5].

Outra Solução:

(a) Como 13 ≡ −11 mod 24, 17 ≡ −7 mod 24, 19 ≡ −5 mod 24 e 23 ≡ −1 mod 24, então basta verificar a afirmação
em apenas metade dos números.

(b) Segue imediatamente da argumentação acima que soma é zero mod 24.

(c) Como o produto de dois inversos aditivos é −[1], e são quatro pares de elementos mutuamente inversos aditi-
vamente, o produto é (−[1])(−[1])(−[1])(−[1]) = [1].

4
AV3 - MA 14 - 2011

Questão 5.

α
(1,0) (a) Seja dado um número natural m = p1 1 . . . prαr decomposto em fatores irredutíveis. Seja n um número
α
natural tal que ϕ( pi i ) divide n, para todo i = 1, . . . , r. Mostre que m divide an − 1 para todo número natural a
primo com m.

(1,0) (b) Mostre que a12 − 1 é divisível por 4095 sempre que ( a, 1365) = 1.

UMA SOLUÇÃO

αi
α
(a) Como ( a, m) = 1 implica ( a, pi i ) = 1, então o Teorema de Euler garante que a ϕ( pi ≡ 1 mod piαi . Como ϕ( piαi )
)
α α
divide n, então an ≡ 1 mod pi i , para todo i = 1, . . . , r. Mas isso é o mesmo que dizer que an − 1 é múltiplo de pi i ,
∀i = 1, . . . , r. Como os piαi são todos primos entre si, an − 1 é múltiplo de m = p1α1 . . . prαr .

(b) Note que 4095 = 32 · 5 · 7 · 13 e que 1365 = 3 · 5 · 7 · 13. Então ( a, 1365) = 1 implica (de fato, equivale a)
( a, 4095) = 1. Portanto queremos saber se a12 − 1 é múltiplo de 4095, sob a hipótese ( a, 4095) = 1. Pelo item
anterior (com n = 12 e m = 4095), é suficiente verificar se ϕ(32 ), ϕ(5), ϕ(7) e ϕ(13) são divisores de 12. De fato, eles
são: ϕ(32 ) = 6, ϕ(5) = 4, ϕ(7) = 6 e ϕ(13) = 12.

Evidentemente é possível responder a parte (b) sem estar muito ciente de um resultado geral como a parte (a),
essencialmente fazendo as mesmas coisas. Queremos que a12 − 1 seja múltiplo de 4095 e, para tanto, basta que
seja simultaneamente múltiplo de 32 , 5, 7 e 13, pois são primos entre si. Então queremos mostrar as congruências
a12 ≡ 1 mod 9, a12 ≡ 1 mod 5, a12 ≡ 1 mod 7 e a12 ≡ 1 mod 13. A hipótese ( a, 1365) = 1 garante que ( a, 9) =
1, ( a, 5) = 1, ( a, 7) = 1 e ( a, 13) = 1. Com isso o Teorema de Euler garante que aφ(9) = a6 ≡ 1 mod 9, logo
a12 = ( a6 )2 ≡ 1 mod 9. Para os demais casos o Teorema de Euler coincide com o Pequeno Teorema de Fermat:
a4 ≡ 1 mod 5 implica a12 = ( a4 )3 ≡ 1 mod 5; a6 ≡ 1 mod 7 implica a12 = ( a6 )2 ≡ 1 mod 7; e a12 ≡ 1 mod 13 já é o
que queríamos demonstrar.

5
MA14
-
2012
Primeira Prova de Aritmética - MA14
2012/2 semestre

NOME: −−−−−−−−−−−−−−−−−−−−−−−−−−−−−−−−−−−−−−−−−−−−−−−−−−−−−−−−−−−−−−−−−−−−−−−−−−−−−−−−−−−−−

Questão 1 (valor: 2 pontos)


Mostre que na representação na base 10 de um número da forma a5 − a, em que a ∈ N, o algarismo das unidades
é sempre igual a 0.

Questão 2 (valor: 2 pontos)


Mostre que, para todo n ∈ N, é irredutı́vel a fração
21n + 4
.
14n + 3

Questão 3 (valor: 2 pontos)


Denotando por (x, y) e por [x, y], respectivamente, o máximo divisor comum e o mı́nimo múltiplo comum de dois
números naturais x e y, resolva o sistema de equações

 (x, y) = 6


 [x, y] = 60

Questão 4 (valor: 2 pontos)


Uma terna de números primos da forma (a, a + 2, a + 4) é chamada de terna de primos trigêmeos.

a) Mostre que dados três números inteiros a, a + 2 e a + 4, um e apenas um deles é múltiplo de 3.

b) Mostre que a única terna de primos trigêmeos é (3, 5, 7).

Questão 5 (valor: 2 pontos)


Um grupo de 30 pessoas entre homens, mulheres e crianças foram a um banquete e juntos gastaram 30 patacas.
Cada homem pagou 2 patacas, cada mulher meia pataca e cada criança um décimo de pataca. Quantos homens,
quantas mulheres e quantas crianças havia no grupo?
Primeira Prova de Aritmética - MA14
2012/2 semestre

SOLUÇÕES

Questão 1 (valor: 2 pontos)


Mostre que na representação na base 10 de um número da forma a5 − a, em que a ∈ N, o algarismo das unidades
é sempre igual a 0.

Uma solução:

Escrevamos
m = a5 − a = a(a4 − 1) = a(a − 1)(a + 1)(a2 + 1).

Como m possui dois fatores formados por inteiros consecutivos, o número m é par. Agora só falta mostrar que m
é múltiplo de 5.
Escrevamos a = 5k + r, com r = 0, 1, 2, 3 ou 4 e façamos uma análise de casos.

• Caso a = 5k. Como a é um divisor de m, temos que 5 divide m.

• Caso a = 5k + 1. Como a − 1 = 5k é divisor de m, temos que 5 divide m.

• Caso a = 5k + 2. Como a2 + 1 = 25k 2 + 20k + 5, temos que 5 divide m.

• Caso a = 5k + 3. Como a2 + 1 = 25k 2 + 30k + 10, temos que 5 divide m.

• Caso a = 5k + 4. Como a + 1 = 5k + 5, temos que 5 divide m.

Observação: O fato que 5 divide a5 − a decorre imediatamente do Pequeno Teorema de Fermat, que não faz parte
da matéria dada.

Questão 2 (valor: 2 pontos)


Mostre que, para todo n ∈ N, é irredutı́vel a fração
21n + 4
.
14n + 3

Uma solução:

Temos que
(21n + 4, 14n + 3) = (21n + 4 − 14n − 3, 14n + 3) = (7n + 1, 14n + 3)

= (7n + 1, 14n + 3 − 14n − 2) = (7n + 1, 1) = 1.


Questão 3 (valor: 2 pontos)
Denotando por (x, y) e por [x, y], respectivamente, o máximo divisor comum e o mı́nimo múltiplo comum de dois
números naturais x e y, resolva o sistema de equações

 (x, y) = 6


 [x, y] = 60

Uma solução:

Sabemos que x e y são múltiplos de 6 e divisores de 60 e são tais que

xy = (x, y)[x, y] = 6 · 60 = 360.

As possı́veis soluções são: x, y ∈ {6, 12, 30, 60}.


A condição xy = 360 implica que as únicas soluções do sistema são x = 6, y = 60; x = 12, y = 30; x = 30, y = 12
e x = 60, y = 6.

Questão 4 (valor: 2 pontos)


Uma terna de números primos da forma (a, a + 2, a + 4) é chamada de terna de primos trigêmeos.

a) Mostre que dados três números inteiros a, a + 2 e a + 4, um e apenas um deles é múltiplo de 3.

b) Mostre que a única terna de primos trigêmeos é (3, 5, 7).

Uma solução:

a) Podemos escrever a = 3k + r, com r = 0, 1 ou 2.

• Se r = 0, temos a = 3k, a + 2 = 3k + 2 e a + 4 = 3(k + 1) + 1, logo somente a é múltiplo de 3.

• Se r = 1, temos a = 3k + 1, a + 2 = 3(k + 1) e a + 4 = 3(k + 1) + 2, logo somente a + 2 é múltiplo de 3.

• Se r = 2, temos a = 3k + 2, a + 2 = 3(k + 1) + 1 e a + 4 = 3(k + 2), logo somente a + 4 é múltiplo de 3.

b) Dados três primos a, a + 2 e a + 4, um deles é múltiplo de 3, sendo este número primo, ele deve ser 3. Portanto,
a única possibilidade é a = 3, a + 2 = 5 e a + 4 = 7.

Questão 5 (valor: 2 pontos)


Um grupo de 30 pessoas entre homens, mulheres e crianças foram a um banquete e juntos gastaram 30 patacas.
Cada homem pagou 2 patacas, cada mulher meia pataca e cada criança um décimo de pataca. Quantos homens,
quantas mulheres e quantas crianças havia no grupo?

Uma solução:

Seja x o número de homens, y o número de mulheres e z o número de crianças. Logo, x + y + z = 30.


Distribuindo os gastos por grupos, devemos ter 2x + 21 y + 1
10 z = 30.
Assim, temos que resolver o sistema:
  
 x + y + z = 30

  x + y + z = 30

  x + y + z = 30


⇐⇒ ⇐⇒
  
 2x + 1 y + 1
= 30  20x + 5y + z = 300  19x + 4y = 270.
10 z
  
2

Sendo (19, 4) = 1, a equação 19x + 4y = 270 possui solução. A solução mı́nima dessa equação é x0 = 2, y0 = 58.
Portanto, a solução geral é dada por 
 x = 2 + 4t


, t∈N

 y = 58 − 19t

Levando em consideração os intervalos de variação de x, y e z:

0 < x, y, z < 30,

a única solução possı́vel é x = 14, y = 1 e z = 15.


Segunda Prova de Aritmética - MA14

2012/2o semestre

NOME: −−−−−−−−−−−−−−−−−−−−−−−−−−−−−−−−−−−−−−−−−−−−−−−−−−−−−−−−−−−−−−−−−−−−−−−−−−−−−−−−−−−−−

Problema 1 (valor: 2 pontos) Sejam n ∈ Z e p um número primo. Denota-se com Ep (n) o expoente da maior
potência de p que divide n.
a) Justifique a seguinte afirmação sobre dois números naturais m e n:

m = n ⇐⇒ Ep (m) = Ep (n) para todo número primo p.

b) Sejam a, b, c ∈ N. Mostre que

[a, b, c]2 (a, b)(a, c)(b, c) = (a, b, c)2 [a, b][a, c][b, c]

Sugestão: Note que dada a simetria dessa expressão em a, b e c, pode-se supor sem perda de generalidade que
Ep (a) 6 Ep (b) 6 Ep (c).

Problema 2 (valor: 2 pontos) Mostre que

a) 7|32n+1 + 2n+2 b) 37|3 |00 {z


. . . 0} 7
3n

Problema 3 (valor: 2 pontos) Considere os números da forma


10n − 1
αn = .
9
a) Mostre que 9|αn ⇐⇒ 9|n.
b) Mostre que 11|αn ⇐⇒ n é par.

Problema 4 (valor: 2 pontos) Ache todos os números que deixam resto 2, quando divididos por 7, deixam resto 3,
quando divididos por 11 e deixam resto 5, quando divididos por 13. Aponte a menor solução positiva.

Problema 5 (valor: 2 pontos) Seja ϕ a função de Euler que associa a cada número natural m > 1 o número de
inteiros entre 0 e m que são primos com m. Mostre que
a) Se m > 2, então ϕ(m) é par. Conclua, nessas condições, que Zm tem sempre um número par de elementos
invertı́veis.
b) O número de elementos invertı́veis de Z2m é igual ao número de elementos invertı́veis de Zm , se m é ı́mpar, e igual
ao dobro desse número, se m é par.
c) Mostre que o número de elementos invertı́veis de Zm2 é m vezes o número de elementos invertı́veis de Zm .
d) Relacione o número de elementos invertı́veis de Zmr com o número de elementos invertı́veis de Zm , sendo r um
inteiro positivo.
Segunda Prova de Aritmética - MA14
GABARITO
2012/2o semestre

Questão 1
Sejam n ∈ Z e p um número primo. Denota-se com Ep (n) o expoente da maior potência de p que divide n.
a) Justifique a seguinte afirmação sobre dois números naturais m e n:

m = n ⇐⇒ Ep (m) = Ep (n) para todo número primo p.

b) Sejam a, b, c ∈ N. Mostre que

[a, b, c]2 (a, b)(a, c)(b, c) = (a, b, c)2 [a, b][a, c][b, c]

Sugestão: Note que dada a simetria dessa expressão em a, b e c, pode-se supor sem perda de generalidade que
Ep (a) 6 Ep (b) 6 Ep (c).

Uma solução:

a) A menção ao Teorema Fundamental da Aritmética já deveria garantir metade da pontuação do ı́tem.
Se n = m é óbvio que Ep (m) = Ep (n) para todo número primo p. A seguir, provaremos a implicação contrária.
β1
Pelo Teorema Fundamental da Aritmética, podemos escrever m = pα αr βs
1 · · · pr e n = q1 · · · qs , com {p1 , . . . , pr } e
1

{q1 , . . . , qs } dois conjuntos formados por números primos, dois a dois distintos em cada um dos conjuntos anteriores.
Como
{p ; p é primo e Ep (m) > 0} = {p1 , . . . , pr }

e
{p ; p é primo e Ep (n) > 0} = {q1 , . . . , qs },

segue-se que
{p1 , . . . , pr } = {q1 , . . . , qs }.

Assim, r = s e, após reordenar os elementos q1 , . . . , qr , podemos supor qi = pi , para i = 1, . . . , r. Como

αi = Epi (m) = Epi (n) = βi , i = 1, . . . , r,

conclui-se que n = m.

b) Pelo fato desta expressão ser simétrica em a, b e c, podemos supor que Ep (a) 6 Ep (b) 6 Ep (c). Portanto,

Ep [a, b, c]2 (a, b)(a, c)(b, c) = 2Ep (c) + 2Ep (a) + Ep (b). (1)

Por outro lado,



Ep (a, b, c)2 [a, b][a, c][b, c] = 2Ep (a) + Ep (b) + 2Ep (c). (2)

O resultado segue de a), usando-se (1) e (2), já que, para todo p primo,
 
Ep [a, b, c]2 (a, b)(a, c)(b, c) = Ep (a, b, c)2 [a, b][a, c][b, c] .
Questão 2
Mostre que

a) 7|32n+1 + 2n+2 . . . 0} 7
b) 37|3 |00 {z
3n

Uma solução:

a) Como 9 ≡ 2 mod 7, temos que

32n+1 = 3 · (32 )n ≡ 3 · 9n ≡ 3 · 2n mod 7.

Por outro lado, como 2n+2 = 4 · 2n , segue-se que

32n+1 + 2n+2 ≡ 3 · 2n + 4 · 2n = 7 · 2n ≡ 0 mod 7.

b) Temos que
. . . 0} 7 = 3 · 103n+1 + 7 = 3 · 10 · 103n + 7 = 30 · 103n + 7.
3 |00 {z
3n
3
Mas, como 10 ≡ 1 mod 37, temos que
103n = (103 )n ≡ 1 mod 37,

logo
. . . 0} 7 = 30 · 103n + 7 ≡ 37 ≡ 0 mod 37.
3 |00 {z
3n

Questão 3
Considere os números da forma
10n − 1
αn = .
9
a) Mostre que 9|αn ⇐⇒ 9|n.
b) Mostre que 11|αn ⇐⇒ n é par.

Uma solução:

Podemos escrever
10n − 1 10n − 1
αn = = = 10n−1 + · · · + 10 + 1.
9 10 − 1
a) Como 10 ≡ 1 mod 9 temos que

αn = 10n−1 + · · · + 10 + 1 ≡ n mod 9,

logo αn ≡ 0 mod 9 se, e somente se, n ≡ 0 mod 9.

b) Note que 10 ≡ −1 mod 11, logo 10n ≡ (−1)n . Portanto,

αn = 1 + 10 + · · · + 10n−1 ≡ 1 − 1 + 1 · · · + (−1)n−1 mod 11.

Consequentemente, αn ≡ 0 mod 11 se, e somente se, n é par.


Questão 4
Ache todos os números que deixam resto 2, quando divididos por 7, deixam resto 3, quando divididos por 11 e
deixam resto 5, quando divididos por 13. Aponte a menor solução positiva.

Uma solução:

Definindo N = 7 × 11 × 13 = 1001, N1 = 11 × 13 = 143, N2 = 7 × 13 = 91 e N3 = 7 × 11 = 77, temos que resolver


cada uma das congruências: 143Y ≡ 1 mod 7, 91Y ≡ 1 mod 11 e 77Y ≡ 1 mod 13
Como 143 ≡ 3 mod 7, 91 ≡ 3 mod 11 e 77 ≡ 12 ≡ −1 mod 13, essas congruências são equivalentes às seguintes:
3Y ≡ 1 mod 7, 3Y ≡ 1 mod 11 e Y ≡ −1 mod 13, que possuem as seguintes soluções: y1 = 5, y2 = 4 e y3 = 12.
Assim, pelo Teorema Chinês dos Restos, uma solução é dada por

x = N1 y1 × 2 + N2 y2 × 3 + N3 y3 × 5 = 1430 + 1092 + 4620 = 7142.

Essa solução é única módulo N = 1001. Assim, todas as soluções são

7142 + t1001, t ∈ Z.

A solução positiva mı́nima é o resto da divisão de 7142 por 1001, que é igual a 135.

Questão 5
Seja ϕ a função de Euler que associa a cada número natural m > 1 o número de inteiros entre 0 e m que são
primos com m. Mostre que
a) Se m > 2, então ϕ(m) é par. Conclua, nessas condições, que Zm tem sempre um número par de elementos
invertı́veis.
b) O número de elementos invertı́veis de Z2m é igual ao número de elementos invertı́veis de Zm , se m é ı́mpar, e igual
ao dobro desse número, se m é par.
c) Mostre que o número de elementos invertı́veis de Zm2 é m vezes o número de elementos invertı́veis de Zm .
d) Relacione o número de elementos invertı́veis de Zmr com o número de elementos invertı́veis de Zm , sendo r um
inteiro positivo.

Uma solução:

Observe inicialmente que o número de elementos invertı́veis de Zm é igual a ϕ(m). Sabemos que se m = 2r pr11 · · · prss ,
então
ϕ(m) = 2r−1 (2 − 1)pr11 −1 · · · prss −1 (p1 − 1) · · · (ps − 1).
a) Se m > 2, então r > 2 ou s > 1. Em qualquer das duas situações ϕ(m) é par, pois uma das parcelas é par.

b) Segue da seguinte afirmação: (


ϕ(m), se m é ı́mpar
ϕ(2m) =
2ϕ(m), se m é par.
(basta escrever a expressão de ϕ(2m)).
c) é um caso particular de d).
d) Escrevendo a expressão de ϕ(mr ), r > 0, segue imediatamente que

ϕ(mr ) = mr−1 ϕ(m).

Logo, o número de elementos invertı́veis de Zmr é igual a mr−1 vezes o número de elementos invertı́veis de Zm .
Terceira Prova de Aritmética - MA14
2012/2 semestre

NOME: −−−−−−−−−−−−−−−−−−−−−−−−−−−−−−−−−−−−−−−−−−−−−−−−−−−−−−−−−−−−−−−−−−−−−−−−−−−−−−−−−−−−−

Questão 1 (valor total: 2 pontos)

(valor: 0,5) a) Mostre que a soma dos quadrados de dois números ı́mpares nunca é um quadrado.

(valor: 0,5) b) Mostre que todo quadrado perfeito é da forma 5k, 5k + 1 ou 5k + 4.

(valor: 1,0) c) Mostre que se três inteiros verificam a2 = b2 + c2 , então b ou c é par e um dos três números a, b ou c
é múltiplo de 5.

Questão 2 (valor: 2 pontos)

Um grupo de 30 pessoas formado por homens, mulheres e crianças, ganhou numa loteria um prêmio de R$ 30.000, 00
que foi dividido entre elas da seguinte forma: Cada homem recebeu R$ 2.000, 00, cada mulher recebeu R$ 500, 00 e
cada criança recebeu R$ 100, 00. Qual é a quantidade de homens, mulheres e crianças que havia no grupo?

Questão 3 (valor: 2 pontos)

Mostre que
21000 |1001 × 1002 × · · · × 2000,

mas que
21001 6 |1001 × 1002 × · · · × 2000.

Questão 4 (valor: 2 pontos)

Ache o resto da divisão de 15 + 25 + · · · + 1835 por 5.

Questão 5 (valor: 2 pontos)

a) Ache o menor número natural M que é termo comum às seguintes progressões aritméticas:

an = 5n + 1, bn = 7n + 3, cn = 9n + 5,

ou seja, determine o menor número natural M para o qual existem r, s e t tais que ar = bs = ct = M .

b) Encontre os valores dos ı́ndices r, s e t tais que ar = bs = ct = M .


Gabarito da 3a. Prova de Aritmética - MA14
2012/2 semestre

Questão 1 (valor total: 2 pontos)

(valor: 0,5) a) Mostre que a soma dos quadrados de dois números ı́mpares nunca é um quadrado.

(valor: 0,5) b) Mostre que todo quadrado perfeito é da forma 5k, 5k + 1 ou 5k + 4.

(valor: 1,0) c) Mostre que se três inteiros verificam a2 = b2 + c2 , então b ou c é par e um dos três números a, b ou c
é múltiplo de 5.

Uma solução:

a) Se b = 2n + 1 e c = 2m + 1, então b2 + c2 = 4(n2 + n + m2 + m) + 2, que é par mas não múltiplo de 4, logo não


pode ser um quadrado, pois se a2 = b2 + c2 , então 2|a2 , logo 2|a e portanto 4|a2 .

b) Todo número inteiro se escreve de uma das seguintes formas: 5m, 5m + 1, 5m + 2, 5m + 3 ou 5m + 4. Elevando
ao quadrado cada uma dessas formas obtemos

(5m)2 = 25m2 = 5k,


(5m + 1)2 = 25m2 + 10m + 1 = 5k + 1,
(5m + 2)2 = 25m2 + 20m + 4 = 5k + 4,
2
(5m + 3) = 25m2 + 30m + 9 = 5k + 4,
(5m + 4)2 = 25m2 + 40m + 16 = 5k + 1.

c) Pelo ı́tem a), um dos números b ou c tem que ser par.


Por outro lado, se nenhum dos números for múltiplo de 5, então a2 , b2 e c2 não são múltiplos de 5, logo, pelo ı́tem
b), são da forma 5k + 1 ou 5k + 4. Logo a soma b2 + c2 resulta em um número da forma 5k + 2 ou 5k + 3, então
a2 = b2 + c2 é também da forma 5k + 2 ou 5k + 3, o que é uma contradição com o ı́tem b).

Questão 2 (valor: 2 pontos)

Um grupo de 30 pessoas formado por homens, mulheres e crianças, ganhou numa loteria um prêmio de R$ 30.000, 00
que foi dividido entre elas da seguinte forma: Cada homem recebeu R$ 2.000, 00, cada mulher recebeu R$ 500, 00 e
cada criança recebeu R$ 100, 00. Qual é a quantidade de homens, mulheres e crianças que havia no grupo?
Uma solução:

Seja X o número de homens, Y o de mulheres e Z o de crianças. Assim, temos

X + Y + Z = 30.

Por outro lado,


2000X + 500Y + 100Z = 30000,

logo, devemos resolver o sistema


( (
X + Y + Z = 30 X + Y + Z = 30
ou, equivalentemente,
20X + 5Y + Z = 300 19X + 4Y = 270

Uma solução minimal (x, y) da equação 19X + 4Y = 270 é tal que 0 < x 6 3. Testando valores, vemos que (2, 58)
é uma solução particular dessa equação, logo a solução geral é x = 2 + 4t e y = 58 − 19t, com t ∈ N. Portanto, a
solução geral do último sistema é dada por

x = 2 + 4t
y = 58 − 19t
z = 30 − x − y = −30 + 15t.

Assim, a única posibilidade do enunciado do problema estar satisfeito ocorre quando t = 3, logo x = 14, y = 1 e
z = 15.

Questão 3 (valor: 2 pontos)

Mostre que
21000 |1001 × 1002 × · · · × 2000,

mas que
21001 6 |1001 × 1002 × · · · × 2000.

Uma solução:

Como
(2n)!
(n + 1)(n + 2) · · · (2n) = ,
n!
temos que
E2 ((n + 1)(n + 2) · · · (2n)) = E2 ((2n)!) − E2 (n!).

Pelo Teorema de Legendre, temos que


   
2n 2n
E2 ((2n)!) = + + · · · = n + E2 (n!).
2 4

Portanto,
E2 ((n + 1)(n + 2) · · · (2n)) = E2 ((2n)!) − E2 (n!) = n + E2 (n!) − E2 (n!) = n.

O resultado segue tomando n = 1000.


Questão 4 (valor: 2 pontos)

Ache o resto da divisão de 15 + 25 + · · · + 1835 por 5.

Uma solução:

Pelo Pequeno Teorema de Fermat temos que n5 ≡ n mod 5, logo


184 × 183
15 + 25 + · · · + 1835 ≡ 1 + 2 + · · · + 183 = mod 5.
2
Mas,
184 × 183
= 92 × 183 ≡ 2 × 3 ≡ 1 mod 5.
2
Portanto, a resposta é 1.

Questão 5 (valor: 2 pontos)

a) Ache o menor número natural M que é termo comum às seguintes progressões aritméticas:

an = 5n + 1, bn = 7n + 3, cn = 9n + 5,

ou seja, determine o menor número natural M para o qual existem r, s e t tais que ar = bs = ct = M .
b) Encontre os valores dos ı́ndices r, s e t tais que ar = bs = ct = M .

Uma solução:

a) Um termo comum às progressões é solução do sistema



 X≡1 mod 5,


X≡3 mod 7,

 X≡5

mod 9,

Com as notações do Teorema Chinês dos Restos, temos N = 5 × 7 × 9 = 315, logo N1 = 7 × 9 = 63, N2 = 5 × 9 = 45
e N3 = 5 × 7 = 35. Devemos resolver as congruências N1 Y = 63Y ≡ 1 mod 5, N2 Y = 45Y ≡ 1 mod 7 e
N3 Y = 45Y ≡ 1 mod 9. Por inspeção, encontramos as seguintes respectivas soluções: y1 = 2, y2 = 5 e y3 = 8.
Assim, o sistema de congruências possui a única solução

x = N1 y1 × 1 + N2 y2 × 3 + N3 y3 × 5 = 126 + 675 + 1 400 = 2 201

módulo N = 315. A menor solução é dada pelo resto da divisão de x por N que é M = 311.

b) Segue de a) que 
 5r + 1 = 311


7s + 3 = 311

 9t + 5 = 311.

Portanto, temos a62 = b44 = c34 = 311.


MA14
-
2013
Terceira Prova de Aritmética - MA14

2013/1 semestre

NOME: −−−−−−−−−−−−−−−−−−−−−−−−−−−−−−−−−−−−−−−−−−−−−−−−−−−−−−−−−−−−−−−−−−−−−−−−−−−−−−−−−−−−−

Questão 1 (valor: 2 pontos)

a)(1,0) Mostre que se 7|a2 + b2 , sendo a e b são números inteiros, então 7|a e 7|b.

b)(1,0) Resolva a equação diofantina x2 + y 2 = 637, x, y ∈ N.

Sugestão: para a), escreva os números a e b na forma 7m + i, com i = 0, 1, 2, 3, 4, 5, 6.

Questão 2 (valor: 2 pontos) Uma pessoa comprou cavalos e bois. Foram pagos 31 escudos por cavalo e 20 por boi
e sabe-se que todos os bois custaram 7 escudos a mais do que todos os cavalos. Determine quantos cavalos e quantos
bois foram comprados, sabendo que o número de bois está entre 40 e 95.

Questão 3 (valor: 1 ponto) Determine todos os números primos p ∈ N tais que p|3p + 7.

Questão 4 (valor: 2 pontos) Um terno de números primos (p1 , p2 , p3 ) é chamado de terno de primos trigêmeos, se
p3 − p2 = p2 − p1 = 2.

a) (0,5) Mostre que (3, 5, 7) é o único terno de primos trigêmeos.

b) (1,5) Determine todos os números primos p ∈ N que se escrevem ao mesmo tempo como soma de dois primos e
como diferença de dois primos.

Questão 5 (valor: 1 ponto) Ache as raı́zes de X 10 − [1] = 0 em Z11 .

Questão 6 (valor: 2 pontos) Ache a menor quantia em Reais (R$) que quando distribuı́da entre 5 pessoas sobra 1
Real, quando distribuı́da entre 7 pessoas sobram 3 Reais e quando distribuı́da entre 9 pessoas sobram 5 Reais.
Terceira Prova de Aritmética - MA14
GABARITO
2013/1 semestre

NOME: −−−−−−−−−−−−−−−−−−−−−−−−−−−−−−−−−−−−−−−−−−−−−−−−−−−−−−−−−−−−−−−−−−−−−−−−−−−−−−−−−−−−−

Questão 1 (valor: 2 pontos)

a)(1,0) Mostre que se 7|a2 + b2 , sendo a e b são números inteiros, então 7|a e 7|b.

b)(1,0) Resolva a equação diofantina x2 + y 2 = 637, x, y ∈ N.

Sugestão: para a), escreva os números a e b na forma 7m + i, com i = 0, 1, 2, 3, 4, 5, 6.

Questão 2 (valor: 2 pontos) Uma pessoa comprou cavalos e bois. Foram pagos 31 escudos por cavalo e 20 por boi
e sabe-se que todos os bois custaram 7 escudos a mais do que todos os cavalos. Determine quantos cavalos e quantos
bois foram comprados, sabendo que o número de bois está entre 40 e 95.

Questão 3 (valor: 1 ponto) Determine todos os números primos p ∈ N tais que p|3p + 7.

Questão 4 (valor: 2 pontos) Um terno de números primos (p1 , p2 , p3 ) é chamado de terno de primos trigêmeos, se
p3 − p2 = p2 − p1 = 2.

a) (0,5) Mostre que (3, 5, 7) é o único terno de primos trigêmeos.

b) (1,5) Determine todos os números primos p ∈ N que se escrevem ao mesmo tempo como soma de dois primos e
como diferença de dois primos.

Questão 5 (valor: 1 ponto) Ache as raı́zes de X 10 − [1] = 0 em Z11 .

Questão 6 (valor: 2 pontos) Ache a menor quantia em Reais (R$) que quando distribuı́da entre 5 pessoas sobra 1
Real, quando distribuı́da entre 7 pessoas sobram 3 Reais e quando distribuı́da entre 9 pessoas sobram 5 Reais.
Soluções

1. a) Escrevendo um número c na forma 7m+i, i = 0, 1, . . . , 6, temos que c2 = 7(7m2 +2mi)+i2 , logo c2 ≡ i2 mod 7.
Portanto, os possı́veis valores de a2 e b2 módulo 7 são

02 ≡ 0 mod 7
12 ≡ 1 mod 7
22 ≡ 4 mod 7
32 ≡ 2 mod 7
42 ≡ 2 mod 7
52 ≡ 4 mod 7
62 ≡ 1 mod 7

Assim, a única possibilidade para que a2 + b2 ≡ 0 mod 7 é que a2 ≡ b2 ≡ 0 mod 7. Logo, 7|a2 e 7|b2 . Sendo 7
primo, temos que 7|a e 7|b.

b) Se (a, b) é uma solução da equação diofantina, pelo item a) temos que 7|a e 7|b, logo a = 7k e b = 7l. Assim,

637 = a2 + b2 = 49(k 2 + l2 ),

logo k 2 + l2 = 13. Portanto, k = 2 e l = 3 ou k = 3 e l = 2, o que nos dá as soluções

a = 28, b = 21 ou a = 21, b = 28.

2. Chamando b o número de bois e c o número de cavalos, temos que 20b = 31c + 7, logo a equação diofantina a ser
resolvida é 20b − 31c = 7. Por inspeção vemos que b0 = 5 e c0 = 3 (alternativamente, pode-se calcular uma solução
particular usando o algoritmo de Euclides estendido). Assim a solução geral é dada por

b = b0 + t31 = 5 + t31, c0 = c0 + t20 = 3 + t20, t ∈ Z.

Como 40 < b < 100, segue-se que 40 < 5 + t31 < 95, ou seja, t = 2. Portanto, foram comprados 67 bois e 43 cavalos.

3. Pelo Pequeno Teorema de Fermat temos que 3p ≡ 3 mod p, logo 3p + 7 ≡ 10 mod p. Portanto p|3p + 7 se, e
somente se p|10. Como p ∈ N é primo, então p = 2 ou p = 5.

4. a) O terno (3, 5, 7) é um terno de primos trigêmeo. Dados três inteiros a, a + 2, a + 4, um deles é divisı́vel por 3,
isto se vê escrevendo a na forma 3m + i, i = 0, 1, 2.
Portanto, se a, a + 2 e a + 4 são primos, um dos três números é igual a 3, por ser divisı́vel por 3. Portanto, a única
possibilidade é a = 3, a + 2 = 5 e a + 4 = 7.
b) Suponhamos que p = p1 + p2 e p = p4 − p3 , sendo p1 , p2 , p3 , p4 números primos. O primo p = 2 não se escreve
como soma de dois primos. Logo p é ı́mpar, o que implica que p1 ou p2 é par e o outro é ı́mpar, o mesmo ocorrendo
para p3 e p4 .
Trocando-se p1 e p2 de posição se necessário, vemos que a única possibilidade é p1 = 2 e p3 = 2, pois são primos
pares.
Portanto, os primos p2 = p − 2, p e p4 = p + 2 são trigêmeos, logo p2 = 3, o que implica p = 5.

5. Pelo Pequeno Teorema de Fermat, temos que a10 ≡ 1 mod 11, para todo a ∈ A = {1, 2, 3, 4, 5, 6, 7, 8, 9, 10}.
Portanto, [a]10 = [a10 ] = [1] para todo a ∈ A. Logo X 10 − [1] tem como raı́zes [1], [2], [3], [4], [5], [6], [7], [8], [9], [10] em
Z11 .

6. Devemos resolver o sistema:

X ≡ 1 mod 5, X ≡ 3 mod 7, X ≡ 5 mod 9.

Nas notações do Teorema Chinês dos Restos, Temos N = 5 · 7 · 9 = 315 e N1 = 63, N2 = 45 e N3 = 35.
As congruências

N1 Y ≡ 1 mod 5

N2 Y ≡ 1 mod 7

N3 Y ≡ 1 mod 9,

possuem as soluções y1 = 2, y2 = 5 e y3 = 8, respectivamente. Assim, pelo teorema, a única solução módulo N = 315
é dada por
x = N1 y1 + N2 y2 3 + N3 y3 5 = 2201.

Portanto, o menor número natural com a propriedade do problema é o resto da divisão de 2201 por 315, ou seja, 311.
OUTROS
MATERIAIS
MA14
ARITMÉTICA
MA 14 - Aritmética

Resumos das Unidades 1 e 2

Abramo Hefez

PROFMAT SBM
Unidade 1

Divisibilidade
O nosso objeto de estudo neste curso é o conjunto dos
números inteiros:

Z = {. . . , −2, −1, 0, 1, 2, . . .}.

Em Z há um subconjunto que se destaca, o conjunto dos


números naturais:

N = {1, 2, 3, . . .}.
Dados dois números inteiros quaisquer, é possı́vel somá-los,
subtraı́-los e multiplicá-los, mas nem sempre é possı́vel
dividir um pelo outro.
Só existe a Aritmética nos inteiros porque a divisão nem
sempre é possı́vel.
Diremos que um número inteiro a divide um número inteiro
b, escrevendo
a|b,
quando existir c ∈ Z tal que b = c · a.
Neste caso, diremos também que a é um divisor ou um fator
de b ou, ainda, que b é um múltiplo de a
Exemplos
• 1|0, pois 0 é múltiplo de 1: 0 = 0 · 1;
• −2|0, pois 0 é múltiplo de −2: 0 = 0 · (−2);
• 1|6, pois 6 é múltiplo de 1: 6 = 6 · 1;
• −1| − 6, pois −6 é múltiplo de −1: −6 = 6 · (−1);
• 2|6, pois 6 é múltiplo de 2: 6 = 3 · 2;
• −3|6, pois 6 é múltiplo de −3: 6 = (−2) · (−3).
Note que se a|b, com um jogo de sinais, é fácil mostrar que
±a| ± b.
A negação da sentença a | b é representada pelo sı́mbolo:
a 6 | b,
significando que não existe nenhum número inteiro c tal que
b = c · a.
Por exemplo, 3 6 | 4 e 2 6 | 5.
Suponha que a|b e seja c ∈ Z tal que b = c · a.
O número inteiro c é chamado de quociente de b por a e
b
denotado por c = .
a
Por exemplo,

0 0 6 −6
= 0, = 0, = 6, = 6,
1 −2 1 −1
6 6
= 3, = −2.
2 −3
Estabeleceremos a seguir algumas propriedades da
divisibilidade.
Proposição
Sejam a, b, c ∈ Z. Tem-se que
i) 1|a, a|a e a|0.
ii) se a|b e b|c, então a|c (Propriedade transitiva).
Demonstração: (i) Isto decorre das igualdades a = a · 1,
a = 1 · a e 0 = 0 · a.
(ii) a|b e b|c implica que existem f, g ∈ Z, tais que
b = f · a e c = g · b.
Substituindo o valor de b da primeira equação na outra,
obtemos
c = g · b = g · (f · a) = (g · f ) · a,
o que nos mostra que a|c.


O item (i) da proposição acima nos diz que todo número


inteiro é divisı́vel por 1 e por si mesmo.
Listaremos a seguir algumas propriedades da divisibilidade,
cujas provas são semelhantes às feitas acima.
Sejam a, b, c, d ∈ Z. Tem-se que
i) a|b e c|d =⇒ a · c|b · d;
ii) a|b =⇒ a · c|b · c;
iii) a|(b ± c) e a|b =⇒ a|c;
iv) a|b e a|c =⇒ a|(xb + yc), para todos x, y ∈ Z.
v) Se a, b ∈ N, tem-se que a|b =⇒ a 6 b.

É importante interiorizar as propriedades acima, pois elas


serão utilizadas a todo momento.
As proposições a seguir serão de grande utilidade.
Proposição
Sejam a, b ∈ Z e n ∈ N. Temos que a − b divide an − bn .
Demonstração: Vamos provar isto por indução sobre n.
A afirmação é obviamente verdadeira para n = 1, pois a − b
divide a1 − b1 = a − b.
Suponhamos, agora, que a − b|an − bn . Escrevamos

an+1 − bn+1 = aan − ban + ban − bbn = (a − b)an + b(an − bn ).

Como a − b|a − b e, por hipótese, a − b|an − bn , decorre da


igualdade acima e da Propriedade (iv) que
a − b|an+1 − bn+1 .
Estabelecendo assim o resultado para todo n ∈ N.


Proposição
Sejam a, b ∈ Z e n ∈ N. Temos que a + b divide
a2n+1 + b2n+1 .
Demonstração: Também por indução sobre n.
A afirmação é, obviamente, verdadeira para n = 0, pois a + b
divide a1 + b1 = a + b.
Suponhamos, agora, que a + b|a2n+1 + b2n+1 . Escrevamos

a2(n+1)+1 +b2(n+1)+1 = a2 a2n+1 −b2 a2n+1 +b2 a2n+1 +b2 b2n+1 =

(a2 − b2 )a2n+1 + b2 (a2n+1 + b2n+1 ).


Como a + b divide a2 − b2 = (a + b)(a − b) e, por hipótese,
a + b|a2n+1 + b2n+1 , decorre das igualdades acima e da
Propriedade (iv) que a + b|a2(n+1)+1 + b2(n+1)+1 .
Estabelecendo, assim, o resultado para todo n ∈ N.

Proposição
Sejam a, b ∈ Z e n ∈ N. Temos que a + b divide a2n − b2n .
Demonstração: Novamente, a prova se faz por indução
sobre n, nos mesmos moldes das provas das duas proposições
anteriores. Deixamos os detalhes por sua conta.


Exercı́cio

Vamos mostrar que o produto de i inteiros consecutivos é


divisı́vel por i!.
De fato, podemos escrever os i inteiros consecutivos como

n, n − 1, n − 2, . . . , n − (i − 1),

cujo produto P = n(n − 1)(n − 2) · · · (n − i + 1) é divisı́vel


por i!, já que
 
P n(n − 1)(n − 2) · · · (n − i + 1) n
= = ∈ N.
i! i! i
Como aplicação vamos mostrar que 6 divide todo número da
forma n(n + 1)(2n + 1), onde n ∈ N.

De fato,

n(n + 1)(2n + 1) = n(n + 1)(n + 2 + n − 1)


= n(n + 1)(n + 2) + n(n + 1)(n − 1).

Como cada uma das parcelas n(n + 1)(n + 2) e


n(n + 1)(n − 1) é o produto de três inteiros consecutivos,
elas são múltiplos de 3! = 6.
Portanto, sendo o número n(n + 1)(2n + 1) soma de dois
múltiplos de 6, ele é também múltiplo de 6.
Este fato não é surpreendente, pois sabemos que

n(n + 1)(2n + 1)
= 12 + 22 + 32 + · · · + n2 .
6
Exercı́cio

Vamos mostrar que 13 | 270 + 370 .

Note que 270 + 370 = 435 + 935 .

Como 35 é ı́mpar, temos que 4 + 9 divide 435 + 935 ,

o que mostra que 13 divide 270 + 370 .


UNIDADE 2

Divisão Euclidiana
Mesmo quando um número inteiro a não divide um número
inteiro b, Euclides (Século 3 a.C), nos seus Elementos,
utiliza, sem enunciá-lo explicitamente, o fato de que é
sempre possı́vel efetuar a divisão de b por a, com resto
pequeno.
Este resultado, de cuja justificativa geométrica damos uma
ideia quando a é natural, não só é um importante
instrumento na obra de Euclides, como também é um
resultado central da teoria elementar dos números.
Suponhamos que a ∈ N e consideremos a decomposição de N
em união de intervalos disjuntos:

N = . . . ∪ [−2a, −a) ∪ [−a, 0) ∪ [0, a) ∪ [a, 2a) ∪ . . .

Fica claro que qualquer número inteiro b pertence a um e


somente um desses intervalos.
Portanto, existe um único q ∈ Z tal que b ∈ [qa, qa + a),
ou seja, existem números inteiros únicos q e r tais que

b = qa + r, com 0 6 r < a.
Agora enunciamos o resultado geral:
Teorema (Divisão Euclidiana)
Sejam a e b dois números inteiros com a 6= 0. Existem dois
únicos números inteiros q e r tais que

b = a · q + r, com 0 6 r < |a|.

Nas condições do teorema, os números a e b são o divisor e o


dividendo, enquanto q e r são chamados, respectivamente,
de quociente e de resto da divisão de b por a.
Note que o resto da divisão de b por a é zero se, e somente
se, a divide b.
Exemplos

• Como 19 = 5 · 3 + 4, o quociente e o resto da divisão de 19


por 5 são q = 3 e r = 4.
• Como −19 = 5 · (−4) + 1 o quociente e o resto da divisão
de −19 por 5 são q = −4 e r = 1.
• O resto da divisão de 10n por 9 é sempre 1, qualquer que
seja o número natural n.
De fato, 9 = 10 − 1 divide 10n − 1n = 10n − 1. Assim,
10n − 1 = 9q, logo 10n = 9q + 1. Como 0 ≤ 1 < 9, pela
unicidade na divisão euclidiana, tem-se que o resto da
divisão de 10n por 9 é sempre 1.
Par ou ı́mpar?

Dado um número inteiro n ∈ Z qualquer, temos duas


possibilidades:
i) o resto da divisão de n por 2 é 0, isto é, existe q ∈ N tal
que n = 2q; ou
ii) o resto da divisão de n por 2 é 1, ou seja, existe q ∈ N tal
que n = 2q + 1.
No caso (i), dizemos que n é par e no caso (ii), dizemos que
n é ı́mpar.
Mais geralmente, fixado um número natural m > 2, pode-se
sempre escrever um número qualquer n, de modo único, na
forma n = mk + r, onde k, r ∈ Z e 0 6 r < m.
Por exemplo, todo número inteiro n pode ser escrito em
uma, e somente uma, das seguintes formas: 3k, 3k + 1, ou
3k + 2.
Ou ainda, todo número inteiro n pode ser escrito em uma, e
somente uma, das seguintes formas: 4k, 4k + 1, 4k + 2, ou
4k + 3.

Este último fato, permite mostrar que nenhum quadrado de


um número inteiro é da forma 4k + 3.
De fato, seja a ∈ Z.
• Se a = 4k, então a2 = 16k 2 = 4k 0 , onde k 0 = 4k 2 .
• Se a = 4k + 1, então a2 = 16k 2 + 8k + 1 = 4k 0 + 1, onde
k 0 = 4k 2 + 2k.
• Se a = 4k + 2, então a2 = 16k 2 + 16k + 4 = 4k 0 , onde
k 0 = 4k 2 + 4k + 1.
• Se a = 4k + 3, então a2 = 16k 2 + 48k + 9 = 4k 0 + 1, onde
k 0 = 4k 2 + 12k + 2.
Vamos aplicar este resultado para mostrar algo interessante:
Nenhum número da forma a = 11 . . . 1 (n algarismos iguais a
1, com n > 1) é um quadrado.
De fato, podemos escrever a = b · 100 + 11 = 4(25 · b + 2) + 3,
onde b = 11 . . . 1 (n − 2 algarismos iguais a 1). Logo, a é da
forma 4k + 3 e, portanto, não pode ser um quadrado.
Com esta técnica pode-se mostrar que nenhum número da
forma 11 . . . 1 é soma de dois quadrados. Deixamos isto
como exercı́cio
Aritmética – MA14 | Daniel Miranda Página 1 de 3

'DQLHO
'DQLHO0LUDQGD
&0&&Æ8)$%&

Î ,Q¯FLR

Î 1RWDVGH$XODV

Î 2SLQL·HV

Î 3HVTXLVD

Î 6RIWZDUHV0DWHP£WLFRV

Î &XUVRV

Î $ULWP«WLFDÆ0$

Î ƒOJHEUD/LQHDU

Î ƒOJHEUD/LQHDU$YDQ©DGD,

Î 'HPRQVWUD©·HVGR7HRUHPDGH-RUGDQ

Î ƒOJHEUD/LQHDU$YDQ©DGD,,

Î $Q£OLVH5HDO,

Î %DVHV0DWHP£WLFDV

Î &,8

Î (TXD©·HV'LIHUHQFLDLV3DUFLDLV

Î (YROX©¥RGRV&RQFHLWRV0DWHP£WLFRV

Î 5HIHU¬QFLDV&RPSOHPHQWDUHV

Î )XQ©·HVGHXPD9DUL£YHO

Î )XQ©·HVGH9DUL£YHLV&RPSOH[DV

Î *HRPHWULDÆ0$

Î *HRPHWULD$QDO¯WLFD

Î *HRPHWULD'LIHUHQFLDO 0HVWUDGR

Î *HRPHWULDQ¥R(XFOLGHDQD

Î ,QWURGX©¥RD3UREDELOLGDGHH(VWDW¯VWLFD

Î 6HT¾¬QFLDVH6«ULHV

$ULWP«WLFDÈ0$ 3ƒ*,1$6

Î 'DQLHO0LUDQGD0DFKDGR
b
Î 1RWDVGH$XODV

Î 2SLQL·HV
b/LYURbË(OHPHQWRVGH
b/LYUR $ULWP«WLFDÌGR+HIH]
Î 3HVTXLVD

/LVWDSDUDGH
/LVWDSDUDGH6HWHPEUR Î 6RIWZDUHV0DWHP£WLFRV

Î &XUVRV
Î S£J Î $ULWP«WLFDÆ0$
Î S£JÆ Î ƒOJHEUD/LQHDU
Î S£Jb Î ƒOJHEUD/LQHDU$YDQ©DGD,
Î S£J Î 'HPRQVWUD©·HVGR7HRUHPD
Î S£Jb GH-RUGDQ
Î S£J Î ƒOJHEUD/LQHDU$YDQ©DGD,,

Î $Q£OLVH5HDO,
/LVWDSDUDGH
/LVWDSDUDGHRXWXEUR
Î %DVHV0DWHP£WLFDV

Î &,8
Î S£Jbbb
Î (TXD©·HV'LIHUHQFLDLV3DUFLDLV
Î S£Jbbb
Î (YROX©¥RGRV&RQFHLWRV
Î S£Jbbbbb
0DWHP£WLFRV
Î S£Jbbbb

http://hostel.ufabc.edu.br/~daniel.miranda/?page_id=789 17/01/2014
Aritmética – MA14 | Daniel Miranda Página 2 de 3

Î S£Jbbbbb Î 5HIHU¬QFLDV&RPSOHPHQWDUHV

Î S£Jbbbbbb Î )XQ©·HVGHXPD9DUL£YHO

Î )XQ©·HVGH9DUL£YHLV&RPSOH[DV
/LVWDSDUDGH1RYHPEUR
Î *HRPHWULDÆ0$

Î *HRPHWULD$QDO¯WLFD
Î S£Jbbbbbbb
Î *HRPHWULD'LIHUHQFLDO
Î S£Jbb
0HVWUDGR
Î S£Jbb
Î *HRPHWULDQ¥R(XFOLGHDQD
Î S£Jbbb
Î ,QWURGX©¥RD3UREDELOLGDGHH
Î S£J
(VWDW¯VWLFD
Î S£Jb
Î 6HT¾¬QFLDVH6«ULHV
Î S£J

/LVWDSDUDb
/LVWDSDUDbGHGH]HPEUR

Î S£J

Î S£J

Î S£J
0DWHP£WLFD
0DWHP£WLFD8)$%&
Î S£J

Î S£J


b
9LVLWDUHVWHJUXSR

0(7$
$UTXLYRV
Î /RJLQ

Î 3RVWV566
DUTXLYR
Î 566GRVFRPHQW£ULRV

b Î :RUG3UHVVRUJ

b 3URXGO\SRZHUHGE\:RUG3UHVV  7KHPH

7UXO\0LQLPDOE\)ODUH7KHPHV

'HL[HXPDUHVSRVWD

2VHXHQGHUH©RGHHPDLOQ¥RVHU£SXEOLFDGR&DPSRVREULJDWµULRVV¥RPDUFDGRV

1RPH 

(PDLO 

6LWH

&$37&+$&RGH 

http://hostel.ufabc.edu.br/~daniel.miranda/?page_id=789 17/01/2014
Lista de exercı́cios de aritmética. 19/08/2011.

1. Demonstrar que, para todo número natural n, Mn = n(n2 − 1)(3n + 2) é múltiplo


de 24.

2. Demonstrar
! que, para quaisquer naturais n ≥ m, o coeficiente binomial
n n!
= é inteiro.
m m!(n − m)!

3. Encontre todos os inteiros positivos tais que:


(a) (n + 1)|(n3 − 1); (b) (2n − 1)|(n3 + 1); (c) (2n3 + 5)|(n4 + n + 1).

4. Demonstrar que (n − 1)2 |nk − 1 se, e somente se, (n − 1)|k.

5. Mostre que, para todo número natural n, o número n! + 1 admite um fator primo
p > n.

6. Prove que se um número natural p > 1 é um divisor de (p − 1)! + 1, então p é


primo.

7. Determinar no sistema decimal os seguintes números: (111111000000)2 , (325610)7


e (2ab39)12 onde a = 10 e b = 11.

8. Construir as tábuas de adição e de multiplicação par ao sistema de base 7 e


calcular:
(a) (1212)7 + (2356)7 + (42631)7 + (6235)7 ;
(b) (1654)7 · (2306)7 .

1
UNIVERSIDADE FEDERAL DE MATO GROSSO

INSTITUTO DE CIÊNCIAS EXATAS E DA TERRA

DEPARTAMENTO DE MATEMÁTICA

GRUPO DE ESTUDO – ALUNOS DO MESTRADO

MA 14 – ARITMÉTICA I – UNIDADES 10 E 11

UNIDADE 10 – EXPRESSÕES BINÔMIAIS - ATIVIDADES

1. Sejam a, m, n naturais. Mostre que an – 1 |am – 1 se, e somente se, n|m.

Resolução:

Se an – 1 |am – 1 temos que existe k natural tal que am – 1 = k(an – 1). Desta forma o temos
que (an – 1, am – 1) = (an – 1, k(an – 1)) = an – 1(1, k) = an – 1.

Da teoria temos também que (an – 1, am – 1) = a(n,m) – 1. Logo temos que:

(an – 1, am – 1) = a(n,m) – 1 = an – 1. Daí a(n,m) = an. Como a é natural temos que a igualdade é
satisfeita somente quando (m,n) = n e, portanto n|m.

Se n|m então existe k natural tal que m=kn. Logo (m,n) = (kn,n) = n. Da teoria sabemos
que:

(an – 1, am – 1) = a(n,m) – 1 = an – 1 e, portanto an – 1 |am – 1.

2. Sejam n, m naturais com n|m e . Se a é natural mostre que


(am + 1, an + 1) = an + 1

Resolução:
Se n|m, então existe k natural tal que m=kn. Desta forma temos que [m,n] = [kn,n] =
kn = m. De modo análogo observamos que (m,n) = n. Logo temos que que é

e ímpar. Logo pelo Corolário 9 temos que (am + 1, an + 1) = a(n,m) + 1 = an + 1.

3. Sejam a, m, n naturais, com m > n. Mostre que:

Resolução:

Seja w = 2m e r = 2n. Se m > n temos que existe k natural tal que m = n + k. Assim temos:

(w,r) = (2m, 2n) = (2n+k, 2n) = (2n.2k, 2n) = 2n(2k, 1) = 2n.

Agora observe que:

Logo pelo Corolário 10 temos que

, ,

4. Calcule:
a) (5202 + 1, 574 + 1)

Resolução:

Veja que (202,74) = 2

Logo [202,74] = 7474. Desta forma, temos que:

Logo pelo Corolário 9 temos que:

(5202 + 1, 574 + 1) = 5(202,74) + 1 = 52 + 1 = 26

b) (36497 + 1, 36210 + 1)

Resolução:

Veja que (497,210) = 7

Logo [497,210] = 14910. Desta forma, temos que:


Logo pelo Corolário 9 temos que:

(36497 + 1, 36210 + 1) = 1
c) (3144 – 1, 378 + 1)

Resolução:

Veja que (144,78) = 6

Agora observe que:

Logo pelo Corolário 10 temos que:

(3144 – 1, 378 + 1) = 36 + 1 = 730.


5. Seja (Mn)n a seqüência definida por Mn = 2n – 1. Mostre que 3|Mn se, e somente se, n
é par.

Resolução:

Como 3|Mn, segue que (3, Mn) = 3.

Agora observe que 3 = 22 – 1. Assim temos que:

(3, Mn) = (22 – 1, 2n – 1)

Desta forma necessariamente n é par.

Se n é par, então n = 2k e Mn= 22k – 1 = 4k – 1.

Afirmação 1: 3|4k – 1.

Mostraremos esta afirmação por indução.

Para k = 1 verdadeiro, pois 4 – 1 = 3.

Suponha válido para k = w, isto é, 3|4w – 1. Devemos provar verdadeiro para w + 1, isto é,
3|4w+1 – 1.

Observe que 4w+1 – 1 = 4w.4 – 1 = 4w.(3+1) – 1 = 3.4w + 4w – 1 = 3(4w + t) para algum t natural,
pois por hipótese 3|4w – 1. Logo 3|4w+1 – 1 e, portanto 3|Mn.
UNIDADE 11 – NÚMEROS DE FIBONACCI – ATIVIDADES

1. Mostre que, se na sequência de Fibonacci existir um termo divisível por um natural


m, então existem infinitos tais termos.

Resolução:

Seja k um natural tal que m|k (m natural). Desta forma temos que m|wk (w natural), ou seja,
se m divide k, segue que m divide todos os múltiplos de k. Como m|wk segue pelo Lema 2 que
um|uwk. Como os múltiplos de k são infinitos (pois os números naturais não são limitados
superiormente) segue que existem infinitos termos de Fibonacci que são divisíveis por m.

Para auxilio nos exercícios seguintes, construiremos os primeiros termos da sequencia de


Fibonacci.

Fn = {1, 1, 2, 3, 5, 8, 13, 21, 34, 55, 89, 144, 233, 377, ...}

2. Na sequência de Fibonacci, mostre que u m é par se, e somente se, m é divisível por 3.

Se um é par então, 2|um. Mas 2 = u3. Logo u3|um. Assim pelo Corolário 4 temos que 3|m.

Se m é divisível por 3, então 3|m. Pelo Corolário 4 temos que u3|um e, portanto um é par,
pois u3 = 2.

3. Na sequência de Fibonacci, mostre que um é divisível por 5 se, e somente se, m é


divisível por 5.

Resolução:

Se um é divisível por 5 então, 5|um. Mas 5 = u5. Logo u5|um. Assim pelo Corolário 4 temos
que 5|m.

Se m é divisível por 5, então 5|m. Pelo Corolário 4 temos que u5|um e, portanto um é
divisível por 5, pois u5 = 5.

4. Na sequência de Fibonacci, mostre que u m é divisível por 7 se, e somente se, m é


divisível por 8.

Resolução:

Se um é divisível por 7
Se m é divisível por 8, então 8|m. Pelo Corolário 4 temos que u8|um. Logo 21|um e,
portanto um é divisível por 7, pois todo múltiplo de 21 é em particular múltiplo de 7.

5. Na sequência de Fibonacci, mostre que u m é divisível por 4 se, e somente se, m é


divisível por 6.

Resolução:

Se um é divisível por 4

Se m é divisível por 6, então 6|m. Pelo Corolário 4 temos que u6|um. Logo 8|um e,
portanto um é divisível por 4, pois todo múltiplo de 8 é em particular múltiplo de 4.

Você também pode gostar